You are on page 1of 624

Civil Law Q&As (2007-2013)

hectorchristopher@yahoo.com
dbaratbateladot@gmail.com
A Compilation of the
Questions and Suggested Answers
In the
PHILIPPINE BAR EXAMINATIONS 2007-2013
In
CIVIL LAW
Compiled and Arranged By:
Baratbate-Ladot, Delight
Salise, Hector Christopher Jay-Arh Jr. M.
(University of San Jose-Recoletos School of Law)
ANSWERS TO BAR EXAMINATION QUESTIONS by the
UP LAW COMPLEX (2007-2013)
&
PHILIPPINE ASSOCIATION OF LAW SCHOOLS (2008)
Never Let The Odds Keep You From Pursuing What You Know In Your Heart You Were Me
ant To Do.-Leroy Satchel Paige
Page 1 of 180

Civil Law Q&As (2007-2013)


hectorchristopher@yahoo.com
dbaratbateladot@gmail.com
FOREWORD
This work is a compilation of the ANSWERS TO BAR
EXAMINATION QUESTIONS by the UP LAW COMPLEX ,
Philippine Association of Law Schools from 2007-2010 and
local law students and lawyers forum sites from 2011-2013
and not an original creation or formulation of the author.
The authors were inspired by the work of Silliman Universitys
College of Law and its students of producing a very good
material to everyone involved in the legal field particularly the
students and the reviewees for free. Hence, this work is a
freeware.
Everyone is free to distribute and mass produce copies of this
work, however, the author accepts no liability for the content of
this reviewer, or for the consequences of the usage, abuse, or
any actions taken by the user on the basis of the information
given.
The answers (views or opinions) presented in this reviewer are
solely those of the authors in the given references and do not
necessarily represent those of the authors of this work.
The Authors.
Never Let The Odds Keep You From Pursuing What You Know In Your Heart You Were Me
ant To Do.-Leroy Satchel Paige
Page 2 of 180

Civil Law Q&As (2007-2013)


hectorchristopher@yahoo.com
dbaratbateladot@gmail.com
TABLE OF CONTENTS
(Titles are based on Sillimans Compilation [Arranged by Topic])
Persons
Capacity: Juridical Capacity (2008).............................................
..................................12
Capacity; Juridical Capacity of Donee; Requisites for Acceptance (2012).........
..............12
Capacity: Legal Capacity; Lex Rei Sitae (2007)..................................
.............................13
Correction of Entries; Clerical Error Act (2008)................................
.............................14
Nationality Principle (2009)....................................................
.......................................14
Nationality Principle; Change of Name not Covered (2009)........................
.....................15
Conflict of Laws
Processual Presumption (2009)...................................................
...................................16
Jurisdiction; Courts may Assume Jurisdiction over Conflict of Laws Cases (2010).
........17
Adoption
Adoption; Termination; Death of Adopter (2009)..................................
..........................17
Adoption; Illegitimate Child (2010).............................................
...................................18
Adoption; Illegitimate Child; Use of Mothers Surname as Middle Name (2012).......
.......19
Consent; Consent of the Adopters Heirs (2008)....................................
.........................19
Qualifications of Adopter (2010)................................................
....................................20
Never Let The Odds Keep You From Pursuing What You Know In Your Heart You Were Me
ant To Do.-Leroy Satchel Paige
Page 3 of 180

Civil Law Q&As (2007-2013)


hectorchristopher@yahoo.com
dbaratbateladot@gmail.com
Family Code
Marriage; Annulment; Grounds (2009).............................................
..............................20
Marriage; Annulment; Grounds (2007).............................................
..............................21
Marriage; Annulment; Parties (2012).............................................
.................................22
Marriage; Annulment; Support Pendente Lite (2010)...............................
.......................22
Marriage; Divorce Decrees; Filipino Spouse Becoming Alien (2009)................
................23
Marriage; Divorce Decrees; Foreign Spouse Divorces Filipino Spouse (2012).......
...........24
Marriage; Divorce Decrees; Foreign Spouse Divorces Filipino Spouse (2010).......
...........25
Marriage; Legal Separation; Prescription (2012).................................
............................25
Marriage; Legal Separation; Prescription (2007).................................
............................26
Marriage; Psychological Incapacity (2013).......................................
..............................26
Marriage; Psychological Incapacity (2012).......................................
..............................28
Marriage; Requisites (2008).....................................................
......................................28
Marriage; Subsequent Marriage (2008)............................................
...............................29
Marriage; Void Marriages; By Reason of Public Policy (2008).....................
.....................30
Marriage; Void Marriages; By Reason of Public Policy (2007).....................
.....................30
Marriage; Void Marriages; Property Relations (2009).............................
.........................30
Marriage; Void Marriages; Status of Children (2009).............................
..........................31
Parental Authority; Illegitimate Minor Child (2009).............................
..........................32
Parental Authority; In Vitro Fertilization (2010)...............................
.............................32
Never Let The Odds Keep You From Pursuing What You Know In Your Heart You Were Me
ant To Do.-Leroy Satchel Paige
Page 4 of 180

Civil Law Q&As (2007-2013)


hectorchristopher@yahoo.com
dbaratbateladot@gmail.com
Paternity & Filiation; Child Born Under a Void Marriage (2010)..................
..................33
Paternity & Filiation; Impugning Legitimacy (2010)..............................
.........................34
Paternity & Filiation; In Vitro Fertilization;
Surrogate Mothers Remedy to Regain Custody (2010)...................34
Paternity & Filiation; Legitimacy; Presumption (2008)...........................
.......................35
Paternity & Filiation; Legitimation of a Child from a Previous Valid Marriage (2
008).....36
Paternity & Filiation; Legitimation of a Dead Child (2009)......................
.......................37
Paternity & Filiation; Support: Ascendants & Descendants;
Collateral Blood Relatives (2008)...............................................
....37
Paternity & Filiation; Use of Surname; Illegitimate Child (2009)................
....................38
Paternity & Filiation; Who May Impugn Legitimacy (2009).........................
...................39
Property Relations; Adulterous Relationship (2009)..............................
.........................39
Property Relations; Accession (2012)............................................
.................................40
Property Relations; Ante-Nuptial Debt (2007)....................................
............................40
Property Relations; Unions Without Marriage (2012)..............................
........................41
Property Relations; Unions Without Marriage (2012)..............................
........................42
Property Relations; Void Marriages (2010).......................................
..............................43
Property Relations; Void Marriages (2010).......................................
..............................45
Never Let The Odds Keep You From Pursuing What You Know In Your Heart You Were Me
ant To Do.-Leroy Satchel Paige
Page 5 of 180

Civil Law Q&As (2007-2013)


hectorchristopher@yahoo.com
dbaratbateladot@gmail.com
Succession
Disposition; Mortis Causa vs. Intervivos; Corpse (2009).........................
........................46
Heirs; Fideicommissary Substitution (2008)......................................
............................46
Heirs; Intestate Succession; Legitime; Computation (2010).......................
....................47
Heirs; Representation; Iron-Curtain Rule (2012).................................
...........................49
Heirs; Reserva Troncal (2009)...................................................
.....................................49
Intestate Succession (2008).....................................................
......................................50
Intestate Succession (2008).....................................................
......................................51
Intestate Succession; Rights of Representation:
Illegitimate, Adopted Child; Iron Curtain Rule (2007)....................51
Legitimes; Compulsory Heirs (2012)..............................................
................................53
Legitime; Compulsory Heirs (2008)...............................................
.................................53
Preterition; Disinheritance (2008)..............................................
...................................54
Succession; Proof of Death between persons called to succeed each other (2008)..
........55
Succession; Rule on Survivorship (2009).........................................
..............................56
Wills; Holographic Wills; Insertions & Cancellations (2012).....................
.......................57
Wills; Holographic Wills; Probate (2009)........................................
.................................57
Wills; Joint Wills (2008).......................................................
..........................................59
Wills; Joint Wills; Probate (2012)..............................................
.....................................59
Never Let The Odds Keep You From Pursuing What You Know In Your Heart You Were Me
ant To Do.-Leroy Satchel Paige
Page 6 of 180

Civil Law Q&As (2007-2013)


hectorchristopher@yahoo.com
dbaratbateladot@gmail.com
Wills; Prohibition to Partition of a Co-Owned Property (2010)...................
.....................60
Wills; Notarial Wills; Blind Testator; Requisites (2008)........................
...........................61
Wills; Testamentary Disposition; Period to Prohibit Partition (2008)............
..................61
Wills; Witnesses to a Will, Presence required; Thumbmark as Signature (2007)....
..........62
Donation
Donations; Formalities; In Writing (2007).......................................
...............................63
Donations; Illegal & Impossible Conditions (2007)...............................
..........................64
Donation; Inter Vivos (2013)....................................................
......................................64
Property
Accretion; Alluvium (2008)......................................................
......................................65
Accretion; Rights of the Riparian Owner (2009)..................................
...........................67
Builder; Good Faith; Requisites (2013)..........................................
.................................68
Easement; Prescription; Acquisitive Prescription (2009).........................
.......................70
Easement; Right of Way (2013)...................................................
...................................70
Easement; Right of Way (2010)...................................................
...................................72
Hidden Treasure (2008)..........................................................
.......................................73
Mortgage; Public or Private Instrument (2013)...................................
............................74
Occupation vs. Possession (2007)................................................
..................................76
Ownership; Co-Ownership (2009)..................................................
.................................76
Ownership; Co-Ownership (2008)..................................................
.................................77
Never Let The Odds Keep You From Pursuing What You Know In Your Heart You Were Me
ant To Do.-Leroy Satchel Paige
Page 7 of 180

Civil Law Q&As (2007-2013)


hectorchristopher@yahoo.com
dbaratbateladot@gmail.com
Property; Movable or Immovable (2007)...........................................
..............................78
Land Titles and Deeds
Acquisition of Lands; Sale of Real Property to an Alien (2009)..................
.....................79
Non-Registrable Properties (2007)...............................................
...................................80
Prescription; Acquisitive Prescription (2008)...................................
..............................81
Prescription; Judicially Foreclosed Real Property Mortgage (2012)...............
.................82
Purchaser in Good Faith; Mortgaged Property (2008)..............................
.......................83
Registration; Governing Law (2007)..............................................
.................................84
Registration; Party Who First took Possession (2013)............................
........................85
Registration; Requisites; Proof (2013)..........................................
..................................86
Remedies; Fraud; Rights of Innocent Purchaser (2009)............................
......................89
Contracts
Contract to Sell vs. Conditional Contract of Sale (2012)........................
........................90
Rescission of Contract; Fortuitous Event (2008).................................
...........................90
Stipulation; Arbitration Clause (2009)..........................................
..................................91
Obligations
Extinguishment; Compensation (2009).............................................
.............................91
Extinguishment; Compensation (2008).............................................
.............................92
Extinguishment; Novation (2008).................................................
.................................93
Extinguishment; Payment of Check (2013).........................................
...........................94
Never Let The Odds Keep You From Pursuing What You Know In Your Heart You Were Me
ant To Do.-Leroy Satchel Paige
Page 8 of 180

Civil Law Q&As (2007-2013)


hectorchristopher@yahoo.com
dbaratbateladot@gmail.com
Extinguishment; Payment of Check; Legal Tender (2008)...........................
...................95
Liability; Solidary Liability (2008)............................................
......................................96
Obligations; Without Agreement (2007)...........................................
..............................97
Trust
Trust De Son Tort (2007)........................................................
.......................................98
Sales
Condominium Act; Partition of a Condominium (2009)..............................
....................99
Mortgage; Equitable Mortgage (2012).............................................
................................99
Option Contract; Liquor & Pulutan as Consideration (2013).........................
.............100
Right of First Refusal; Lessee; Effect (2008)...................................
..............................101
Lease
Builder; Good Faith; Useful Improvements (2013).................................
.......................103
Lease; Caveat Emptor (2009).....................................................
..................................104
Agency
Agency; Sale of a Real Property through an Agent (2010).........................
....................104
Partnership
Liability; Liability of a Partner (2010)........................................
..................................105
Oral Partnership (2009).........................................................
......................................106
Share; Demand during the Existence of Partnership (2012)........................
..................107
Never Let The Odds Keep You From Pursuing What You Know In Your Heart You Were Me
ant To Do.-Leroy Satchel Paige
Page 9 of 180

Civil Law Q&As (2007-2013)


hectorchristopher@yahoo.com
dbaratbateladot@gmail.com
Commodatum & Mutuum
Mutuum; Interest; Solutio Indebiti (2012).......................................
.............................107
Guaranty
Guaranty (2009).................................................................
..........................................108
Surety
Surety (2010)...................................................................
............................................108
Pledge
Pledge; Pactum Commissorium (2009)..............................................
...........................109
Torts and Damages
Damages (2012)..................................................................
.........................................109
Damages; Moral & Exemplary (2009)...............................................
.............................110
Damages; Public Officers acting in the Performance of their Duties (2012).......
...........111
Death Indemnity (2009)..........................................................
.....................................112
Doctrine of Discovered Peril (Last Clear Chance) (2007).........................
......................112
Liability; Owner of a Pet; Fortuitous Event (2010)..............................
.........................113
Liability; Special Parental Authority (2010)....................................
.............................115
Quasi-Delict; Claims; Requisites (2013).........................................
...............................115
Quasi Tort (2010)...............................................................
..........................................116
Never Let The Odds Keep You From Pursuing What You Know In Your Heart You Were Me
ant To Do.-Leroy Satchel Paige
Page 10 of 180

Civil Law Q&As (2007-2013)


hectorchristopher@yahoo.com
dbaratbateladot@gmail.com
MULTIPLE CHOICE QUESTIONS
2013 Civil Law Exam MCQ (October 13, 2013)..............118
2012 Civil Law Exam MCQ (October 14, 2012).................130
2011 Civil Law Exam MCQ (November 13, 2011)..............149
2010 Civil Law Exam MCQ (September 12, 2010).............176
2007 Civil Law Exam MCQ (September 09, 2007).............179
Never Let The Odds Keep You From Pursuing What You Know In Your Heart You Were Me
ant To Do.-Leroy Satchel Paige
Page 11 of 180

Civil Law Q&As (2007-2013)


hectorchristopher@yahoo.com
Persons
dbaratbateladot@gmail.com
mother s womb. However, if the fetus
had an intra-uterine life of less than
seven months, it is not deemed born if it
Capacity: Juridical Capacity (2008)
dies within twenty-four (24) hours after
No. II. At age 18, Marian found out that she
its complete delivery from the maternal
was pregnant. She insured her own life and
womb. The act of naming the unborn
named
sole
child as sole beneficiary in the insurance
beneficiary. When she was already due to
is favorable to the conceived child and
give birth, she and her boyfriend Pietro, the
therefore the fetus acquires presumptive
father
were
or provisional personality. However, said
kidnapped in a resort in Bataan where they
presumptive personality only becomes
were vacationing. The military gave chase
conclusive if the child is born alive. The
and after one week, they were found in an
child need not survive for twenty-four
abandoned hut in Cavite. Marian and Pietro
(24) hours as required under Art. 41 of
were hacked with bolos. Marian and the

the Code because "Marian was already


baby delivered were both found dead, with
due to give birth," indicating that the
the baby s umbilical cord already cut. Pietro
child was more than seven months old.
her
unborn
of
her
child
unboarn
as
her
child,
survived.
(A). Can Marian s baby be the beneficiary of
the insurance taken on the life of the
mother? (2%)
Capacity; Juridical Capacity of Donee;
Requisites for Acceptance (2012)
No.I. b) Ricky donated P 1 Million to the
SUGGESTED ANSWER:
unborn child of his pregnant girlfriend,
Yes, the baby can be the beneficiary of
which she accepted. After six (6) months of
the life insurance of Marian. Art. 40 NCC
pregnancy, the fetus was born and baptized
provides
as Angela. However, Angela died 20 hours
that
personality;
but

"birth
the
determines
conceived
child
after birth. Ricky sought to recover the P 1
shall be considered born for all purposes
Million.
that are favorable to it, provided that it
Explain. (5%)
be
born
later
with
the
conditions
specified in Art. 41. Article 41 states
Is
Ricky
entitled
to
recover?
SUGGESTED ANSWER:
that "for civil purposes, the fetus shall
Yes, Ricky is entitled to recover the
be considered born if it is alive at the
P1,000,000.00. The NCC considers a
time it is completely delivered from the
Never Let The Odds Keep You From Pursuing What You Know In Your Heart You Were Me
ant To Do.-Leroy Satchel Paige
Page 12 of 180

Civil Law Q&As (2007-2013)


hectorchristopher@yahoo.com
a
dbaratbateladot@gmail.com
fetus a person for purposes favorable to
acquire
house
in
Australia
because
it provided it is born later in accordance
Australian Laws allow aliens to acquire
with the provision of the NCC. While the
property from the age of 16.
donation is favorable to the fetus, the
donation did not take effect because the
fetus was not born in accordance with
the NCC.
SUGGESTED ANSWER:
TRUE. Since Australian Law allows alien
to acquire property from the age of 16,
To be considered born, the fetus that
Roberta may validly own a house in
had an intrauterine life of less than
Australia, following the principle of lex
seven (7) months should live for 24
rei sitae enshrined in Art. 16, NCC,
hours from its complete delivery from
which states "Real property as well as
the mothers womb. Since Angela had an
personal property is subject to the law of
intrauterine life of less than seven (7)
the

months but did not live for 24 hours, she


Moreover,
was not considered born and, therefore,
capacity of
did not become a person. Not being a
contract in Australia is governed by
person, she has no juridical capacity to
Philippine
be a donee, hence, the donation to her
ownership
did not take effect. The donation not
until the contract is annulled.
being effective, the amount donated may
be recovered. To retain it will be unjust
enrichment.
country
where
even
over
is
assuming
Roberta
Law,
it
that
legal
in entering the
she
the
situated."
will
property

acquire
bought
ALTERNATIVE ANSWER:
FALSE. Laws relating to family rights
and duties, or to the status, condition or
legal capacity of persons are binding
Capacity: Legal Capacity; Lex Rei Sitae
upon the citizens of the Philippines,
(2007)
even though living abroad (Art. 15, NCC).
The age of majority under Philippine law
No.VII. Write "TRUE" if the statement is
is 18 years (R.A. No. 6809); hence,
true or "FALSE" if the statement is false. If
Roberta, being only 17 years old, has no
the statement is FALSE, state the reason.
legal capacity to acquire and own land.
(2% each).
(1). Roberta, a Filipino, 17 years of age,
without the knowledge of his parents, can
Never Let The Odds Keep You From Pursuing What You Know In Your Heart You Were Me
ant To Do.-Leroy Satchel Paige
Page 13 of 180

Civil Law Q&As (2007-2013)


hectorchristopher@yahoo.com
dbaratbateladot@gmail.com
Correction of Entries; Clerical Error Act
the Rules of Court because said changes
(2008)
are substantive corrections.
No. IV. Gianna was born to Andy and
(B).
Aimee, who at the time Gianna s birth were
administrative proceedings be brought for
not married to each other. While Andy was
the
single at the time, Aimee was still in the
corrections? (2%)
Instead
of
purpose
a
of
judicial
action,
making
the
can
above
process of securing a judicial declaration of
nullity on her marriage to her ex-husband.
Gianna s birth certificate, which was signed
by both Andy and Aimee, registered the
status
of
Gianna

as
"legitimate",
her
surname carrying that of Andy s and that
her parents were married to each other.
SUGGESTED ANSWER:
No. An administrative proceeding cannot
be brought for the purpose of making the
above corrections. R.A. 9048, otherwise
known as the Clerical Error Act, which
authorizes the city or municipal civil
(A). Can a judicial action for correction of
registrar or the consul general to correct
entries in Gianna s birth certificate be
a clerical or typographical error in an
successfully maintained to:
entry and/or change the first name or
nickname in the civil register without
a). Change her status from "legitimate" to
need of a judicial order. Errors that
"illegitimate" (1%);
involve the change of nationality, age,
status, surname or sex of petitioner are
and
not included from the coverage of the
b). Change her surname from that of Andy s
to Aimee s maiden surname? (1%)
SUGGESTED ANSWER:
entries in Gianna s birth certificate can
be successfully maintained to change (a)
status
from
"legitimate"
to
"illegitimate," and (b) her surname from
that
of

Andy s
to
174689, 22 Oct., 2007).
Nationality Principle (2009)
Yes, a judicial action for correction of
her
said Act (Silverio v. Republic, G.R. No.
Aimee s
maiden
surname in accordance with Rule 108 of
No.XII. Emmanuel and Margarita, American
citizens and employees of the U.S. State
Department, got married in the African
state of Kenya where sterility is a ground
for annulment of marriage. Thereafter, the
spouses were assigned to the U.S. Embassy
in Manila. On the first year of the spouses
Never Let The Odds Keep You From Pursuing What You Know In Your Heart You Were Me
ant To Do.-Leroy Satchel Paige
Page 14 of 180

Civil Law Q&As (2007-2013)


hectorchristopher@yahoo.com
dbaratbateladot@gmail.com
tour of duty in the Philippines, Margarita
filed an annulment case against Emmanuel
ALTERNATIVE ANSWER:
before a Philippine court on the ground of
The
her husbands sterility at the time of the
action for the annulment of marriage
celebration of the marriage.
solemnized elsewhere but only when the
forum
has
jurisdiction
over
an
party bringing the actions is domiciled
(A). Will the suit prosper? Explain your
in the forum. In this case, none of the
answer. (3%)
parties to the marriage is domiciled in
the
SUGGESTED ANSWER:
No, the suits will not prosper. As applied
to foreign nationals with the respect to
family relations and status of persons,
the nationality principle set forth in
Article 15 of the Civil Code will govern
the
relations
Margarita.
of
Since
Emmanuel

they
are
Philippines.
They
are
here
as
officials of the US Embassy whose stay in
the country is merely temporary, lasting
only during their fixed tour of duty.
Hence, the Philippine courts have no
jurisdiction over the action.
and
American
citizens, the governing law as to the
ground for annulment is not Kenyan Law
Nationality Principle; Change of Name
not Covered (2009)
which Magarita invokes in support of
No.XX. (A). If Ligaya, a Filipino citizen
sterility as such ground; but should be
residing in the United States, files a petition
U.S. Law, which is the national Law of
for change of name before the District Court
both
of New York, what law shall apply? Explain.
Emmanuel
and
Margarita
as
recognized under Philippine Law. Hence,
(2%)
the Philippine court will not give due

course to the case based on Kenyan Law.


SUGGESTED ANSWER:
The nationality principle as expressed in
New York law shall apply. The petition of
the application of national law of foreign
change of name file din New York does
nationals
by
Philippine
courts
is
not concern the legal capacity or status
precedents
(Pilapil
v.
of the petitioner. Moreover, it does nto
653[1989],
affect the registry of any other country
Garcia v. Recio, 366 SCRA 437 [2001],
including the country of birth of the
Llorente v. Court of Appeals 345 SCRA
petitioner.
92 [2000], and Bayot v. Court of Appeals
rendered in that petition will have effect
570 SCRA 472 [2008]).
only in New York. The New York court
established
Ibay-Somera,
by
174
SCRA

Whatever
judgment
is
Never Let The Odds Keep You From Pursuing What You Know In Your Heart You Were Me
ant To Do.-Leroy Satchel Paige
Page 15 of 180

Civil Law Q&As (2007-2013)


hectorchristopher@yahoo.com
dbaratbateladot@gmail.com
cannot, for instance, order the Civil
petitioner and his transactions in the
Registrar in the Philippines to change its
Philippines. The Philippine court can
records. The judgment of the New York
never
court allowing a change in the name of
custodian in the US of the records of the
the petitioner will be limited to the
petitioner. Moreover, change of name
records of the petitioner in New York
has nothing to do with the legal capacity
and the use of her new name in all
or status of the alien. Since Philippine
transactions in New York. Since the
records and transactions are the only
records and processes in New York are
ones affected, the Philippine court may
the only ones affected, the New York
effect the change only in accordance
court
with the laws governing those records
will
apply
New
YorK
law

in
acquire
jurisdiction
over
the
resolving the petition.
and transactions that law cannot be but
ALTERNATIVE ANSWER:
Philippine law.
Philippine law shall apply (Art 15, NCC).
ALTERNATIVE ANSWER:
Status, conditions, family rights and
U.S. law shall apply as it is his national
duties are governed by Philippine laws as
law. This is pursuant to the application
to
of
Filipinos
even
though
sojourning
lex
patriae
or
the
nationality
abroad.
principle, by which his legal status is
ALTENATIVE ANSWER:
governed by national law, the matter of

If Ligaya, a Filipino, files a petition for


change of name being included in the
change of name with the District Court
legal status. The Supreme Court has
of New YoRk, the laws of New York will
reiterate in several cases, that the lex
govern since change of name is not one
patriae as provided in Article 15 of the
of those covered by the principles of
Civil
nationality.
nationals
Code
is
in
applicable
to
determining
foreign
their
legal
status (supra).
(B). If Henry, an American citizen residing
in the Philippines, files a petition for change
Conflict of Laws
of name before a Philippine court, what law
shall apply? Explain. (2%)
Processual Presumption (2009)
SUGGESTED ANSWER:
Philippine law will apply. The petition
for change of name in the Philippines
will
affect

only
the
records
of
the
No.I. TRUE or FALSE. Answer TRUE if the
statement
is
true,
or
FALSE
if
the
statement is false. Explain your answer in
not more than two (2) sentences.
Never Let The Odds Keep You From Pursuing What You Know In Your Heart You Were Me
ant To Do.-Leroy Satchel Paige
Page 16 of 180

Civil Law Q&As (2007-2013)


(A).
The
hectorchristopher@yahoo.com
doctrine
of
dbaratbateladot@gmail.com
"processual
(1) Public Order. To maintain peace and
presumption" allows the court of the forum
order, disputes that disturb the peace of
to presume that the foreign law applicable
the forum should be settled by the court
to the case is the same as the local or
of the forum even though the application
domestic law. (1%)
of the foreign law is necessary for the
purpose.
SUGGESTED ANSWER:
TRUE. If the foreign law necessary to the
(2) Humanitarian Principle. An aggrieved
resolve an issue is not proven as a fact,
party should not be left without remedy
the court of the forum may presume that
in a forum even though the application
the foreign law is the same as the law of
of the foreign law by the courts of the
the forum.
forum is unavoidable in order to extend
relief.
Jurisdiction;
Courts

may
Adoption
Assume
Jurisdiction over Conflict of Laws Cases
(2010)
Adoption; Termination; Death of Adopter
(2009)
No.III. Define, Enumerate or Explain. (2%
each)
No.XIII. Rafael, a wealthy bachelor, filed a
petition for the adoption of Dolly, a one(C) Give at least two reasons why a court
year old foundling who had a severe heart
may assume jurisdiction over a conflict of
ailment.
laws case.
adoption proceedings, Rafael died of natural
During
the
pendency
of
the
causes. The Office of the Solicitor General
files a motion to dismiss the petition on the
SUGGESTED ANSWER:
(1) Statute theory. There is a domestic
law
authorizing
the
local
court
to
assume jurisdiction.
(2)

Comity
assumes
theory.
ground that the case can no longer proceed
because of the petitioners death.
(A). Should the case be dismissed? Explain.
The
jurisdiction
local
based
principle of comity or courtesy.
ALTERNATIVE ANSWER:
on
court
the
(2%)
SUGGESTED ANSWER:
It
depends
on
the
stage
of
the
proceedings when Rafael died. If he died
after all the requirements under the law
Never Let The Odds Keep You From Pursuing What You Know In Your Heart You Were Me
ant To Do.-Leroy Satchel Paige
Page 17 of 180

Civil Law Q&As (2007-2013)


hectorchristopher@yahoo.com
have been complied with and the case is
dbaratbateladot@gmail.com
Adoption; Illegitimate Child (2010)
already submitted for resolution, the
court may grant the petition and issue a
No.VIII. Spouses Rex and Lea bore two
decree of adoption despite the death of
children now aged 14 and 8. During the
the
8552).
subsistence of their marriage, Rex begot a
Otherwise, the death of the petitioner
child by another woman. He is now 10
shall have the effect terminating the
years of age.
adopter
(Section
13,
RA
proceedings.
On Leas discovery of Rexs fathering a child
(B). Will your answer be the same if it was
by another woman, she filed a petition for
Dolly who died during the pendency of the
legal separation which was granted.
adoption proceedings? Explain. (2%)
Rex now wants to adopt his illegitimate
child.
SUGGESTED ANSWER:
No, if it was Dolly who died, the case

should
be
terminates
dismissed.
the
Her
death
proceedings (Art. 13,
Domestic Adoption Law).
(A) Whose consent is needed for Rexs
adoption of his illegitimate child? (2.5%)
SUGGESTED ANSWER:
ALTERNATIVE ANSWER:
The
It depends. If all the requirements under
legitimate child, of the 10- year -old
the law have already been complied with
illegitimate child and of the biological
and the case is already submitted for
mother of the illegitimate child are
resolution, the death of the adoptee
needed for the adoption (Section 7 and
should not abate the proceedings. The
9, RA 8552). The consent of Lea is no
court
longer
should
issue
the
decree
of
adoption if will be for the best interest of

consent
required
of
the
14-year-old
because
there
was
already a final decree of legal separation.
the adoptee. While RA8552 provides only
for the case where it is the petitioner
(B) If there was no legal separation, can Rex
who dies before the decree is issued, it is
still adopt his illegitimate child? Explain.
with more compelling reason that the
(2.5%)
decree should be allowed in case it is the
adoptee who dies because adoption is
primarily for his benefit.
SUGGESTED ANSWER:
Yes, he can still adopt his illegitimate
child but with the consent of his spouse,
Never Let The Odds Keep You From Pursuing What You Know In Your Heart You Were Me
ant To Do.-Leroy Satchel Paige
Page 18 of 180

Civil Law Q&As (2007-2013)


hectorchristopher@yahoo.com
dbaratbateladot@gmail.com
of his 14-year-old legitimate child, of the
Adoption of Stephanie Nathy Astorga
illegitimate child, and of the biological
Garcia, G.R. No. 148311, March 31,
mother of the illegitimate child (Section
2005, the Supreme Court ruled that the
7 and 9, RA 8552).
adopted child may use the surname of
the natural mother as his middle name
because there is no prohibition in the
Adoption;
Mothers
Illegitimate
Surname
as
Child;
Use
Middle
of
Name
(2012)
minor
illegitimate
child
Stephanie,
alleging that Stephanies mother is Gemma
Astorga Garcia; that Stephanie has been
using
for the benefit of the adopted child who
shall
preserve

his
lineage
on
his
mothers side and reinforce his right to
No.IV.b) Honorato filed a petition to adopt
his
law against it. Moreover, it will also be
her
mothers
middle
name
and
inherit from his mother and her family.
Lastly, it will make the adopted child
conform with the time-honored Filipino
tradition
of
carrying
the
mothers
surname as the persons middle name.
surname; and that he is now a widower and
qualified to be her adopting parent. He
prayed that Stephanies middle name be
Consent; Consent of the Adopters Heirs
changed from "Astorga" to "Garcia," which
(2008)
is her mothers surname and that her
surname "Garcia" be changed to "Catindig,"
No.V. Despite several relationships with
which is his surname. This the trial court
different

denied. Was the trial court correct in


unmarried.
denying Hororatos request for Stephanies
Brenda produced a daughter, Amy, now 30
use of her mothers surname as her middle
years old. His second, with Carla, produced
name? Explain. (5%)
two sons: Jon and Ryan. His third, with
women,
His
Andrew
first
remained
relationship
with
Donna, bore him no children although
SUGGESTED ANSWER:
No, the trial court was not correct. There
is no law prohibiting an illegitimate child
adopted by his natural father to use as
middle name his mothers surname. The
law is silent as to what middle name an
adoptee may use. In case of In re:
Elena has a daughter Jane, from a previous
relationship. His last, with Fe, produced no
biological
children
but
they
informally
adopted without court proceedings, Sandy s
now 13 years old, whom they consider as
their own. Sandy was orphaned as a baby
and was entrusted to them by the midwife
who attended to Sandy s birth. All the
Never Let The Odds Keep You From Pursuing What You Know In Your Heart You Were Me
ant To Do.-Leroy Satchel Paige
Page 19 of 180

Civil Law Q&As (2007-2013)


hectorchristopher@yahoo.com
children, including Amy, now live with
dbaratbateladot@gmail.com
Qualifications of Adopter (2010)
andrew in his house.
No.IX. Eighteen-year old Filipina Patrice
(A). Is there any legal obstacle to the legal
had a daughter out of wedlock whom she
adoption of Amy by Andrew? To the legal
named
adoption of Sandy by Andrew and Elena?
American citizen John who brought her to
(2%)
live with him in the United States of
Laurie.
America.
John
At
at
26,
Patrice
once
married
signified
his
SUGGESTED ANSWER:
willingness to adopt Laurie.
Yes, there is a legal obstacle to the legal
Can John file the petition for adoption? If
adoption of Amy by Andrew. Under Sec.

yes, what are the requirements? If no, why?


9(d) of RA 8552, the New Domestic
(5%)
Adoption
Act
consent
of
of
1998,
the
the
written
illegitimate
sons/daughters, ten (10) years of age or
over, of the adopter, if living with said
adopter and the latter s spouse, if any, is
necessary
to
the
adoption.
All
the
children of Andrew are living with him.
Andrew needs to get the written consent
of Jon, Ryan, Vina and Wilma, who are
all ten (10) years old or more. Sandy s
consent
to
Amy s
adoption
is
SUGGESTED ANSWER:
No, John cannot file the petition to
adopt
alone.

Philippine
law
requires
husband and wife to adopt jointly except
on certain situations enumerated in the
law. The case of John does not fall in
any of the exceptions (R.A. 8552).
Family Code
not
necessary because she was not legally
Marriage; Annulment; Grounds (2009)
adopted by Andrew. Jane s consent is
likewise not necessary because she is
No.XII. Emmanuel and Margarita, American
not a child of Andrew. Sandy, an orphan
citizens and employees of the U.S. State
since birth, is eligible for adoption under
Department, got married in the African
Sec. 8(f) of RA 8552, provided that
state of Kenya where sterility is a ground
Andrew obtains the written consent of
for annulment of marriage. Thereafter, the
the other children mentioned above,
spouses were assigned to the U.S. Embassy
including Amy and Elena obtains the
in Manila. On the first year of the spouses
written consent of Jane, if she is over
tour of duty in the Philippines, Margarita
ten years old (Sec. 9(d), RA 8552).
filed an annulment case against Emmanuel
before a Philippine court on the ground of
Never Let The Odds Keep You From Pursuing What You Know In Your Heart You Were Me
ant To Do.-Leroy Satchel Paige

Page 20 of 180

Civil Law Q&As (2007-2013)


hectorchristopher@yahoo.com
dbaratbateladot@gmail.com
her husbands sterility at the time of the
shall be determined by applying Kenyan
celebration of the marriage.
law and not Philippine law.
(B). Assume Emmanuel and Margarita are
However, while Kenyan law governs the
both
in
formal validity of the marriage, the legal
up
capacity of the Filipino parties to the
residence in the Philippines. Can their
marriage is governed not by Kenyan law
marriage be annulled on the ground of
but by Philippine law (Article 15, NCC).
Emmanuels sterility? Explain. (3%)
Sterility of a party as a ground for the
Filipinos.
Kenya,
they
After
come
their
back
wedding
and
take

annulment of the marriage is not a


SUGGESTED ANSWER:
No, the marriage cannot be annulled
under the Philippine law. Sterility is not
a ground for annulment of marriage
under Article 45 of the Family Code.
must
apply
Phillippine
law
in
determining the status of the marriage
legal capacity of the Filipino parties.
No, the marriage cannot be annulled in
the Philippines.
Philippine
capacity. Hence, the Philippine court
on the ground of absence or defect in the
ALTERNATIVE ANSWER:
The
matter of form but a matter of legal
Since
sterility
does
not
constitute
absence or defect in the legal capacity of
the parties under Philippine law, there is
court
shall
have
jurisdiction over the action to annul the
marriage not only because the parties
no
ground

to
avoid
or
annul
the
marriage. Hence, the Philippine court
has to deny the petition.
are residents of the Philippines but
because they are Filipino citizens. The
Philippine court, however, shall apply
the law of the place where the marriage
Marriage; Annulment; Grounds (2007)
was celebrated in determining its formal
No. VII. Write "TRUE" if the statement is
validity (Article 26, FC; Article 17, NCC).
true or "FALSE" if the statement is false. If
the statement is FALSE, state the reason.
Since the marriage was celebrated in
(2% each).
Kenya in accordance with Kenyan law,
the formal validity of such marriage is
(4). The day after John and Marsha got
governed by Kenyan law and any issue as
married,
to the formal validity of that marriage
impotent. Marsha continued to live with
John
told
her
that
he
was
Never Let The Odds Keep You From Pursuing What You Know In Your Heart You Were Me

ant To Do.-Leroy Satchel Paige


Page 21 of 180

Civil Law Q&As (2007-2013)


hectorchristopher@yahoo.com
dbaratbateladot@gmail.com
John for 2 years. Marsha is now estopped
No, I do not agree. There are others who
from filing an annulment case against
may file a petition for declaration of
John.
nullity such as the other spouse in
bigamous marriages.
SUGGESTED ANSWER:
FALSE. Marsha is not estopped from
filing an annulment case against John
Marriage; Annulment; Support Pendente
on the ground of his impotence, because
Lite (2010)
she learned of his impotence after the
celebration of the marriage and not
No.V. G filed on July 8, 2000 a petition for
before.
to
declaration of nullity of her marriage to B.
consummate is a valid ground for the
During the pendency of the case, the couple
annulment of marriage if such incapacity
entered into a compromise agreement to
was existing at the time of the marriage,
dissolve
continues and appears to be incurable.
property. B ceded his right to their house
The marriage may be annulled on this
and lot and all his shares in two business

ground
firms to G and their two children, aged 18
Physical
within
incapacity
five
years
from
its
their
absolute
community
of
and 19.
celebration.
B also opened a bank account in the
amount of P3 million in the name of the two
children to answer for their educational
Marriage; Annulment; Parties (2012)
expenses until they finish their college
No.IX.b) A petition for declaration of nullity
degrees.
of a void marriage can only be filed by
either the husband or the wife? Do you
For her part, G undertook to shoulder the
agree? Explain your answer. (5%)
day-to-day living expenses and upkeep of
the
I
agree.
Under
The
Court

approved
the
spouses agreement on September 8, 2000.
SUGGESTED ANSWER:
Yes,
children.
the
rules
(A) Suppose the business firms suffered
promulgated by the Supreme Court, a
reverses, rendering G unable to support
direct action for declaration of nullity
herself and the children. Can G still ask for
may only be filed by any of the spouses.
support pendente lite from B? Explain. (3%)
ALTERNATIVE SUGGESTED ANSWER:
Never Let The Odds Keep You From Pursuing What You Know In Your Heart You Were Me
ant To Do.-Leroy Satchel Paige
Page 22 of 180

Civil Law Q&As (2007-2013)


hectorchristopher@yahoo.com
Marriage;
SUGGESTED ANSWER:
If B acquiesces and does not file the
dbaratbateladot@gmail.com
Divorce
Decrees;
Filipino
Spouse Becoming Alien (2009)
action to impugn the legitimacy of the
No.IV. Harry married Wilma, a very wealthy
child within the prescriptive period for
woman. Barely five (5) years into the
doing so in Article 170 of the Family
marriage, Wilma fell in love with Joseph.
Code, G s daughter by another man shall
Thus, Wilma went to a small country in
be
Europe, became a naturalized citizen of
conclusively
presumed
as
the
legitimate daughter of B by G.
that country, divorced Harry, and married
(B) Suppose in late 2004 the two children
had squandered the P3 million fund for
their education before they could obtain
Joseph.
Joseph
A

year
thereafter,
returned
Wilma
and
and
established
permanent residence in the Philippines.
their college degrees, can they ask for more
(A). Is the divorce obtained by Wilma from
support from B? Explain. (3%)
Harry
recognized
in
the
Philippines?
Explain your answer. (3%)
SUGGESTED ANSWER:
Yes, the two children can still ask for
SUGGESTED ANSRWER :
support for schooling or training for
some professions,
trade
or
vocation,
even beyond the
they shall have
their education
2, Family Code;
Phil.
634
{1954}].Their
having

age of majority until


finished or completed
(Article 194, Paragraph
Javier v. Lucero, 94

squandered the money given to them for


their education will not deprive them of
their right to complete an education, or
to
extinguish
the
obligation
of
the
parents to ensure the future of their
children.
As to Wilma, the divorced obtained by
her
is
recognized
Philippines
as
because
valid
she
is
in
the
now
a
foreigner. Philippine personal laws do
not
apply
to
a
foreigner.
However,
recognition of the divorce as regards
Harry will depend on the applicability to

his case of the second paragraph of


Article 26 of the Family Code. If it is
applicable, divorce is recognized as to
him and, therefore, he can remarry.
However, if it is not applicable, divorce
is
not
recognized
as
to
him
and,
consequently, he cannot remarry.
ALTERNATIVE ANSWER:
Never Let The Odds Keep You From Pursuing What You Know In Your Heart You Were Me
ant To Do.-Leroy Satchel Paige
Page 23 of 180

Civil Law Q&As (2007-2013)


hectorchristopher@yahoo.com
Yes , the divorce obtained by Wilma is
dbaratbateladot@gmail.com
SUGGESTED ANSWER :
recognized as valid in the Philippines. At
the time she got the divorce, she was
Yes, he can validly marry Elizabeth,
already a foreign national having been
applying the doctrine laid down by the
naturalized as a citizen of that small
Supreme Court in Republic v. Obrecido
country in Europe. Based on precedents
(427 SCRA 114 [2005]). Under the second
established by the Supreme Court (
paragraph of Article 26 of the Family
Bayot v. CA, 570 SCRA 472 [2008]),
Code, for the Filipino spouse to have
divorce
is
capacity to remarry, the law expressly
recognized in the Philippines if validly
requires the spouse who obtained the
obtained in accordance with his or her
divorce to be a foreigner at the time of
national law .
the marriage. Applying this requirement
obtained
by
a
foreigner

to the case of Harry it would seem that


(B). If Harry hires you as his lawyer, what
he is not given the capacity to remarry.
legal recourse would you advise him to
This is because Wilma was a Filipino at
take? Why? (2%)
the time of her marriage to Harry.
SUGGESTED ANSWER:
In Republic v. Obrecido, however, the
Supreme Court ruled that a Filipino
I will advice Harry to:
spouse is given the capacity to remarry
(1) Dissolve and liquidate his property
relations with Wilma ; and
the recognition and enforcement of the
judgment
of
divorced
the divorce was a Filipino at the time of
the marriage, if the latter was already a
(2) If he will remarry, file a petition for
foreign
even though the spouse who obtained
(Rule
39,Rules of Court ).
(C). Harry tells you that he has fallen in
love with another woman, Elizabeth, and
wants to marry her because, after all,
foreigner when the divorce was already
obtained abroad. According to the court,
to rule otherwise will violate the equal
protection clause of the Constitution.
Marriage;
Divorce
Decrees;
Foreign

Spouse Divorces Filipino Spouse (2012)


Wilma is already married to Joseph. Can
(b) Cipriano and Lady Miros married each
Harry legally marry Elizabeth? Explain.
other. Lady Miros then left for the US and
(2%)
there, she obtained American citizenship.
Never Let The Odds Keep You From Pursuing What You Know In Your Heart You Were Me
ant To Do.-Leroy Satchel Paige
Page 24 of 180

Civil Law Q&As (2007-2013)


Cipriano
later
hectorchristopher@yahoo.com
learned
all
about
dbaratbateladot@gmail.com
this
proving only that the foreign spouse has
including the fact that Lady Miros has
obtained a divorce against her or him
divorced him in America and that she had
abroad. (1%)
remarried there. He then filed a petition for
authority to remarry, invoking Par. 2, Art.
26
of
the
Family
Code.
Is
Cipriano
capacitated to re-marry by virtue of the
divorce decree obtained by his Filipino
spouse who was later naturalized as an
American citizen? Explain. (5%)
SUGGESTED ANSWER :
FALSE, In Garcia v. Recio , 366 SCRA
437 (2001) , the SC held
Filipino
spouse
to
that for a

have
capacity
to
contract a subsequent marriage, it must
also be proven that the foreign divorced
obtained abroad by the foreigner spouse
SUGGESTED ANSWER:
give such foreigner spouse capacity to
Yes, he is capacitated to remarry. While
the second paragraph of Art 26 of the
Family Code is applicable only to a
remarry.
ALTERNATIVE ANSWER:
Filipino who married a foreigner at the
TRUE, Art 26 (2) (FC), clearly provides
time of marriage, the Supreme Court
that the decree of divorce obtained
ruled in the case of Republic v. Orbecido,
abroad
G.R. No. 154380, 5 Oct, 2005, that the
sufficient
said
spouse to remarry.
provision
equally
applies
to
a
by
the
to
foreigner
capacitate

spouse
the
is
Filipino
Filipino who married another Filipino at
the time of the marriage, but who was
already a foreigner when the divorce was
Marriage; Legal Separation; Prescription
obtained.
(2012)
No.IV.a)
Marriage;
Divorce
Decrees;
Foreign
Spouse Divorces Filipino Spouse (2010)
No.I. True or False.
(A). Under Article 26 of the Family Code,
when a foreign spouse divorces his/her
Filipino spouse, the latter may re-marry by
After
they
got
married,
Nikki
discovered that Christian was having an
affair with another woman.
But
Nikki
decided to give it a try and lived with him
for two (2) years. After two (2) years, Nikki
filed an action for legal separation on the
ground of Christians sexual infidelity. Will
the action prosper? Explain. (5%)
Never Let The Odds Keep You From Pursuing What You Know In Your Heart You Were Me
ant To Do.-Leroy Satchel Paige
Page 25 of 180

Civil Law Q&As (2007-2013)


hectorchristopher@yahoo.com
dbaratbateladot@gmail.com
the sexual infidelity committed in 2003,
SUGGESTED ANSWER:
Although the action for legal separation
has not yet prescribed, the prescriptive
period being 5 years, if Obecidos affair
with another woman was ended when
the prescriptive period runs from 2003
and
so
on.
The
action
for
legal
separation for the last act of sexual
infidelity in 2005 will prescribe in 2010.
Nikki decided to live with him again,
Nikkis
action
account
of
will
not
condonation.
prosper
on
However,
if
such affair is still continuing, Nikkis
Marriage;
Psychological

Incapacity
(2013)
action would prosper because the action
will surely be within five (5) years from
No.I. You are a Family Court judge and
the commission of the latest act of
before you is a Petition for the Declaration
sexual infidelity. Every act of sexual
of Nullity of Marriage (under Article 36 of
liaison is a ground for legal separation.
the Family Code)filed by Maria against Neil.
Maria claims that Neil is psychologically
incapacitated to comply with the essential
Marriage; Legal Separation; Prescription
(2007)
obligations of marriage because Neil is a
drunkard, a womanizer, a gambler, and a
mama s boy- traits that she never knew or
No.VII. Write "TRUE" if the statement is
true or "FALSE" if the statement is false. If
the statement is FALSE, state the reason.
(2% each).
saw when Neil was courting her. Although
summoned, Neil did not answer Maria s
petition and never appeared in court.
To support her petition, Maria presented
(2). If a man commits several acts of sexual
infidelity, particularly in 2002, 2003, 2004,
2005, the prescriptive period to file for legal
three witnesses- herself, Dr. Elsie Chan,
and Ambrosia. Dr. Chan testified on the
psychological
report
on
Neil
that
she
prepared. Since Neil never acknowledged

separation runs from 2002.


n9r
responded
to
her
invitation
for
interviews, her report is solely based on her
SUGGESTED ANSWER:
interviews with Maria and the spouses
FALSE. The five-year prescriptive period
minor children. Dr. Chan concluded that
for filing legal separation runs from the
Neil
occurrence
infidelity
Personality Disorder, an ailment that she
committed in 2002 runs from 2002, for
found to be already present since Neil s
of
sexual
is
suffering
from
Narcissistic
Never Let The Odds Keep You From Pursuing What You Know In Your Heart You Were Me
ant To Do.-Leroy Satchel Paige
Page 26 of 180

Civil Law Q&As (2007-2013)


hectorchristopher@yahoo.com
dbaratbateladot@gmail.com
early adulthood and one that is grave and
mere conclusions. Being a drunkard, a
incurable. Maria testified on the specific
womanizer, a gambler and a mamas boy,
instances when she found Neil drunk, with
merely shows Neils failure to perform
another
the
his marital obligations. In a number of
family s resources in a casino. Ambrosia,
cases, the Supreme Court did not find
the
the existence of psychological incapacity
woman,
spouses
or
current
squandering
household
help,
corroborated Maria s testimony.
in cases where the respondent showed
habitual
drunkenness
(Republic
v.
On the basis of the evidence presented, will
Melgar, G.R. No. 139676, 2006), blatant

you grant the petition? (8%)


display of infidelity and irresponsibility
(Dedel v. CA, 2004) or being hooked to
SUGGESTED ANSWER:
gambling and drugs (Republic v. TanyagSan Jose, G.R. No. 168328, 2007).
No. The petition should be denied.
The psychological incapacity under Art.
36
of
the
Family
Code
must
be
characterized by (a) gravity, (b) juridical
ALTERNATIVE ANSWER:
Yes. The petition should be granted.
antecedence, and (c) incurability. It is
The personal medical or psychological
not enough to prove that the parties
examination
failed to meet their responsibilities and
requirement
duties as married persons; it is essential
psychological
that they must be shown to be incapable
totality of the evidence presented which
of doing so, due to some physiological
shall
(not physical) illness (Republic v. CA and
psychological
Molina, G.R. No. 108763, Feb 13, 1997).

Marcos, G.R. No. 136490, Oct 19, 2000).


In this case, the pieces of evidence
presented are not sufficient to conclude
that
indeed
Neil
psychological
Personality
before
the
is
suffering
incapacity
Disorder]
marriage,
from
[Narcissistic
existing
already
incurable
Dr.
of
respondent
for
Chans
Marias
therefore,
the
incapacity
report
and
not
declaration
incapacity.

determine
is
It
of
is
the
existence
of
(Marcos
v.
corroborated
Ambrosias
sufficiently
a
by
testimonies,
prove
Neils
psychological incapacity to assume his
marital obligations.
and
serious enough to prevent Neil from
performing
his
essential
marital
obligations. Dr. Chans report contains
Never Let The Odds Keep You From Pursuing What You Know In Your Heart You Were Me
ant To Do.-Leroy Satchel Paige
Page 27 of 180

Civil Law Q&As (2007-2013)


Marriage;
hectorchristopher@yahoo.com
Psychological
Incapacity
(2012)
dbaratbateladot@gmail.com
the said report is the only evidence of
respondents psychological incapacity.
No.II.b) The petitioner filed a petition for
declaration of nullity of marriage based
Marriage; Requisites (2008)
allegedly on the psychological incapacity of
the respondent, but the psychologist was
No. III. Roderick and Faye were high school
not
the
sweethearts. When Roderick was 18 and
respondent and the psychological report
Faye, 16 years old, they started to live
was
of
together as husband and wife without the
be
benefit of marriage. When Faye reached 18
able
to
based
petitioner.
personally
only
on

Should
examine
the
the
narration
annulment
years of age, her parents forcibly took her
granted? Explain. (5%)
back and arranged for her marriage to
Brad. Although Faye lived with Brad after
SUGGESTED ANSWER:
The annulment cannot be guaranteed
solely on the basis of the psychological
report. For the report to prove the
psychological
respondent,
it
incapacity
of
the
is
that
the
required
psychologist should personally examine
the respondent and the psychological
report
should
be
based
on
the
psychologists independent assessment
of the facts as to whether or not the
respondent

is
psychologically
incapacitated.
Since,
the
the
marriage,
Roderick
continued
to
regularly visit Faye while Brad was away at
work. During their marriage, Faye gave
birth to a baby girl, Laica. When Faye was
25 years old, Brad discovered her continued
liason with Roderick and in one of their
heated arguments, Faye shot Brad to death.
She lost no time in marrying her true love
Roderick,
without
a
marriage
license,
claiming that they have been continuously
cohabiting for more than 5 years.
(A). Was the marriage of Roderick and Faye
psychologist
did
not
personally examine the respondent, and
valid? (2%)
SUGGESTED ANSWER:
his report is based solely on the story of
the petitioner who has an interest in the
No. The marriage of Roderick and Faye is
outcome of the petition, the marriage
not valid. Art. 4, FC provides that the

cannot be annulled on the ground of


absence of any of the essential or formal
respondents psychological incapacity if
requisites renders the marriage void ab
initio.
However,
no
license
shall
be
Never Let The Odds Keep You From Pursuing What You Know In Your Heart You Were Me
ant To Do.-Leroy Satchel Paige
Page 28 of 180

Civil Law Q&As (2007-2013)


hectorchristopher@yahoo.com
dbaratbateladot@gmail.com
necessary for the marriage of a man and
reqiured to submit the required certificate
a woman who have lived together as
of capacity to marry from the German
husband and wife for at least 5 years and
Embassy in Manila, Adolf stated in the
without any legal impediment to marry
application for marriage license that he was
each other. In Republic v. Dayot, G.R.
a Filipino citizen. With the marriage license
No. 175581, 28 March 2008, reiterating
stating that Adolf was a Filipino, the couple
the doctrine in Nial v. Bayadog, G.R.
got married in a ceremony officiated by the
No. 133778, 14 March 2000, this fiveParish Priest of Calamba, Laguna in a
year
beach in Nasugbu, Batangas, as the local
period
exclusivity
is
and
characterized
continuity.
by
In
the
parish

priest
refused
to
solemnize
present case, the marriage of Roderick
marriages except in his church. Is the
and Faye cannot be considered as a
marriage valid? Explain fully. (5%)
marriage
of
exceptional
character,
because there were 2 legal impediments
during their cohabitation: minority on
the part of Faye, during the first two
years of cohabitation; and, lack of legal
capacity, since Faye married Brad at the
age of 18. The absence of a marriage
license made the marriage of Faye and
Roderick void ab initio.
SUGGESTED ANSWER:
No. The marriage is not valid. Art. 41 FC
allows the present spouse to contract a
subsequent
marriage
during
the
subsistence of his previous marriage
provided that: (a) his prior spouse in the
first marriage had been absent for four
consecutive years; (b) that the spouse
present has a well-founded belief that
the absent spouse was already dead, and
Marriage; Subsequent Marriage (2008)
(C) present spouse instituted a summary
No. I. Ana Rivera had a husband, a Filipino
proceeding for the declaration of the
citizen

the
presumptive death of absent spouse.
passengers on board a commercial jet plane
Otherwise, the second marriage shall be
which crashed in the Atlantic Ocean ten
null and void. In the instant case, the
(10) years earlier and had never been heard
husband
of ever since. Believing that her husband
passengers on board a commercial jet
had died, Ana married Adolf Cruz Staedtler,
plane which crashed in the Atlantic
a divorced German national born of a
Ocean.
German
father
mother
husband was not recovered to confirm
residing
in
being
his death. Thus, following Art. 41, Ana
like
her,
who
and
a
Stuttgart.
was
among

Filipino
To
avoid
of
The
Ana
body
was
of
among
the
the
deceased
Never Let The Odds Keep You From Pursuing What You Know In Your Heart You Were Me
ant To Do.-Leroy Satchel Paige
Page 29 of 180

Civil Law Q&As (2007-2013)


should
have
first
hectorchristopher@yahoo.com
secured
a
judicial
dbaratbateladot@gmail.com
while Jane is a child of Elena from a
declaration of his presumptive death
previous
before she married Adolf. The absence of
marriage is not one of the prohibited
the
marriages enumerated under Art. 38 of
said
judicial
declaration
incapacitated Ana from contracting her
relationship.
Thus,
their
the FC.
second marriage, making it void ab
initio.
Marriage; Void Marriages; By Reason of
Marriage; Void Marriages; By Reason of
Public Policy (2007)
Public Policy (2008)
No. VII. Write "TRUE" if the statement is
true or "FALSE" if the statement is false. If

No.V. Despite several relationships with


the statement is FALSE, state the reason.
different
(2% each).
unmarried.
women,
His
Andrew
first
remained
relationship
with
Brenda produced a daughter, Amy, now 30
(5). Amor gave birth to Thelma when she
years old. His second, with Carla, produced
was 15 years old. Thereafter, Amor met
two sons: Jon and Ryan. His third, with
David and they got married when she was
Donna, bore him no children although
20 years old. David had a son, Julian, with
Elena has a daughter Jane, from a previous
his ex-girlfriend Sandra. Julian and Thelma
relationship. His last, with Fe, produced no
can get married.
biological
children
but
they
informally
adopted without court proceedings, Sandy s
now 13 years old, whom they consider as
their own. Sandy was orphaned as a baby

and was entrusted to them by the midwife


who attended to Sandy s birth. All the
children, including Amy, now live with
andrew in his house.
(D). Can Jon and Jane legally marry? (1%)
SUGGESTED ANSWER:
TRUE.
Julian
and
Thelma
can
get
married. Marriage between stepbrothers
and
stepsisters
are
not
among
the
marriages prohibited under the Family
Code.
Marriage;
Void
Marriages;
Property
SUGGESTED ANSWER:
Relations (2009)
Yes. Jon and Jane can marry each other;
No. III. In December 2000, Michael and
Jon is an illegitimate child of Andrew
Anna, after obtaining a valid marriage
Never Let The Odds Keep You From Pursuing What You Know In Your Heart You Were Me
ant To Do.-Leroy Satchel Paige
Page 30 of 180

Civil Law Q&As (2007-2013)


hectorchristopher@yahoo.com
dbaratbateladot@gmail.com
license, went to the Office of the Mayor of
impediment for them to validity marry
Urbano,
each other.
Bulacan, to get married. The
Mayor was not there, but the Mayors
secretary asked Michael and Anna and
their witnesses to fill up and sign the
Marriage;
required
Children (2009)
marriage
contract
forms.
The
Void
Marriages;
Status
of
secretary then told them to wait, and went
out to look for the Mayor who was attending
No. III. In December 2000, Michael and
a wedding in a neighboring municipality.
Anna, after obtaining a valid marriage
license, went to the Office of the Mayor of
When the secretary caught up with the
Urbano,
Mayor
she

Mayor was not there, but the Mayors


showed him the marriage contract forms
secretary asked Michael and Anna and
and told him that the couple and their
their witnesses to fill up and sign the
witnesses were waiting in his office. The
required
Mayor forthwith signed all the copies of the
secretary then told them to wait, and went
marriage
out to look for the Mayor who was attending
at
the
wedding
contract,
gave
reception,
them
to
the
secretary who returned to the Mayors
Bulacan, to get married. The
marriage
contract
forms.
The
a wedding in a neighboring municipality.
office. She then gave copies of the marriage
contract to the parties, and told Michael
When the secretary caught up with the
and Anna that they were already married.

Mayor
Thereafter, the couple lived together as
showed him the marriage contract forms
husband and wife, and had three sons.
and told him that the couple and their
at
the
wedding
reception,
she
witnesses were waiting in his office. The
(C). What property regime governs the
Mayor forthwith signed all the copies of the
properties acquired by the couple? Explain.
marriage
(2%)
secretary who returned to the Mayors
contract,
gave
them
to
the
office. She then gave copies of the marriage
SUGGESTED ANSWER:
contract to the parties, and told Michael
and Anna that they were already married.
The marriage being void, the property
Thereafter, the couple lived together as
relationship that governed their union is
husband and wife, and had three sons.
special co-ownership under Article 147
of the Family Code. This is on the

(A). Is the marriage of Michael and Anna


assumption
valid,
that
there
was
no
voidable,
or
void?
Explain
your
answer. (3%)
Never Let The Odds Keep You From Pursuing What You Know In Your Heart You Were Me
ant To Do.-Leroy Satchel Paige
Page 31 of 180

Civil Law Q&As (2007-2013)


hectorchristopher@yahoo.com
SUGGESTED ANSWER :
dbaratbateladot@gmail.com
(C). When Rona reaches seven (7) years old,
she tells Rodolfo that she prefers to live
The marriage is void because the formal
with
requisite
financially than Nanette. If Rodolfo files an
of
marriage
ceremony
was
absent ( Art.3, F.C. 209, Family Code).
him,
because
he
is
better
off
action for the custody of Rona, alleging that
he is Ronas choice as custodial parent, will
ALTERNATIVE ANSWER:
The
marriage
is
void
the court grant Rodolfos petition? Why or
because
an
why not? (2%)

essential requisite was absent: consent


of
the
parties
freely
given
in
the
SUGGESTED ANSWER:
presence of the solemnizing officer (Art
No, because Rodolfo has no parental
.2, FC).
authority over Rona. He who has the
parental
authority
has
the
right
to
(B). What is the status of the three children
custody. Under the Family Code, the
of Michael and Anna? Explain your answer.
mother alone has parental authority over
(2%)
the illegitimate child. This is true even if
illegitimate father recognized the child
SUGGESTED ANSWER:
and even though he is giving support for
The children are illegitimate, having
the child. To acquire custody over Rona,
been born outside a valid marriage.
Rodolfo should first deprive Nanette of

parental authority if there is ground


under the law, and in a proper court
Parental Authority; Illegitimate Minor
proceedings. In the same action, the
Child (2009)
court may award custody of Rona to
No.XIV. Rodolfo, married to Sharon, had an
Rodolfo if it is for her best interest.
illicit affair with his secretary, Nanette, a
19-year old girl, and begot a baby girl,
Rona. Nanette sued Rodolfo for damages:
actual, for hospital and other medical
expenses
in
delivering
No.VI. Gigolo entered into an agreement
caesarean section; moral, claiming that
with Majorette for her to carry in her womb
Rodolfo
her,
his baby via in vitro fertilization. Gigolo
representing that he was single when, in
undertook to underwrite Majorettes prefact, he was not; and exemplary, to teach a
natal expenses as well as those attendant
to
child
(2010)
by
promised
the
Parental Authority; In Vitro Fertilization
marry

lesson to like-minded Lotharios.


Never Let The Odds Keep You From Pursuing What You Know In Your Heart You Were Me
ant To Do.-Leroy Satchel Paige
Page 32 of 180

Civil Law Q&As (2007-2013)


hectorchristopher@yahoo.com
dbaratbateladot@gmail.com
to her delivery. Gigolo would thereafter pay
in Paraaque. After four (4) years or in
Majorette P2 million and, in return, she
2001,
would give custody of the baby to him.
college degree as a fulltime student, she
and
After Majorette gives birth and delivers the
G
B
having
completed
contracted
her
marriage
4-year
without
a
license.
baby to Gigolo following her receipt of P2
million, she engages your services as her
The marriage of B and G was, two years
lawyer to regain custody of the baby.
later, declared null and void due to the
absence of a marriage license.
(C) Who of the two can exercise parental
authority over the child? Explain. (2.5%)
(B). Is Venus legitimate, illegitimate, or
legitimated? Explain briefly. (3%)
SUGGESTED ANSWER:
Majorette,

the
mother,
can
exercise
SUGGESTED ANSWER:
parental authority. Since the child was
Venus is illegitimate. She was conceived
born
is
and born outside a valid marriage. Thus,
illegitimate and the mother has the
she is considered illegitimate (Art 165,
exclusive parental authority and custody
Family
over the child.
legitimated by the subsequent marriage
out
of
wedlock,
the
child
ALTERNATIVE ANSWER:
Code).
While
Venus
was
of her parents, such legitimation was
rendered
ineffective
when

the
said
Gigolo can exercise parental authority
marriage was later on declared null and
over the child. Majorette has no blood
void
relation to the child. She is just a
license.
carrier of the child.
due
to
absence
of
a
marriage
Under Article 178 of the Family Code,
legitimation
subsequent
shall
valid
take
place
marriage
by
a
between
Paternity & Filiation; Child Born Under a
parents. The annulment of a voidable
Void Marriage (2010)
marriage
shall
legitimation.

not
The
affect
inclusion
the
of
the
No.X. In 1997, B and G started living
underscored
together without the benefit of marriage.
necessarily implies that the Article s
The relationship produced one offspring,
application
Venus. The couple acquired a residential lot
marriages.
portion
is
It
in
limited
follows
the
to
that
Article
voidable
when
the
Never Let The Odds Keep You From Pursuing What You Know In Your Heart You Were Me
ant To Do.-Leroy Satchel Paige
Page 33 of 180

Civil Law Q&As (2007-2013)


hectorchristopher@yahoo.com
dbaratbateladot@gmail.com
subsequent marriage is null or void, the
action to impugn, B can pray for the
legitimation must also be null and void.
correction of the status of the said
In the present problem, the marriage
daughter in her record of birth.
between B and G was not voidable but
void. Hence, Venus has remained an
(B). If B acquiesces to the use of his
illegitimate child.
surname by Gs daughter by another man,
what is/are the consequence/s? Explain.
(5%)
Paternity
&
Filiation;
Impugning
Legitimacy (2010)
SUGGESTED ANSWER:
If B acquiesces and does not file the
two
action to impugn the legitimacy of the
serious
child within the prescriptive period for
spouses
doing so in Article 170 of the Family
continued to live under one roof. B begot a
Code, G s daughter by another man shall
son by another woman. G also begot a

be
daughter by another man.
legitimate daughter of B by G.
No.IV.
Spouses
offsprings.
B
Albeit
personality
and
they
differences,
G
begot
had
the
conclusively
presumed
as
the
(A). If G gives the surname of B to her
daughter by another man, what can B do to
protect their legitimate children s interests?
Explain. (5%)
Paternity
&
Fertilization;
Filiation;
In
Surrogate
Vitro
Mothers
Remedy to Regain Custody (2010)

SUGGESTED ANSWER:
No.VI. Gigolo entered into an agreement
B can impugn the status of G s daughter
with Majorette for her to carry in her womb
by
his baby via in vitro fertilization. Gigolo
another
daughter
on
man
the
as
his
ground
legitimate
that
for
undertook to underwrite Majorettes prebiological reason he could not have been
natal expenses as well as those attendant
the father of the child, a fact that may
to her delivery. Gigolo would thereafter pay
be proven by the DNA test. Having been
Majorette P2 million and, in return, she
born during the marriage between B and
would give custody of the baby to him.
G, G s daughter by another man is
presumed as the child of B under Article
After Majorette gives birth and delivers the
164 of the Family Code. In the same
baby to Gigolo following her receipt of P2

Never Let The Odds Keep You From Pursuing What You Know In Your Heart You Were Me
ant To Do.-Leroy Satchel Paige
Page 34 of 180

Civil Law Q&As (2007-2013)


hectorchristopher@yahoo.com
dbaratbateladot@gmail.com
million, she engages your services as her
No, he cannot. Both he and Majorette are
lawyer to regain custody of the baby.
guilty of violating the provision of the
Anti-Child Abuse Law (RA7610) on child
(A) What legal action can you file on behalf
trafficking. Being in pari delicto, the
of Majorette? Explain. (2.5%)
partners shall be left where they are and
Gigolo cannot demand the return of
SUGGESTED ANSWER:
what he paid.
As her lawyer, I can file a petition for
habeas corpus on behalf Majorette to
ALTERNATIVE ANSWER:
recover custody of her child. Since she is
Yes. The agreement between Gigolo and
the mother of the child that was born
Majorette is a valid agreement.
out
of
wedlock,
she
has
exclusive
parental authority and custody over the
(D) Is the child entitled to support and
child. Gigolo, therefore, has no right to
inheritance from Gigolo? Explain. (2.5%)

have custody of the child and his refusal


to give up custody will constitute illegal
SUGGESTED ANSWER:
detention for which habeas corpus is the
If Gigolo voluntarily recognized the child
proper remedy.
as his illegitimate child in accordance
ALTERNATIVE ANSWER:
with Article 175 in relation to Article
172 of the Family Code, the child is
The action to regain custody will not
entitled to support and inheritance from
prosper. In the first place Majorette
Gigolo.
cannot regain custody of the baby. As
surrogate mother she merely carries the
ALTERNATIVE ANSWER:
child in her womb for its development.
Yes, because Gigolo is the natural and
The child is the child of the natural
biological parent of the baby.
parents- Gigolo and his partner. The
agreement between Gigolo and Majorette
is a valid agreement.
Paternity
&
Filiation;
Legitimacy;
(B) Can Gigolo demand from Majorette the
Presumption (2008)
return of the P2 million if he returns the
No. III. Roderick and Faye were high school
baby? Explain. (2.5%)

sweethearts. When Roderick was 18 and


Faye, 16 years old, they started to live
SUGGESTED ANSWER:
together as husband and wife without the
Never Let The Odds Keep You From Pursuing What You Know In Your Heart You Were Me
ant To Do.-Leroy Satchel Paige
Page 35 of 180

Civil Law Q&As (2007-2013)


hectorchristopher@yahoo.com
dbaratbateladot@gmail.com
benefit of marriage. When Faye reached 18
March 2002, the Supreme Court ruled
years of age, her parents forcibly took her
that impugning the legitimacy of the
back and arranged for her marriage to
child is a strictly personal right of
Brad. Although Faye lived with Brad after
husband, except: (a) when the husband
the
to
died before the expiration of the period
regularly visit Faye while Brad was away at
fixed for bringing the action; (b) if he
work. During their marriage, Faye gave
should
birth to a baby girl, Laica. When Faye was
complaint,
25 years old, Brad discovered her continued
therefrom, or (c) if the child was born
liason with Roderick and in one of their
after the death of the husband. Laica s
heated arguments, Faye shot Brad to death.
case does not fall under any of the
She lost no time in marrying her true love
exceptions.
marriage,
Roderick,

Roderick
without
a
continued
marriage
die
after
the
without
filing
having
of
the
desisted
license,
claiming that they have been continuosly
(D).
cohabiting for more than 5 years.
marriage of her biological parents? (1%)
(B). What is the filiation status of Laica?
SUGGESTED ANSWER:
(2%)
Can
Laica
be
legitimated
by
the
No. Laica cannot be legitimated by the
marriage
SUGGESTED ANSWER:

of
her
biological
parents
because only children conceived and
Laica
is
legitimate
children
born outside of wedlock of parents who
conceived or born during the marriage of
at the time of the conception of the
the
former were not disqualified by any
parents
are
because
presumed
to
be
legitimate (Art. 164, FC).
impediment to marry each other may be
legitimated (Art. 177, FC).
(C).Can Laica bring an action to impugn her
own status on the ground that based on
DNA results, Roderick is her biological
father? (2%)
Paternity & Filiation; Legitimation of a
Child from a Previous Valid Marriage
SUGGESTED ANSWER:
(2008)
No. Laica cannot bring an action to
impugn her own status. In Liyao Jr. v.
Tanhoti-Liyao,

G.R.
No.
138961,
07
No. IV. Gianna was born to Andy and
Aimee, who at the time Gianna s birth were
not married to each other. While Andy was
single at the time, Aimee was still in the
Never Let The Odds Keep You From Pursuing What You Know In Your Heart You Were Me
ant To Do.-Leroy Satchel Paige
Page 36 of 180

Civil Law Q&As (2007-2013)


hectorchristopher@yahoo.com
dbaratbateladot@gmail.com
process of securing a judicial declaration of
statement is false. Explain your answer in
nullity on her marriage to her ex-husband.
not more than two (2) sentences.
Gianna s birth certificate, which was signed
by both Andy and Aimee, registered the
status
of
Gianna
as
"legitimate",
her
surname carrying that of Andy s and that
her parents were married to each other.
(E). A dead child can be legitimated. (1%)
SUGGESTED ANSWER:
TRUE To be legitimated, the law does
(C). Assuming that Aimee is successful in
declaring her former marriage void, and
Andy and Aimee subsequently married each
other, would Gianna be legitimated? (1%)
not require a child to be alive at the
same time of the marriage of his / her
parents ( Article 177, FC ). Furthermore,
Art. 181 of the Family Code which states
that [Th]e llegitimation of children who
died before the celebration of marriage
SUGGESTED ANSWER:
will benefit their descendants, does not
Gianna cannot be legitimated by the
preclude
instances
where
such

subsequent marriage of Andy and Aimee.


legitimation will benefit no one but the
Art. 177 of the FC provides that "only
child s ascendants ,or other relatives .
children conceived and born outside of
wedlock of parents who, at the time of
the conception of the former, were not
Paternity
disqualified by any impediment to marry
Ascendants & Descendants; Collateral
each other may be legitimated." In the
Blood Relatives (2008)
present case, a legal impediment was
existing at the time of the conception of
Gianna. Her mother, Aimee, was still
alive in the process of securing judicial
declaration of nullity on her marriage to
her ex-husband.
&
Filiation;
Support:
No.V. Despite several relationships with
different
unmarried.
women,
His
Andrew
first
remained
relationship
with
Brenda produced a daughter,
years old. His second, with
two sons: Jon and Ryan. His
Donna, bore him no children

Amy, now 30
Carla, produced
third, with
although

Paternity & Filiation; Legitimation of a


Elena has a daughter Jane, from a previous

Dead Child (2009)


relationship. His last, with Fe, produced no
biological
children
but
they
informally
No. I. TRUE or FALSE. Answer TRUE if the
adopted without court proceedings, Sandy s
statement
now 13 years old, whom they consider as
is
true,
or
FALSE
if
the
Never Let The Odds Keep You From Pursuing What You Know In Your Heart You Were Me
ant To Do.-Leroy Satchel Paige
Page 37 of 180

Civil Law Q&As (2007-2013)


hectorchristopher@yahoo.com
dbaratbateladot@gmail.com
their own. Sandy was orphaned as a baby
Paternity & Filiation; Use of Surname;
and was entrusted to them by the midwife
Illegitimate Child (2009)
who attended to Sandy s birth. All the
children, including Amy, now live with
No.XIV. Rodolfo, married to Sharon, had an
andrew in his house.
illicit affair with his secretary, Nanette, a
19-year old girl, and begot a baby girl,
(B). In his old age, can Andrew be legally
Rona. Nanette sued Rodolfo for damages:
entitled to claim support from Amy, Jon,
actual, for hospital and other medical
Ryan, Vina, Wilma, and Sandy assuming
expenses
that all of them have the means to support
caesarean section; moral, claiming that
him? (1%)
Rodolfo
in
delivering
promised
to
the
child
marry
by
her,

representing that he was single when, in


SUGGESTED ANSWER:
Andrew, in his old age, cannot be legally
fact, he was not; and exemplary, to teach a
lesson to like-minded Lotharios.
entitled to claim support because Art.
(B). Suppose Rodolfo later on acknowledges
195, par 2 of the FC limits the giving of
Rona and gives her regular support, can he
support to "legitimate ascendants and
compel her to use his surname? Why or
descendants."
why not? (2%)
(C). Can Amy, Jon, Ryan, Vina, Wilma, and
SUGGESTED ANSWER:
Sandy legally claim support from each
No. he has no right to compel Rona to
other? (2%)
use his surname. The law does not give
SUGGESTED ANSWER:
Amy, Jon, Ryan, Vina, Wilma and Sandy
cannot legally claim support from each
other because Art. 195, par 5 limits the
giving of support to "legitimate brothers
and sisters, whether full or half blood."
him the right simply because he gave her
support (RA 9255).
Under the Family Code, an illegitimate
child was required to use only the
surname of the mother. Under RA 9255,
otherwise known as the Revilla law,
however, the illegitimate child is given
the option to use the surname of the
illegitimate father when the latter has
recognized the former in accordance
with law. Since the choice belongs to the
illegitimate
child,
Rodolfo

cannot
Never Let The Odds Keep You From Pursuing What You Know In Your Heart You Were Me
ant To Do.-Leroy Satchel Paige
Page 38 of 180

Civil Law Q&As (2007-2013)


hectorchristopher@yahoo.com
dbaratbateladot@gmail.com
compel Rona, if already of age, to use
They are not related at all to Edilberto.
the surname against her will. If Rona is
They were born during the marriage of
still a minor, to use the surname of
Conrado
Rodolfo
of
considered legitimate children of the
Rona s mother who has sole parental
said spouses. This status is conferred on
authority over her.
them at birth by law.
will
require
the
consent
and
Clarita,
hence,
are
Under Philippine law, a person cannot
Paternity & Filiation; Who May Impugn
have more than one natural filiation.
Legitimacy (2009)
The legitimate filiation of a person can
be changed only if the legitimate father
No.V.

Four
children,
namely:
Alberto,
will successfully impugn such status.
Baldomero, Caridad, and Dioscoro, were
born to the spouses Conrado and Clarita de
In the problem, therefore, the filiation of
la Costa. The childrens birth certificates
Alberto and Baldomero as legitimate
were duly signed by Conrado, showing
children of Condrado cannot be changed
them to be the couples legitimate children.
by their recognition by Edilberto as his
Later, one Edilberto de la Cruz executed a
notarial document acknowledging Alberto
and Baldomero as his illegitimate children
>with
Clarita.
Edilberto
died
leaving
substantial properties. In the settlement of
his
estate,
Alberto
intervened
claiming
deceaseds
illegitimate
and
shares
Baldomero
as

the
children.
The
legitimate family of Edilberto opposed the
claim.
illegitimate children. Before they can be
conferred
illegitimate
first
the
status
children,
impugn
their
of
Edilbertos
Condrado
must
legitimacy.
Since
Condrado has not initiated any action to
impugn their legitimacy, they continue
to be the legitimate of Condrado. They
cannot be the illegitimate children of
Edilberto at the same time. Not being
the illegitimate children of Edilberto,
they have no right to inherit from him.
Are Alberto and Baldomero entitled to share
in the estate of Edilberto? Explain. (4%)
Property
Alberto
and
Baldomero
Adulterous
Relationship (2009)

SUGGESTED ANSWER:
No,
Relations;
are
not
entitled to share in Edilbertos estate.
No. XI. TRUE or FALSE. Answer TRUE if
the statement is true, or FALSE if the
Never Let The Odds Keep You From Pursuing What You Know In Your Heart You Were Me
ant To Do.-Leroy Satchel Paige
Page 39 of 180

Civil Law Q&As (2007-2013)


hectorchristopher@yahoo.com
statement is false. Explain your answer in
not more than two (2) sentences.
dbaratbateladot@gmail.com
SUGGESTED ANSWER:
It depends. If the value of the building is
(B). If there is no marriage settlement, the
more than the value of the land, the
salary of a "spouse" in an adulterous
building
marriage
becomes conjugal property under Art.
belongs
to
the
conjugal
is
conjugal
and
the
land
120 of the Family Code. This is a case of
partnership of gains. (1%)
reverse accession, where the building is
SUGGESTED ANSWER:
considered as the principal and the land,
False. In adulterous relationship, the
the accessory. If, on the other hand, the
salary of a married partner belongs to
value of the land is more than the value
the absolute community, or conjugal

of the building, then the ordinary rule of


partnership, of such married partner
accession applies where the land is the
with his or her lawful spouse. Under
principal
and
Articles 148 of the Family Code, the
accessory.
In
property
remains paraphernal property and the
partner
relations
and
governed
between
his/her
by
married
paramour
ordinary
is
co-ownership
where the partners become co-owners
only
when
they
acquisition
paramour
of
is
contributed

the
deemed
to
property.
to
have
the
The
not
contributed in the earning of the salary
of the married partner.
the
such
building,
case,
the
the
land
building becomes paraphernal propery.
Note: The rule on reverse accession is
applicable only to the regime of conjugal
partnership of gains in both the Family
Code and the New Civil Code. The foregoing
answer assumes that CPG is the regime of
the property relations of the spouses.
Property Relations; Ante-Nuptial Debt
Property Relations; Accession (2012)
No.III.(a) Maria, wife of Pedro, withdrew P 5
Million from their conjugal funds. With this
money, she constructed a building on a lot
which she inherited from her father. Is the
(2007)
No. VII. Write "TRUE" if the statement is
true or "FALSE" if the statement is false. If
the statement is FALSE, state the reason.
(2% each).
building conjugal or paraphernal? Reasons.
(5%)
(3). An individual, While single, purchases a
house and lot in 1990 and borrows money
Never Let The Odds Keep You From Pursuing What You Know In Your Heart You Were Me
ant To Do.-Leroy Satchel Paige
Page 40 of 180

Civil Law Q&As (2007-2013)


in
1992
to
repair
hectorchristopher@yahoo.com
it.
In
1995,
such
individual gets married while the debt is
Property
dbaratbateladot@gmail.com
Relations;
Unions
Without
Marriage (2012)
still being paid. After the marriage, the debt
is still the responsibility of such individual.
No.V. a) Spouses Primo and Monina Lim,
childless, were entrusted with the custody
SUGGESTED ANSWER:
of two (2) minor children, the parents of
whom were unknown. Eager of having
FALSE.
The
absolute
Community
of
children of their own, the spouses made it
property is liable for the ante-nuptial
appear

debts of either spouse in so far as the


parents by naming them Michelle P. Lim
same redounded to the benefit of the
and
family (Art. 94 par.7, FC).
Monina married Angel Olario after Primos
The
responsibility
debt
is
of
the
already
the
community
property, because the property already
constitutes
absolute
Michael
they
were
Jude
the
Lim.
childrens
Subsequently,
death.
ALTERNATIVE ANSWER:
FALSE.
that
community

property under Art. 91 of FC which took


effect in 1988 while the house and lot
here involved was purchased in 1990.
She decided to adopt the children by
availing the amnesty given under R.A. 8552
to those individuals who simulated the
birth of a child. She filed separate petitions
for the adoption of Michelle, then 25 years
old and Michael, 18. Both Michelle and
Michael gave consent to the adoption.
There is no indication that the spouse
The trial court dismissed the petition and
who bought the property had legitimate
ruled that Monina should have filed the
descendants by a former marriage, which
petition jointly with her new husband.
would exclude the house and lot from
Monina, in a Motion for Reconsideration
the community property, Art. 92 par 3,
argues that mere consent of her husband
FC). If the spouses established a conjugal
would suffice and that joint adoption is not
partnership, the property belongs to the
needed,
individual spouse if full ownership was
emancipated.
for
the
adoptees
are
already
vested before marriage (Art. 118, FC).
Is the trial court correct in dismissing the
petitions for adoption? Explain. (5%)
SUGGESTED ANSWER:

Never Let The Odds Keep You From Pursuing What You Know In Your Heart You Were Me
ant To Do.-Leroy Satchel Paige
Page 41 of 180

Civil Law Q&As (2007-2013)


hectorchristopher@yahoo.com
dbaratbateladot@gmail.com
Yes, the trial court was correct. At the
Borromeo discovered that titles to the three
time the petitions for adoptions were
(3) lots have been transfereed in the name
filed, petitioner had already remarried.
of Descallar. Who is the rightful owner of
Under the law, husband and wife shall
the properties? Explain. (5%)
adopt
jointly,
except
in
the
cases
enumerated in the law. The adoption
cases of Michelle and James do not fall
in any of the exceptions provided in the
law where a spouse is permitted to adopt
alone.
Hence,
Monina
should
adopt
jointly with her husband Angel (Adoption
of Michelle P. Lim, G.R. Nos. 168992-93,
May 21, 2009).
SUGGESTED ANSWER:
It depends. On the assumption that the
Family Code is the applicable law, the
ownership of the properties depends on
whether or not, Jambrich and Descallar
are capacitated to marry each other
during their cohabitation, and whether
or not both have contributed funds for

the acquisition of the properties.


Property
Relations;
Unions
Without
Marriage (2012)
If both of them are capacitated to marry
each other, Art 147- co-ownership will
apply to their property relations and the
No.V. b) Jambrich, an Austrian, fell in-love
properties in question are owned by
and lived together with Descallar
and
them in equal shares even though all the
bought their houses and lots at Agro-Macro
funds used in acquiring the properties
Subdivision.
came only from the salaries or wages, or
In
the
Contracts
to
Sell,
Jambrich and Descallar were referred to as
the
the buyers. When the Deed of Absolute Sale
business or profession. In such case,
was presented for registration before the
while Jambrich is disqualified to own
Register of Deeds, it was refused because
any

Jambrich was an alien and could not


subsequent transfer of all his interest
acquire
public
therein to Borromeo, a Filipino, was
Descallar
valid as it removed the disqualification.
separated, Jambrich purchased an engine
In such case, the properties are owned
and some accessories for his boat from
by Borromeo and Descallar in equal
Borromeo. To pay for his debt, he sold his
shares.
domain.
alienable
After
lands
Jambrich
of
the
and
rights and interests in the Agro-Macro
properties to Borromeo.
income
part
of
of
Jambrich
the
from
properties,

his
his
If, on the other hand, Jambrich and
Descallar were not capacitated to marry
Never Let The Odds Keep You From Pursuing What You Know In Your Heart You Were Me
ant To Do.-Leroy Satchel Paige
Page 42 of 180

Civil Law Q&As (2007-2013)


each
other
Art.
hectorchristopher@yahoo.com
148-co-ownership
governs their property relations. Under
marriage,
dbaratbateladot@gmail.com
the
couple
possessed
the
following properties:
this regime, Jambrich and Descallar are
co-owners of the properties but only if
both
of
them
contributed
in
a house and lot acquired by B on
August 3, 1988, one third (1/3) of
their
acquisition. If all the funds used in
the
acquiring
downpayment) of which he paid; one
the
properties
in

question
purchase
price
came from Jambrich, the entire property
third (1/3)
is his even though he is disqualified from
February 14, 1990 out of a cash gift
owning it. His subsequent transfer to
given to her by her parents on her
Borromeo,
it
graduation on April 6, 1989; and the
removed the disqualification. In such
balance was paid out of the spouses
case, all the properties are owned by
joint income; and
Borromeo.
Descallar
however,
If,
on
is
the
valid
as
other
hand
to
their
contributed

was
(representing
paid
by
G
on
an apartment unit donated to B by
an uncle on June 19, 1987.
acquisition, the properties are co-owned
by Descallar and Borromeo in proportion
to the respective contributions of the
Descallar and Jambrich.
(A) Who owns the foregoing properties?
Explain. (5%)
SUGGESTED ANSWER:
Note: The facts of the problem are not
exactly the same as in the case of
Borromeo v. Descallar, G.R. NO. 159310,
Feb 24, 2009, hence, the difference in
the resulting answer.
Since the marriage was declared void ab
initio in 2001, no Absolute Community
or
Conjugal
Partnership
established between
property
relation
is
was
ever
B and G. Their
governed
by
a
special co-ownership under Article 147
Property

Relations;
Void
Marriages
(2010)
No.VII. G and B were married on July 3,
1989. On March 4, 2001, the marriage,
which bore no offspring, was declared
void ab initio under Article 36 of the Family
Code. At the time of the dissolution of the
of the Family Code because they were
capacitated to marry each other.
Under
that
Article
147,
wages
and
salaries of the former spouses earned
during their cohabitation shall be owned
by them in equal shares while properties
acquired thru their work for industry
shall be owned by them in proportion to
Never Let The Odds Keep You From Pursuing What You Know In Your Heart You Were Me
ant To Do.-Leroy Satchel Paige
Page 43 of 180

Civil Law Q&As (2007-2013)


hectorchristopher@yahoo.com
dbaratbateladot@gmail.com
their respective contributions. Care and
G. She is an undivided co-owner to the
maintenance of the family is recognized
extent
as
the
acquisition when she paid 1/3 of the
absence of proof as to the value of their
purchase price using the gift from her
respective
parents. Although the gift was acquired
a
valuable
contribution.
contributions,
In
they
shall
for
her
contribution
in
its
by G during her cohabitation with B, it is
share equally.
If ownership of the house and lot was
acquired by B on August 3, 1988 at the
time he bought it on installment before
he got married, he shall remain owner of
the house and lot but he must reimburse

G for all the amounts she advanced to


pay the purchase price and for one-half
share in the last payment from their
joint income. In such case, the house
and lot were not acquired during their
cohabitation, hence, are not co-owned by
B and G.
her exclusive property. It did not consist
of wage or salary or fruit of her work or
industry.
(3) 1/3 of the house is co-owned by B
and G because the payment came from
their co-owned funds, i.e., their joint
income during their cohabitation which
is shared by them equally in the absence
of any proof to the contrary.
After
summing
up
their
prospective
shares, B and G are undivided co-owners
But if the ownership of the house and lot
was acquired during the cohabitation,
the house and lot will be owned as
follows:
of the house and lot in equal shares.
As
to
the
apartment,
it
is
owned
exclusive by B because he acquired it
before their cohabitation. Even if he
(1) 1/3 of the house and lot is owned by
acquired it during their cohabitation, it
B. He is an undivided co-owner to that
will

extent
its
because it did not come from his wage or
acquisition in the form of the down
salary, or from his work or industry. It
payment he made before the celebration
was
of the marriage. The money he used to
uncle.
for
his
contributions
in
still
be
acquired
his
exclusive
gratuitously
property
from
his
pay the down payment was not earned
during the cohabitation, hence, it is his
exclusive property.
(B) If G and B had married on July 3, 1987
and their marriage was dissolved in 2007,
who owns the properties? Explain. (5%)
(2) 1/3 of the house and lot is owned by
SUGGESTED ANSWER:
Never Let The Odds Keep You From Pursuing What You Know In Your Heart You Were Me
ant To Do.-Leroy Satchel Paige
Page 44 of 180

Civil Law Q&As (2007-2013)


hectorchristopher@yahoo.com
dbaratbateladot@gmail.com
The answer is the same as in letter A.
in Paraaque. After four (4) years or in
Since the parties to the marriage which
2001,
was later declared void ab initio were
college degree as a fulltime student, she
capacitated to marry each other, the
and
applicable law under the New Civil Code
license.
was
Article
144.This
Article
G
B
having
contracted
completed
her
marriage
4-year
without
a
is
substantially the same as Article 147 of
The marriage of B and G was, two years
the Family Code.

later, declared null and void due to the


absence of a marriage license.
Hence, the determination of ownership
will remain the same as in question A.
(A). If you were the judge who declared the
And
nullity of the marriage, to whom would you
even
assuming
that
the
two
provisions are not the same, Article 147
award the lot? Explain briefly. (3%)
of the Family Code is still the law that
will govern the property relations of B
and G because under Article 256, the
Family
Code
has
retroactive
effect
insofar as it does not prejudice or impair
vested or acquired rights under the new
Civil Code or other laws. Applying Article
147 retroactively to the case of G and B
will not impair any vested right. Until
the declaration of nullity of the marriage
under the Family Code, B and G have not
as yet acquired any vested right over the
properties
acquired
during
their
cohabitation.
SUGGESTED ANSWER:
Since the marriage was null and void, no
Absolute

Community
or
Conjugal
Partnership was established between B
and G. Their properties are governed by
the special co-ownership provision of
Article 147 of the Family Code because
both B and G were capacitated to marry
each other. The said Article provides
that when a man and a woman who are
capacitated to marry each other,
live
exclusively with each other as husband
and wife without the benefit of marriage,
or under a void marriage: (1) their wages
Property
Relations;
Void
Marriages
(2010)
No.X. In 1997, B and G started living
together without the benefit of marriage.
The relationship produced one offspring,
Venus. The couple acquired a residential lot
and salaries shall be owned by them in
equal shares; and (2) property acquired
by both of them through their work or
industry shall be governed by the rules
on co-ownership. In co-ownership, the
parties are co-owners if they contributed
something of value in the acquisition of
Never Let The Odds Keep You From Pursuing What You Know In Your Heart You Were Me
ant To Do.-Leroy Satchel Paige
Page 45 of 180

Civil Law Q&As (2007-2013)


the
property.
proportion
hectorchristopher@yahoo.com
Their
to
contributions.
share
their
In
an
is
family
is
not
Succession
in
respective
ordinary
coownership the care and maintenance of
the
dbaratbateladot@gmail.com
recognized
as
Disposition; Mortis Causa vs. Intervivos;
Corpse (2009)
a
valuable contribution for the acquisition
No. XI. TRUE or FALSE. Answer TRUE if

of a property. In the Article 147 special


the statement is true, or FALSE if the
co-ownership
statement is false. Explain your answer in
however,
care
and
maintenance is recognized as a valuable
contribution
contributor
which
to
half
will
of
entitle
the
the
property
(E). A person can dispose of his corpse
through an act intervivos. (1%)
acquired.
Having
not more than two (2) sentences.
been
cohabitation,
acquired
the
during
residential
their
lot
is
SUGGESTED ANSWER:
False.

A persons cannot dispose of his


presumed acquired through their joint
corpse through an act inter vivos, i.e.,
work and industr
under Article 147,
an act to take effect during his lifetime.
hence, B and G are co-owners of the said
Before his death there is no corpse to
property in equal shares.
dispose. But he is allowed to do so
Article 147 also provides that when a
party to the void marriage was in bad
through an act mortis causa, i.e., an act
to take effect upon his death.
faith, he forfeits his share in the coownership
in
favor
of
the
common
children or descendants, the default of
Heirs;
children or descendants, the forfeited
(2008)
Fideicommissary
Substitution
share shall belong to the innocent party.
In the foregoing problem, there is no
No. XIII. Raymond, single, named his sister
showing that one party was in bad faith.
Ruffa in his will as a devisee of a parcel of
Hence, both shall be presumed in good

land which he owned. The will imposed


faith and no forfeiture shall take place.
upon Ruffa the obligation of preseving the
land and transferring it, upon her death, to
her illegitimate daughter Scarlet who was
then only one year old. Raymond later died,
leaving behind his widowed mother, Ruffa
and Scarlet.
Never Let The Odds Keep You From Pursuing What You Know In Your Heart You Were Me
ant To Do.-Leroy Satchel Paige
Page 46 of 180

Civil Law Q&As (2007-2013)


hectorchristopher@yahoo.com
dbaratbateladot@gmail.com
(A). Is the condition imposed upon Ruffa, to
Ruffa (Art. 992, Civil Code). Moreover,
preserve the property and to transmit it
Scarlet is not a compulsory heir of
upon her death to Scarlet, valid? (1%)
Raymond, hence she can inherit only by
testamentary
SUGGESTED ANSWER:
succession.
Since
Raymond executed a will in the case at
Yes, the condition imposed upon Ruffa
bar, Scarlet may inherit from Raymond.
to preserve the property and to transmit
it upon her death to Scarlet is valid
because
it
is
tantamount
to
fideicommissary substitution under Art.
Heirs; Intestate Succession; Legitime;
Computation (2010)
863 of the Civil Code.
No.XI. The spouses Peter and Paula had
(B).
If
Scarlet
predeceases
Ruffa,

who
inherits the property? (2%)
three (3) children. Paula later obtained a
judgment of nullity of marriage. Their
absolute community of property having
SUGGESTED ANSWER:
Ruffa
will
inherit
been dissolved, they delivered P1 million to
the
property
as
Scarlet s heir. Scarlet acquires a right to
the
succession
3
children
as
their
presumptive legitimes.
she
children by his second wife Marie. Peter
should predecease Ruffa (Art. 866, Civil
and Marie, having successfully engaged in
Code).
business, acquired real properties. Peter
even
time
their
Peter later re-married and had two (2)
death,
the

of
of
Raymond s
from
each
though
(C). If Ruffa predeceases Raymond, can
later died intestate.
Scarlet inherit the property directly from
(A). Who are Peters legal heirs and how will
Raymond? (2%)
his estate be divided among them? (5%)
SUGGESTED ANSWER:
SUGGESTED ANSWER:
If
Ruffa
Raymond s
predeceases
widowed
mother
Raymond,
will
be
entitled to the inheritance. Scarlet, an
illegitimate child, cannot inherit the
property by intestate succession from
Raymond who is a legitimate relative of
The legal heirs of Peter are his children
by the first and second marriages and
his surviving second wife.
Their shares in the estate of Peter will
depend, however, on the cause of the
Never Let The Odds Keep You From Pursuing What You Know In Your Heart You Were Me
ant To Do.-Leroy Satchel Paige
Page 47 of 180

Civil Law Q&As (2007-2013)


hectorchristopher@yahoo.com
nullity of the first marriage. If the
(B)
nullity
psychological capacity:
of
the
first
marriage
was
If
the
dbaratbateladot@gmail.com
psychological incapacity of one or both
2
spouses, the three children of that void
children
ground
in equal shares. If the judgment of
nullity was for other causes, the three
each
Surviving
shall
children
such
that
an
not
of
second

 of the estate
second spouse
3
distributed
is
marriage
children are illegitimate and the estate
be
nullity
legitimate  of the estate for
marriage are legitimate and all of the
legal heirs shall share the estate of Peter
of
illegitimate 1/12
of
estate
for
each of first marriage
illegitimate child of the first marriage
shall receive half of the share of a
legitimate child of the second marriage,
and the second wife will inherit a share
equal to that of a legitimate child. In no
case may the two legitimate children of
the second marriage receive a share less
than one-half of the estate which is their
legitime.
When
the
estate
is
not
sufficient to pay all the legitimes of the
compulsory heirs, the legitime of the
Note: The legitime of an illegitimate
child is supposed to be the legitime of
a legitimate child or 1/8 of the estate.

But the estate will not be sufficient to


pay
the
said
legitime
of
the
3
illegitimate children, because only  of
the
estate
is
left
after
paying
the
legitime of the surviving spouse which is
preferred.
spouse is preferred and the illegitimate
Hence, the remaining  of the estate
children suffer the reduction.
shall be divided among the 3 illegitimate
children.
Computation:
(A)
If
the
ground
of
nullity
is
psychological incapacity:
3

children
marriage
by
first 1/6 of the estate
for each
2 children by second 1/6 of the estate
marriage
for each
Surviving
second 1/6 of the estate
spouse
(B). What is the effect of the receipt by
Peters 3 children by his first marriage of
their presumptive legitimes on their right to
inherit following Peters death? (5%)
SUGGESTED ANSWER:
In the distribution of Peters estate, of
the
presumptive
received
by
the
3
children of the first marriage shall be
collated to Peters estate and shall be
Never Let The Odds Keep You From Pursuing What You Know In Your Heart You Were Me
ant To Do.-Leroy Satchel Paige
Page 48 of 180

Civil Law Q&As (2007-2013)


imputed
Art 992 of the NCC, an illegitimate child
respective inheritance from Peter. Only
has no right to inherit ab intestato from
half
the legitimate children and relatives of
the
an
advance
presumptive
of
dbaratbateladot@gmail.com
their
of
as
hectorchristopher@yahoo.com
legitime
is
collated to the estate of Peter because
his
father
or
the other half shall be collated to the
disqualified
estate of his first wife.
because Arnel is an illegitimate child of
to
mother.
inherit
Arnel

from
is
Ricky
Franco and Ricky is a legitimate relative
of Franco.
Heirs; Representation; Iron-Curtain Rule
(2012)
Heirs; Reserva Troncal (2009)
No.VIII.a) Ricky and Arlene are married.
They begot Franco during their marriage.
No. I. TRUE or FALSE. Answer TRUE if the
Franco had an illicit relationship with
statement
Audrey and out of which, they begot Arnel.
statement is false. Explain your answer in
Frnaco
not more than two (2) sentences.
predeceased
Ricky,
Arlene
and
is
true,
or
FALSE
if
the
Arnel. Before Ricky died, he executed a will
which when submitted to probate was
(B).In reservatroncal, all reservatarios (reser
opposed by Arnel on the ground that he
vees) inherit as a class and in equal shares
should be given the share of his father,
regardless of their proximity in degree to

Franco. Is the opposition of Arnel correct?


the prepositus. (1%)
Why? (5%)
SUGGESTED ANSWER:
FALSE. Not all the relatives within the
SUGGESTED ANSWER:
third degree will inherit as reservatario ,
No, his opposition is not correct. Arnel
and not all those who are entitled to
cannot
inherit will inherit in the equal shares .
inherit
from
Ricky
in
the
representation of his father Franco. In
The
representation, the representative must
succession will determine who among
not only be a legal heir of the person he
the relatives will inherit as reservatarios
is representing, he must also be a legal
and what shares they will tak, i.e., the
heir of the decedent he seeks to inherit
direct line excludes the collateral, the
from.
descending
While Arnel is a legal heir of Franco, he
is not a legal heir of Ricky because under
applicable

direct
laws
line
of
intestate
excludes
the
ascending ,the nearer excludes the more
remote, the nephews and nieces exclude
Never Let The Odds Keep You From Pursuing What You Know In Your Heart You Were Me
ant To Do.-Leroy Satchel Paige
Page 49 of 180

Civil Law Q&As (2007-2013)


hectorchristopher@yahoo.com
dbaratbateladot@gmail.com
the uncles and the aunts, and half blood
(1). The wife of Ramon will, therefore,
relatives
receive one half () of the estate or the
inherit
half
the
share
of
full-blooded relatives.
amount of P5,000,000.00.
(2). The three (3) full-blood brothers, will,
therefore, receive P1,000,000.00 each.
(3).
Intestate Succession (2008)
leaving a net estate of P10,000,000.00.
Determine how much each heir will receive
from the estate:
full-blood brothers, two half-brothers, and
one nephew (the son of a deceased fullblood brother)? Explain. (3%)
by
receive
right
of
representation.
(4). The two (2) half-brothers will receive
P500,000.00 each.
Ramon shall be inherited by his wife and
his full and half blood siblings or their
respective representatives. In intestacy,
if the wife concurs with no one but the
siblings of the husband, all of them are
of
the

deceased
husband. The wife will receive half of the
intestate estate, while the siblings or
respective
of a
deceased full-blood brother)? Explain. (3%)
SUGGESTED ANSWER:
estate or P5,000,000.00. The other half
Having died intestate, the estate of
heirs
sister, and three nephews (sons
The wife will receive one half (1/2) of the
SUGGESTED ANSWER:
their
will
(B). If Ramon is survived by his wife, a half(A). If Ramon is survived by his wife, three
intestate
nephew
P1,000,000.00
No. VII. Ramon Mayaman died intestate,
the
The
representatives,
will
inherit the other half to be divided
among them equally. If some siblings are
of the full-blood and the other of the half
blood, a half blood sibling will receive
half the share of a full-blood sibling.
shall be inherited by (1) the full-blood
brother,
represented
by

his
three
children, and (2) the half-sister. They
will divide the other half between them
such that the share of the half-sister is
just half the share of the full-blood
brother.
The share
of
the full-blood
brother shall in turn be inherited by the
three nephews in equal shares by right of
presentation.
Therefore, the three (3) nephews will
receive P1,111,111.10 each the halfsister
will
receive
the
sum
of
P1,666,666.60.
Never Let The Odds Keep You From Pursuing What You Know In Your Heart You Were Me
ant To Do.-Leroy Satchel Paige
Page 50 of 180

Civil Law Q&As (2007-2013)


hectorchristopher@yahoo.com
Intestate Succession (2008)
dbaratbateladot@gmail.com
(D). How should the house and lot, and the
cash be distributed? (1%)
No.X.
Arthur
executed
a
will
which
contained only: (i) a provision disinheriting
SUGGESTED ANSWER:
his daughter Bernica for running off with a
married man, and (ii) a provision disposing
Since the probate of the will cannot be
of his share in the family house and lot in
allowed,
favor of his other children Connie and Dora.
succession apply. Under Art. 996 of the
He did not make any provisions in favor of
Civil Code, if a widow or widower and
his wife Erica, because as the will stated,
legitimate children or descendants are
she would anyway get of the house and
left, the surviving spouse has the same
lot as her conjugal share. The will was very
share as of the children. Thus, ownership
brief and straightforward and both the
over the house and lot will be created

above provisions were contained in page 1,


among
which Arthur and his instrumental witness,
Bernice, Connie and Dora. Similarly, the
signed at the bottom. Page 2 contained the
amount of P 1 million will be equally
attestation clause and the signatures, at
divided among them.
the
wife
rules
Erica
and
on
her
intestate
children
the bottom thereof, of the 3 instrumental
witnesses which included Lambert, the
driver of Arthur; Yoly, the family cook, and
Attorney Zorba, the lawyer who prepared
the will. There was a 3rd page, but this only
contained the notarial acknowledgement.
Intestate
Succession;
Representation:
Rights
Illegitimate,
of
Adopted
Child; Iron Curtain Rule (2007)
The attestation clause stated the will was
No. X. For purpose of this question, assume

signed on the same occasion by Arthur and


all formalities and procedural requirements
his instrumental witnesses who all signed
have been complied with.
in the presence of each other, and the
notary public who notarized the will. There
In 1970, Ramon and Dessa got married.
are no marginal signatures or pagination
Prior to their marriage, Ramon had a child,
appearing on any of the 3 pages. Upon his
Anna. In 1971 and 1972, Ramon and Dessa
death, it was discovered that apart from the
legally
house and lot, he had a P 1 million account
respectively. In 1973, Dessa died while
deposited with ABC bank.
giving birth to Larry Anna had a child, Lia.
adopted
Cherry
and
Michelle
Anna never married. Cherry, on the other
hand, legally adopted Shelly. Larry had
Never Let The Odds Keep You From Pursuing What You Know In Your Heart You Were Me
ant To Do.-Leroy Satchel Paige
Page 51 of 180

Civil Law Q&As (2007-2013)


hectorchristopher@yahoo.com
dbaratbateladot@gmail.com
twins, Hans and Gretel, with his girlfriend,
also of the person from whom the person
Fiona. In 2005, Anna, Larry and Cherry
being
died in a car accident. In 2007, Ramon
inherit. While Shelly is a legal heir of
died. Who may inherit from Ramon and
Cherry, Shelly is not a legal heir of
who may not? Give your reason briefly.
Ramon.
Adoption
(10%)
personal
legal
represented
was
supposed
created
relation
a
only
to
purely
between
Cherry and Shelly.
SUGGESTED ANSWER:
(2). Hans and Gretel are barred from
The following may inherit from Ramon:
(1). Michelle, as an adopted child of
Ramon, will inherit as a legitimate child
of Ramon. As an adopted child, Michelle

inheriting from Ramon under Art. 992,


NCC. Being illegitimate children, they
cannot inherit ab intestao from Ramon.
ALTERNATIVE ANSWER:
has all the rights of a legitimate child
(Sec 18, Domestic Adoption Law).
The problem expressly mentioned the
dates of the adoption of Cherry and
(2). Lia will inherit in representation of
Michelle as 1971 and 1972. During that
Anna. Although Lia is an illegitimate
time, adoption was governed by the New
child, she is not barred by Articles 992,
Civil Code. Under the New Civil Code,
because
an
husband and wife were allowed to adopt
illegitimate herself. She will represent
separately or not jointly with the other
Anna as regards Anna s legitime under
spouse. And since the problem does not
Art. 902, NCC and as regards Anna s
specifically and categorically state, it is
intestate share under Art. 990, NCC.
possible to construe the use of the word
her
mother
Anna
is
The following may not inherit from
Ramon:
cannot represent Cherry. This is because
adoption creates a personal legal relation
between

adopted.
The
the
adopter
law
on
and
the
representation
requires the representative to be a legal
heir of the person he is representing and
in
the
problem
as
indicative of the situation that Cherry
was
(1). Shelly, being an adopted child, she
only
"respectively"
adopted
by
Ramon
alone
and
Michelle was adopted by Dessa alone. In
such
case
of
separate
adoption
the

alternative answer to the problem will be


as follows: Only Lia will inherit from
Ramon in representation of Ramon s
illegitimate daughter Anna. Although Lia
is an illegitimate child, she is not barred
from inheriting from Ramon because her
Never Let The Odds Keep You From Pursuing What You Know In Your Heart You Were Me
ant To Do.-Leroy Satchel Paige
Page 52 of 180

Civil Law Q&As (2007-2013)


hectorchristopher@yahoo.com
mother is herself illegitimate. Shelly
cannot
inherit
in
representation
of
Cherry because Shelly is just an adopted
child of Cherry. In representation, the
representative must not only be a legal
heir of the person he is representing but
also of the decedent from whom the
represented
person
is
supposed
to
inherit. In the case of Shelly, while she
is a legal heir of Cherry by virtue of
dbaratbateladot@gmail.com
SUGGESTED ANSWER:
A testator may dispose of by will the free
portion of his estate. Since the legitime
of JCP is 1/8 of the estate, SGO is  of
the estate and that of HBR and RVC is
of the hereditary estate under Art 889 of
the NCC, the remaining 1/8 of the estate
is the free portion which the testator
may dispose of by will.
adoption, she is not a legal heir of
Ramon.
Adoption
creates
a
personal
legal relation only between the adopting
Legitime; Compulsory Heirs (2008)

parent and the adopted child (Teotico v.


Del Val, 13 SCRA 406, 1965. Michelle
No.
cannot inherit from Ramon, because she
worker, was coming home to the Philippines
was adopted not by Ramon but by Dessa.
after working for so many years in the
In the eyes of the law, she is not related
Middle East. He had saved P100.000 in his
to Ramon at all. Hence, she is not a legal
saving account in Manila which intended to
heir of Ramon. Hans and Gretel are not
use to start a business in his home
entitled to inherit from Ramon, because
country. On his flight home, Ernesto had a
they are barred by Art. 992 NCC. Being
fatal heart attack. He left behind his
illegitimate
they
widowed mother, his common-law wife and
legitimate
their twins sons. He left no will, no debts,
relatives of their father Larry. Ramon is
no other relatives and no other properties
a legitimate relative of Larry who is the
except the money in his saving account.
legitimate father.
Who are the heirs entitled to inherint from
cannot
children
inherit

from
of
Larry,
the
him
XII.
Ernesto,
and
how
an
overseas
much
should
Filipino
each
receive?(3%)
Legitimes; Compulsory Heirs (2012)
SUGGESTED ANSWER:
No.VIII.b) How can RJP distribute his estate
The mother and twin sons are entitled to
by will, if his heirs are JCP, his wife; HBR
inherit from Ernesto. Art. 991 of the
and RVC, his parents; and an illegitimate
Civil Code, provides that if legitimate
child, SGO?
Never Let The Odds Keep You From Pursuing What You Know In Your Heart You Were Me
ant To Do.-Leroy Satchel Paige
Page 53 of 180

Civil Law Q&As (2007-2013)


hectorchristopher@yahoo.com
dbaratbateladot@gmail.com
ascendants are left, the twin sons shall
The attestation clause stated the will was
divide the inheritance with them taking
signed on the same occasion by Arthur and
one-half of the estate. Thus, the widowed
his instrumental witnesses who all signed
mother gets P50,000.00 while the twin
in the presence of each other, and the
sons shall receive P25,000.00 each. The
notary public who notarized the will. There
common-law wife cannot inherit from
are no marginal signatures or pagination
him because when the law speaks "widow
appearing on any of the 3 pages. Upon his
or widower" as a compulsory heir, the
death, it was discovered that apart from the
law refers to a legitimate spouse (Art.
house and lot, he had a P 1 million account
887, par 3, Civil Code).
deposited with ABC bank.
(A). Was Erica preterited? (1%)
Preterition; Disinheritance (2008)
No.X.
Arthur
executed
a
will
SUGGESTED ANSWER:
which

Erica cannot be preterited. Art. 854 of


contained only: (i) a provision disinheriting
the
Civil
Code
provides
that
only
his daughter Bernica for running off with a
compulsory heirs in the direct line can
married man, and (ii) a provision disposing
be preterited.
of his share in the family house and lot in
favor of his other children Connie and Dora.
(B). What other defects of the will, if any,
He did not make any provisions in favor of
can cause denial of probate? (2%)
his wife Erica, because as the will stated,
she would anyway get of the house and
lot as her conjugal share. The will was very
brief and straightforward and both the
above provisions were contained in page 1,
which Arthur and his instrumental witness,
signed at the bottom. Page 2 contained the
attestation clause and the signatures, at
the bottom thereof, of the 3 instrumental
witnesses which included Lambert, the
driver of Arthur; Yoly, the family cook, and
Attorney Zorba, the lawyer who prepared
the will. There was a 3rd page, but this only
contained the notarial acknowledgement.
SUGGESTED ANSWER:
The other defects of the will that can
cause its denial are as follows: (a) Atty.
Zorba, the one who prepared the will was
one of the three witnesses, violating the
three-witnesses rule;
(b) no marginal
signature at the last page; (c ) the
attestation did not state the number of

pages upon which the will is written;


and,
(d)
no
pagination
appearing
correlatively in letters on the upper part
of the three pages (Azuela v. C.A., G.R.
Never Let The Odds Keep You From Pursuing What You Know In Your Heart You Were Me
ant To Do.-Leroy Satchel Paige
Page 54 of 180

Civil Law Q&As (2007-2013)


hectorchristopher@yahoo.com
dbaratbateladot@gmail.com
No. 122880, 12 Apr 2006 and cited cases
(B). Between Marian and the baby, who is
therein, Art 805 and 806, Civil Code).
presumed to have died ahead? (1%)
(C). Was the disinheritance valid? (1%)
SUGGESTED ANSWER:
SUGGESTED ANSWER:
Marian is presumed to have died ahead
of the baby. Art. 43 applies to persons
Yes, the disinheritance was valid. Art.
who are called to succeed each other.
919, par 7, Civil Code provides that
The proof of death must be established
"when a child or descendant leads a
by positive or circumstantial evidence
dishonorable
like
derived from facts. It can never be
running off with a married man, there is
established from mere inference. In the
sufficient cause for disinheritance."
present case, it is very clear that only
or
disgraceful
life,
Marian and Pietro were hacked with
bolos. There was no showing that the
Succession;
Proof

of
Death
between
persons called to succeed each other
(2008)
No. II. At age 18, Marian found out that she
was pregnant. She insured her own life and
named
her
unborn
child
as
her
sole
beneficiary. When she was already due to
give birth, she and her boyfriend Pietro, the
father
of
her
unboarn
child,
were
kidnapped in a resort in Bataan where they
were vacationing. The military gave chase
and after one week, they were found in an
abandoned hut in Cavite. Marian and Pietro
were hacked with bolos. Marian and the
baby delivered were both found dead, with
the baby s umbilical cord already cut. Pietro
survived.
baby was also hacked to death. The
baby s death could have been due to lack
of nutrition.
ALTERNATIVE ANSWER:
The baby is presumed to have died ahead
of Marian. Under Par. 5, rule 131, Sec. 5
(KK) of the Rules of Court, if one is
under 15 or above 60 and the age of the
other is in between 15 and 60, the latter
is presumed to have survived. In the
instant case, Marian was already 18
when

she
found
out
that
she
was
pregnant. She could be of the same age
or maybe 19 years of age when she gave
birth.
(C). Will Pietro, as surviving biological
father of the baby, be entitled to claim the
Never Let The Odds Keep You From Pursuing What You Know In Your Heart You Were Me
ant To Do.-Leroy Satchel Paige
Page 55 of 180

Civil Law Q&As (2007-2013)


hectorchristopher@yahoo.com
dbaratbateladot@gmail.com
proceeds of the life insurance on the life of
Marian? (2%)
Marilyn is not entitled to a share in the
estate of Dr. Lopez.
SUGGESTED ANSWER:
succession,
Pietro, as the biological father of the
baby, shall be entitled to claim the
proceeds of life insurance of the Marian
because he is a compulsory
heir of his
child.
Succession; Rule on Survivorship (2009)
Dr.
For purpose of
Lopez
and
his
son
Roberto are presumed to have died at
the same time, there being no evidence
to prove otherwise, and there shall be no
transmission of rights from one to the
other (Article 43, NCC). Hence, Roberto,
inherited nothing from his father that
Marilyn
would
Roberto
.The
in
turn
children
inherit

of
from
Roberto,
however, will succeed their grandfather,
Dr. Lopez ,in representation of their
No. II. Dr. Lopez, a 70-year old widower,
father Roberto and together Roberto will
and his son Roberto both died in a fire that
receive 1/3 of the estate of Dr. Lopez
gutted their home while they were sleeping
since their father Roberto was one of the
in their air-conditioned rooms. Robertos
three children of Dr. Lopez . Marilyn
wife, Marilyn, and their two children were
cannot represent her husband Roberto
spared because they were in the province at
because the right is not given by the law
the time. Dr. Lopez left an estate worth
to a surviving spouse.
P20M and a life insurance policy in the
amount of P1M with his three children --As to the proceeds of the insurance on
one of whom is Roberto --- as beneficiaries.
the life of Dr. Lopez:
Marilyn is now claiming for herself and her
Since succession is
children her husbands share in the estate
regards
left by Dr. Lopez, and her husbands share
provisions of the Rules of Court (Rule
in the proceeds of Dr. Lopezs life insurance

131, Sec. 3 , [jj] [5] ) on survivorship


policy. Rule on the validity of Marilyns
shall apply. Under the Rules, Dr. Lopez,
claims with reasons. (4%)
who was 70 years old, is presumed to
SUGGESTED ANSWER :
As to the Estate of Dr. Lopez:
the
not involved as
insurance
contract,
the
have died ahead of Roberto who is
presumably between the ages 15 and 60.
Having survived the insured, Roberto s
right as a beneficiary became vested
Never Let The Odds Keep You From Pursuing What You Know In Your Heart You Were Me
ant To Do.-Leroy Satchel Paige
Page 56 of 180

Civil Law Q&As (2007-2013)


hectorchristopher@yahoo.com
dbaratbateladot@gmail.com
upon the death of Dr. Lopez. When
should be given effect must be denied.
Roberto died after Dr. Lopez, his right to
The said cancellation has revoked the
receive the insurance became part of his
entire will as nothing remains of the will
hereditary estate, which in turn was
after the name of Rosa was cancelled.
inherited in equal shares by his legal
Such cancellation is valid revocation of
heirs, namely, his spouse and children.
the
Therefore, Roberto s children and his
authentication by the full signature of
spouse
the testator to be effective.
are
one-third
entitled
share
in
to
Roberto s
the
insurance
proceeds.
will

and
does
not
require
However, if the cancellation of Rosas
name was not done by the testator
himself, such cancellation shall not be
effective and the will in its original tenor
Wills; Holographic Wills; Insertions &
shall remain valid. The effectively of the
Cancellations (2012)
holographic will cannot be left to the
mercy of unscrupulous third parties.
No.VII.a) Natividads holographic will, which
had only one (1) substantial provision, as
The writing of Gregorios name as sole
first written, named Rosa as her sole heir.
heir was ineffective, even though written
However, when Gregorio presented it for
by the testator himself, because such is
probate, it already contained an alteration,
an
naming Gregorio, instead of Rosa, as sole
authentication by the full signature of
heir,
by
the testator to be valid and effective. Not
Natividads signature. Rosa opposes the
having an authenticated, the designation
probate
of Gregorio as an heir was ineffective,
but
without

alleging
authentication.
authentication
such
She
lack
claims
of
proper
that
the
unaltered form of the will should be given
alteration
that
requires
(Kalaw v. Relova, G.R. No. L-40207, Sept
28, 1984).
effect. Whose claim should be granted?
Explain. (5%)
SUGGESTED ANSWER:
Wills; Holographic Wills; Probate (2009)
It depends. If the cancellation of Rosas
No.VI. On December 1, 2000, Dr. Juanito
name in the will was done by the
Fuentes
testator himself, Rosas claimed that the
wherein he gave nothing to his recognized
holographic will in its original tenor
illegitimate son, Jay. Dr. Fuentes left for the
executed
a
holographic

will,
Never Let The Odds Keep You From Pursuing What You Know In Your Heart You Were Me
ant To Do.-Leroy Satchel Paige
Page 57 of 180

Civil Law Q&As (2007-2013)


hectorchristopher@yahoo.com
dbaratbateladot@gmail.com
United States, passed the New York medical
court shall apply the New Civil Code in
licensure examinations, resided therein,
determining the formal validity of the
and became a naturalized American citizen.
holographic will. The subsequent change
He died in New York in 2007. The laws of
in the citizenship of Dr. Fuentes did not
New York do not recognize holographic wills
affect the law governing the validity of
or compulsory heirs.
his will. Under the new Civil Code, which
was the law used by Dr. Fuentes, the law
(A). Can the holographic will of Dr. Fuentes
enforced at the time of execution of the
be admitted to probate in the Philippines?
will shall govern the formal validity of
Why or why not? (3%)
the will (Art. 795, NCC).
SUGGESTED ANSWER:
(B). Assuming that the will is probated in
Yes, the holographic will of Dr. Fuentes
the Philippines, can Jay validly insist that
may be admitted to probate in the
he be given his legitime? Why or why not?
Philippines because there is no public
(3%)
policy violated by such probate. The only

issue at probate is the due execution of


SUGGESTED ANSWER:
the
formal
No, Jay cannot insist because under New
validity of the will. As regards formal
York law he is not a compulsory heir
validity, the only issue the court will
entitled to a legitime.
will
which
includes
the
resolve at probate is whether or not the
will was executed in accordance with the
The
form prescribed by the law observed by
determines who his heirs are, the order
the testator in the execution of his will.
that
For
the
successional rights are, and whether or
may
not a testamentary disposition in his will
observe the law of the place where the
is valid (Art 16, NCC). Since, Dr. Fuentes
will was executed (Art 17, NCC), or the
was a US citizen, the laws of the New
formalities of the law of the place where
York determines who his heirs are. And

he
since
purposes
Philippines,
resides,
of
an
or
probate
alien
in
testator
according
to
the
national
they
the
law
succeed,
New
of
the
how
York
testator
much
law
does
their

not
formalities of the law of his own country,
recognize the concept of compulsory
or in accordance with the Philippine
heirs, Jay is not a compulsory heir of Dr.
Civil Code (Art. 816, NCC). Since Dr.
Fuentes entitled to a legitime.
Fuentes executed his will in accordance
with the Philippine law, the Philippine
Never Let The Odds Keep You From Pursuing What You Know In Your Heart You Were Me
ant To Do.-Leroy Satchel Paige
Page 58 of 180

Civil Law Q&As (2007-2013)


hectorchristopher@yahoo.com
dbaratbateladot@gmail.com
Wills; Joint Wills (2008)
SUGGESTED ANSWER:
No. XI. John and Paula, British citizens at
No. The testamentary dispositions are
birth, acquired Philippine citizenship by
not valid because (a) omission of Mary, a
naturalization after their marriage. During
legitimate
their
preterition
marriage
the
couple
acquired
child,
is
which
tantamount
shall
annul
to
the
substanial landholdings in London and in
institution of Peter and Paul as heirs
Makati. Paula bore John three children,
(Art.
Peter, Paul and Mary. In one of their trips
disposition that Peter and Paul could not

to London, the couple executed a joint will


dispose of nor divide the London estate
appointing each other as their heirs and
for more than 20 years is void (Art. 870,
providing that upon the death of the
Civil Code).
854,
Civil
Code);
and,
(b)
the
survivor between them the entire estate
would go to Peter and Paul only but the two
could not dispose of nor divide the London
estate as long as they live. John and Paul
died
tragically
in
the
London
Subway
terrorist attack in 2005. Peter and Paul
filed a petition for probate of their parent s
will before a Makati Regional Trial Court.
(A). Should the will be admitted to probate?
(2%)
Wills; Joint Wills; Probate (2012)
No.VII.b) John Sagun and Maria Carla
Camua, British citizens at birth, acquired
Philippine
citizenship
by
naturalization
after their marriage. During their marriage,
the

couple
acquired
substantial
landholdings in London and in Makati.
Maria
begot
three
(3)
children,
Jorge,
Luisito, and Joshur. In one of their trips to
SUGGESTED ANSWER:
London, the couple executed a joint will
No. The will cannot be admitted to
appointing each other as their heirs and
probate because a joint will is expressly
providing that upon the death of the
prohibited under Art. 818 of the Civil
survivor between them, the entire estate
Code. This provision applies John and
would go to Jorge and Luisito only but the
Paula became Filipino citizens after their
two (2) could not dispose of nor divide the
marriage.
London estate as long as they live. John
and Maria died tragically in the London
(B).
Are
valid? (2%)
the
testamentary

dispositions
subway terrorist attack in 2005. Jorge and
Luisito filed a petition for probate of their
parents will before a Makati Regional Trial
Never Let The Odds Keep You From Pursuing What You Know In Your Heart You Were Me
ant To Do.-Leroy Satchel Paige
Page 59 of 180

Civil Law Q&As (2007-2013)


hectorchristopher@yahoo.com
Court. Joshur vehemently objected because
he was preterited.
dbaratbateladot@gmail.com
SUGGESTED ANSWER:
Assuming the will of John and Maria was
(1) Should the will be admitted to probate?
valid, the testamentary prohibition on the
Explain. (2%)
division of the London estate shall be valid
but only for 20 years. Under Arts 1083 and
494 of the NCC, a testamentary disposition
SUGGESTED ANSWER:
No, the will should not be admitted to
probate. Since the couples are both
Filipino citizens, Art 818 and 819 of the
of the testator cannot forbid the partition of
all or part of the estate for a period longer
than twenty (20) years.
NCC shall apply. Said articles prohibits
the execution of joint wills and make
them void, even though authorized of
Wills; Prohibition to Partition of a Cothe country where they were executed.
Owned Property (2010)
(2) Are the testamentary dispositions valid?
No.I. True or False.
Explain. (2%)
(B) X, a widower, died leaving a will stating
that the house and lot where he lived
SUGGESTED ANSWER:
Since the joint will is void, all the
testamentary disposition written therein
are also void. However, if the will is
valid, the institutions of the heirs shall
be
annulled
because

Joshur
was
cannot be partitioned for as long as the
youngest of his four children desires to stay
there. As coheirs and co-owners, the other
three may demand partition anytime. (1%)
SUGGESTED ANSWER:
preterited. He was preterited because he
will receive nothing from the will, will
FALSE, The other three co heirs may
receive nothing in testacy, and the facts
not anytime demand the partition of the
do not show that he received anything as
house and lot since it was expressly
an advance on his inheritance. He was
provided by the decedent in his will that
totally excluded from the inheritance of
the same cannot be partitioned while his
his parents.
youngest child desires to stay there.
Article 1083 of the New Civil Code allows
(3) Is the testamentary prohibition against
a decedent to prohibit, by will, the
the division of the London estate valid?
partition of a property and his estate for
Explain. (1%)
a period not longer than 20 years no
Never Let The Odds Keep You From Pursuing What You Know In Your Heart You Were Me
ant To Do.-Leroy Satchel Paige
Page 60 of 180

Civil Law Q&As (2007-2013)


hectorchristopher@yahoo.com
matter what his reason maybe. Hence,
dbaratbateladot@gmail.com
(B). Act as a witness to a will? (1%)
the three co-heir cannot demand its
partition at anytime but only after 20
years from the death of their father.
Even if the deceased parent did not
leave
a
will,
if
the
house
and
lot
constituted their family home, Article
159 of the Family Code prohibits its
partition for a period of ten (10) years, or
for
as
long
as
there
is
a
minor
beneficiary living in the family home.
SUGGESTED ANSWER:
Stevie cannot be a witness to a will. Art.
820 of the Civil Code provides that "any
person of sound mind and of the age of
eighteen years or more, and not blind,
deaf or dumb, and able to read and write,
may be a witness to the execution of a
will.
(C). In either of the above instances, must

the will be read to him? (1%)


Wills;
Notarial
Wills;
Blind
Testator;
Requisites (2008)
No. XIV. Stevie was born blind. He went to
school for the blind, and learned to read in
Baille
Language.
He
Speaks
English
fluently. Can he:
SUGGESTED ANSWER:
If Stevie makes a will, the will must be
read to him twice, once by one of the
subscribing witnesses, and again, by the
notary public before whom the will is
acknowledged (Art. 808, Civil Code).
(A). Make a will? (1%)
SUGGESTED ANSWER:
Assuming that he is of legal age (Art.
797, Civil Code) and of sound mind at
the time of execution of the will (Art.
798, Civil Code), Stevie, a blind person,
can make a notarial will, subject to
compliance with the "two-reading rule"
(Art. 808, Civil Code) and the provisions
of Arts. 804, 805 and 806 of the Civil
Code.
Wills; Testamentary Disposition; Period
to Prohibit Partition (2008)
No. XI. John and Paula, British citizens at
birth, acquired Philippine citizenship by
naturalization after their marriage. During
their
marriage
the
couple
acquired

substanial landholdings in London and in


Makati. Paula bore John three children,
Peter, Paul and Mary. In one of their trips
to London, the couple executed a joint will
appointing each other as their heirs and
Never Let The Odds Keep You From Pursuing What You Know In Your Heart You Were Me
ant To Do.-Leroy Satchel Paige
Page 61 of 180

Civil Law Q&As (2007-2013)


hectorchristopher@yahoo.com
dbaratbateladot@gmail.com
providing that upon the death of the
that she can sign her full name later. While
survivor between them the entire estate
the
would go to Peter and Paul only but the two
experienced a stomach ache and kept going
could not dispose of nor divide the London
to the restroom for long periods of time.
estate as long as they live. John and Paul
Hannah, while waiting for her turn to sign
died
Subway
the will, was reading the 7th Harry Potter
terrorist attack in 2005. Peter and Paul
book on the couch, beside the table on
filed a petition for probate of their parent s
which everyone was signing. Benjamin,
will before a Makati Regional Trial Court.
aside from witnessing the will, also offered
tragically
in
the
London
will
was
being
signed,

Roberta
to notarize it. A week after, Clara was run
(C). Is the testamentary prohibition against
over by a drunk driver while crossing the
the division of the London estate valid? (2%)
street in Greenbelt.
SUGGESTED ANSWER:
May the will of Clara be admitted to
No. the testamentary prohibition against
the division of the London estate is void
(Art.
870,
Civil
Code).
A
probate? Give your reasons briefly. (10%)
SUGGESTED ANSWER:
testator,
however, may prohibit partition for a
Probate
should
be
denied.
The
period which shall not exceed twenty
requirement that the testator and at
(20) years (Art. 870 in relation to Art.
least three (3) witnesses must sign all in
494, par 3, Civil Code).
the "presence" of one another was not
complied with. Benjamin who notarized
the will is disqualified as a witness,
Wills; Witnesses to
required;

a
Thumbmark
Will, Presence
as
Signature
(2007)
the three witnesses (Cruz v. Villasor, 54
SCRA 31, 1973). The testatrix and the
other witnesses signed the will not in
No.VI. Clara, thinking of her mortality,
drafted a will and asked Roberta, Hannah,
Luisa
hence he cannot be counted as one of
and
Benjamin
to
be
witnesses.
During the day of signing of her will, Clara
fell down the stairs and broke her arms.
Coming from the hospital, Clara insisted on
signing her will by thumb mark and said
the presence of Roberta because she was
in the restroom for extended periods of
time. Inside the restroom, Roberta could
not have possibly seen the testatrix and
the other witnesses sign the will by
merely casting her eyes in the proper
direction (Jaboneta v. Gustilo, 5 Phil
541, 1906; Nera v. Rimando, 18 Phil
Never Let The Odds Keep You From Pursuing What You Know In Your Heart You Were Me
ant To Do.-Leroy Satchel Paige
Page 62 of 180

Civil Law Q&As (2007-2013)


451,
1914).
hectorchristopher@yahoo.com
Therefore,
the
dbaratbateladot@gmail.com
testatrix
Because the Picasso painting reminded
signed the will in the presence of only
Angie of him, Brad in his will bequeathed
two witnesses, and only two witnesses
the painting to Angie. Brad died in 1995.
signed the will in the presence of the
Saddened by Brad s death, Jennifer asked
testatrix and of one another.
for the Picasso painting as a remembrance
of him. Angie refused and claimed that
It is to be noted, however, that the
Brad, in his will, bequeathed the painting to
thumb mark intended by the testator to
her. Is Angie correct? Why or why not?
be his signature in executing his last will
(10%)
and
testament
is
valid
(Payad
v.
Tolentino, 62 Phil 848, 1936; Matias v.

SUGGESTED ANSWER:
Salud, L-104 Phil 1046, 23 June, 1958).
The problem, however, states that Clara
NO. Angie is not correct. The Picasso
"said that she can sign her full name
painting is not given or donated by
later;" Hence, she did not consider her
Jennifer to Brad. She merely "placed it
thumb mark as her "complete" signature,
in his bedroom." Hence, she is still the
and intended further action on her part.
owner of the painting. Not being the
The testatrix and the other witness
owner of the Picasso painting, Brad
signed
of
cannot validly bequeath the same to
Hannah, because she was aware of her
Angie (Art. 930, NCC). Even assuming
function and role as witness and was in a
that the painting was impliedly given or
position to see the testatrix and the
donated
other witnesses sign by merely casting
donation is nevertheless void for not
her eyes in the proper direction.
being in writing. The Picasso painting
the
will
in

the
presence
Donation
Donations; Formalities; In Writing (2007)
by
Jennifer
to
Brad,
the
must be worth more than 5,000 pesos.
Under Art. 748, NCC, the donation and
acceptance of a movable worth more
than 5,000 pesos must be in writing,
No. VIII. In 1986, Jennifer and Brad were
otherwise the donation is void. The
madly in love. In 1989, because a certain
donation being void, Jennifer remained
Picasso painting reminded Brad of her,
the owner of the Picasso painting and
Jennifer acquired it and placed it in his
Brad could not have validly disposed of
bedroom. In 1990, Brad and Jennifer broke
said painting in favor of Angie in his will.
up. While Brad was mending his broken
heart, he met Angie and fell in love.
ALTERNATIVE ANSWER:
Never Let The Odds Keep You From Pursuing What You Know In Your Heart You Were Me
ant To Do.-Leroy Satchel Paige
Page 63 of 180

Civil Law Q&As (2007-2013)


hectorchristopher@yahoo.com
dbaratbateladot@gmail.com
YES. Angie is correct. Even assuming
illegal and impossible donations imposed
that there was void donation because the
in an onerous donation shall annul the
same was not in writing, Brad was in
donation (Art. 1183, NCC). This is so,
uninterrupted possession of the Picasso
because onerous donations are governed
painting from 1989 to 1995, lasting for
by the law on contracts (Art. 733, NCC).
six (6) years prior to his death. Brad has
already
acquired
ownership
painting
through
prescription.
Under
ownership
of
of
the
acquisitive
Art.
movables
1132,
NCC,
prescribes

through continuous possession for four


(4) years in good faith and for eight (8)
years without need of other conditions.
A void donation may be the basis of
Donation; Inter Vivos (2013)
No.V. Josefa executed a deed of donation
covering a one-hectare rice land in favor of
her
daughter,
Jennifer.
The
deed
specifically provides that:
possession in the concept of owner and
"For and in consideration of her love
of just title for purposes of acquisitive
and service Jennifer has shown and
prescription.
given
to
me,
I
hereby
freely,
voluntarily and irrevocably donate to
her my one-hectare rice land covered
Donations;
Illegal
&
by TCT No. 11550, located in San
Impossible
Fernando, Pampanga. This donation
Conditions (2007)
No.I. Distinguish the following concepts:

shall take effect upon my death."


The deed also contained Jennifer s signed
(B). Illegal and impossible conditions in a
acceptance,
and
an
attached notarized
simple donation v. illegal and impossible
declaration by Josefa and Jennifer that the
conditions in an onerous donation. (5%)
land will remain in Josefa s possession and
cannot be alienated, encumbered, sold or
SUGGESTED ANSWER:
disposed of while Josefa is still alive.
Illegal and impossible conditions in a
Advise Jennifer on whether the deed is a
simple donation are considered as not
donation inter vivos or mortis causa and
written. Such conditions, shall therefore,
explain the reasons supporting your advice.
be disregarded but the donation remains
(8%)
valid (Art. 727, NCC). On the other hand,
Never Let The Odds Keep You From Pursuing What You Know In Your Heart You Were Me
ant To Do.-Leroy Satchel Paige
Page 64 of 180

Civil Law Q&As (2007-2013)


hectorchristopher@yahoo.com
SUGGESTED ANSWER:
dbaratbateladot@gmail.com
same should be harmonized with its
express irrevocability (Austria-Magat v.
The donation is a donation inter vivos.
CA, G.R. No. 106755, Feb 1, 2002).
When
ALTERNATIVE ANSWER:
the
donor
intends
that
the
donation shall take effect during the
lifetime
of
the
donor,
though
the
The donation is donation mortis causa.
property shall not be delivered till after
the
donors
death,
this
shall
be
a
The

deed
clearly
states
that
the
donation inter vivos (Art. 729, Civil
donation shall take effect upon the
Code).
death of the donor, Josefa. The donor,
moreover, retained ownership of the
The
Civil
Code
prefers
inter
vivos
subject property as it was declared that
transmissions. Moreover, mortis causa
the
property
cannot
be
alienated,
donations should follow the formalities
encumbered, sold or disposed of while
of a will (Art. 728, Civil Code). Here there
the donor is still alive.
is no showing that such formalities were
followed. Thus, it is favorable to Jennifer
As the donation is in the nature of a
that the deed is a donation inter vivos.

mortis causa disposition, the formalities


of a will should have been complied with
Furthermore, what is most significant in
under
determining the type of donation is the
otherwise, the donation is void and
absence of stipulation that the donor
would produce no effect (The National
could
the
Treasure of the Philippines v. Vda. de
contrary, the deeds expressly declare
Meimban, G.R. No. L-61023, Aug 22,
them
1984).
revoke
to
be
the
donation;
irrevocable,
a
on
quality
Art.
728
of
the
Civil
Code,
absolutely incompatible with the idea of

conveyances
mortis
causa
where
Property
revocability is the essence of the act, to
the
extent
that
a
testator
cannot
Accretion; Alluvium (2008)
lawfully waive or restrict his right of
revocation. The provisions of the deed of
No. IX. The properties of Jessica and Jenny,
donation which state that the same will
who are neighbors, lie along the banks of
only take effect upon the death of the
the Marikina River. At certain times of the
donor and that there is a prohibition to
year, the river would swell and as the water
alienate, encumber, dispose, or sell the
recedes, soil, rocks and other materials are
Never Let The Odds Keep You From Pursuing What You Know In Your Heart You Were Me
ant To Do.-Leroy Satchel Paige
Page 65 of 180

Civil Law Q&As (2007-2013)


deposited
properties.
on
hectorchristopher@yahoo.com
Jessica s
This
swelling, receding and depositing soil and
man, it is man-made accretion and a
other materials being deposited on the
part of the public domain (Tiongco v.
neighbors properties have gone on for
Director of Lands, 16 C.A. Rep 211, cited
many years. Knowing his pattern, Jessica
in Nazareno v. C.A., G.R. No. 98045, 26
constructed a concrete barrier about 2
June 1996). Thus, Jessica cannot legally
meters
claim ownership of the additional 2
property
the
land but is also the consequences of the
direct and deliberate intervention of
her
of
Jenny s
river
from
pattern
and
dbaratbateladot@gmail.com
line

and
extending towards the river, so that when
meters
the water recedes, soil and other materials
because
are trapped within this barrier. After several
barrier about 2 meters from her property
years, the area between Jessica s property
causing
line to the concrete barrier was completely
materials when the water recedes. In
filled
increasing
other words, the increase in her property
Jessica s property by 2 meters. Jenny s
was not caused by nature but was manproperty, where no barrier was constructed,
made.
with
soil,
effectively
of
land
she
along
her
constructed
deposits
of
a

soil
property
concrete
and
other
also increased by one meter along the side
(B). If Jessica s and Jenny s properties are
of the river.
registered,
will
the
benefit
of
such
(A). Can Jessica and Jenny legally claim
registration extend to the increased area of
ownership over the additional 2 meters and
their properties? (2%)
one meter, respectively, of land deposited
along their properties?(2%)
SUGGESTED ANSWER:
SUGGESTED ANSWER:
If the properties of Jessica and Jenny
are
registered,
the
the
land
increased
deposited along her property. Art. 457 of
Accretion
the Civil Code provides that "to the

become registered land because there is


owners of lands adjoining the banks of
a specific technical description of the lot
river belong the accretion which they
in its Torrens title. There must be a
gradually receive from the effects of the
separate application for registration of
current of the water." Where the land is
the alluvial deposits under the Torrens
not formed solely by the natural effect of
System (Grande v. CA, G.R. No. L-17652,
the water current of the river bordering
30 June, 1962).
meter
of
area
does
of
extend
their
not
to
such
registration
one
not
of
Only Jenny can claim ownership over
additional
does
benefit

the
properties.
automatically
Never Let The Odds Keep You From Pursuing What You Know In Your Heart You Were Me
ant To Do.-Leroy Satchel Paige
Page 66 of 180

Civil Law Q&As (2007-2013)


hectorchristopher@yahoo.com
dbaratbateladot@gmail.com
(C). Assume the two properties are on a cliff
Ulpiano built three huts on this additional
adjoining the shore of Laguna Lake. Jessica
area,
and Jenny had a hotel built on the
children live. On this same area, Ulpiano
properties. They had the erath and rocks
and his family planted peanuts, monggo
excavated from the properties dumped on
beans
the adjoining shore, giving rise to a new
regularly paid taxes on the land, as shown
patch of dry land. Can they validly lay claim
by tax declarations, for over thirty years.
where
he
and
and
his
vegetables.
two
married
Ulpiano
also
to the patch of land? (2%)
When Marciano learned of the increase in
SUGGESTED ANSWER:
the size of the land, he ordered Ulpiano to
demolish the huts, and demanded that he

No. Jessica and Jenny cannot validly lay


be paid his share in the proceeds of the
claim to the patch of land because in
harvest. Marciano claims that under the
order to acquire land by accretion, there
Civil Code, the alluvium belongs to him as a
should
actual
registered riparian owner to whose land the
continuity of the accretion to the land of
accretion attaches, and that his right is
the riparian owner caused by natural ebb
enforceable against the whole world.
be
a
natural
and
and flow of the current of the river
(Delgado v. Samonte, CA-G.R. No. 34979(A). Is Marciano correct? Explain. (3%)
R, 10 Aug 1966).
SUGGESTED ANSWER:
Marcianos contention is correct. Since
that accretion was deposited on his land
Accretion; Rights of the Riparian Owner
by the action of the waters of the river
(2009)
and he did not construct any structure
No.XVI. Marciano is the owner of a parcel of
land through which a river runs out into
the sea. The land had been brought under
the Torrens System, and is cultivated by
Ulpiano and his family as farmworkers
therein. Over the years, the river has
brought silt and sediment from its sources
up in the mountains and forests so that

gradually the land owned by Marciano


increased
in
area
by
three
hectares.
to increase the deposition of soil and
silt, Marciano automatically owns the
accretion. His real right of ownership is
enforceable
against
the
whole
world
including Ulpiano and his two married
children. Although Marcianos land is
registered, the three (3) hectares land
deposited
through accretion was not
automatically
unregistered
registered.
land,
it
is
As
an
subject
to
acquisitive prescription by third persons.
Never Let The Odds Keep You From Pursuing What You Know In Your Heart You Were Me
ant To Do.-Leroy Satchel Paige
Page 67 of 180

Civil Law Q&As (2007-2013)


hectorchristopher@yahoo.com
dbaratbateladot@gmail.com
production, gathering and preservation
Although Ulpiano and his children live in
of the fruits (Art 443, NCC).
the three (3) hectare unregistered land
owned
by
Marciano,
they
are
farm
He may also ask for reimbursement of
workers; therefore, they are possessors
the taxes he has paid, as these are
not in the concept of owners but in the
charges on the land owned by Marciano.
concept of mere holders. Even if they
This obligation is based on a quasipossess the land for more than 30 years,
contract (Art 2175, NCC).
they cannot become the owners thereof
through
extraordinary
acquisitive
prescription, because the law requires
possession in the concept of the owner.
Payment of taxes and tax declaration are
not enough to make their possession one
in the concept of owner. They must
repudiate the possession in the concept
of holder by executing unequivocal acts
of repudiation amounting to ouster of
Marciano, known to Marciano and must
be

proven
evidence.
by
Only
clear
and
then
convincing
would
his
possession become adverse.
against Marciano? Explain. (3%)
faith, because he knew he does not own
the land, he will lose the three huts he
faith
meter land (Lot A) in Paranaque. The land
now has a fair market value of Pl,200,000.
CRC likewise sold to the spouses Rodriguez,
a 700-square meter land (Lot B) which is
adjacent to Lot A. Lot B has a present fair
market value of P1,500,000.
on Lot B, relying on their presentation of
the CRC sales agent that it is the property
of their house did the spouses Dela Cruz
Although Ulpiano is a possessor in bad
bad
sold to the spouses Del a Cruz a500-square
they purchased. Only upon the completion
SUGGESTED ANSWER:
in
No.VIII. Ciriaco Realty Corporation (CRC)
The spouses Dela Cruz constructed a house
(B). What rights, if any, does Ulpiano have
built
Builder; Good Faith; Requisites (2013)
and

make
an
accounting of the fruits he has gathered,
he has the right to deduct from the
value of the fruits the expenses for
discovered that they had built on Lot B
owned by the spouses Rodriguez, not on Lot
A that they purchased. They spent P 1
000,000 for the house.
As their lawyer, advise the spouses Dela
Cruz on their rights and obligations under
the given circumstances, and the recourses
Never Let The Odds Keep You From Pursuing What You Know In Your Heart You Were Me
ant To Do.-Leroy Satchel Paige
Page 68 of 180

Civil Law Q&As (2007-2013)


hectorchristopher@yahoo.com
dbaratbateladot@gmail.com
and options open to them to protect their
However, the builder cannot be obliged
interests. (8%)
to
buy
the
considerable
SUGGESTED ANSWER:
land
more
if
its
than
value
that
of
is
the
building.. In such case, he shall pay
Based on the fact as stated, the spouses
Dela Cruz as builders and the spouses
Rodriguez as land owners, are both in
good faith. The spouses Dela Cruz are
reasonable rent of the owner of the land
does
not
choose
to
appropriate
the
building or trees after proper indemnity

(Art 448, Civil Code).


builder in good faith because before
The house constructed by the spouses
constructing the house they exercised
Dela Cruz is considered as a useful
due diligence by asking the Agent of CRC
expense, since it increased the value of
the location of the lot A, and they relied
the lot. As such, should the spouses
on the information given by the agent
Rodriguez decides to appropriate the
who is presumed to know the identity of
house,
the lot purchased by the Dela Cruz
entitled to the right of retention pending
spouses (Pleasantville v. CA, 253 SCRA
reimbursement of the expenses they
10, 1996). On the other hand, there is no
incurred or the increase in value which
showing that the land owners, spouse
the thing may have acquired by reason
Rodriguez acted in bad faith. The facts
of
do not show that the building was done
Code). Thus, the spouses Dela Cruz may
with
without
demand P1,000,000.00 as payment of
opposition on their part (Art 453, Civil
the expenses in building the house or

Code). The good faith is always presumed


increase in value of the land because of
(Art. 527, Civil Code).
the house as a useful improvement, as
their
knowledge
and
The owner of the land on which anything
has been built, sown, or planted in good
faith shall have the right:
(1) to appropriate as his own the works
the
the
spouses
improvement
Dela
(Art
Cruz
546,
are
Civil
may be determined by the court form
the evidence presented during the trial
(Depra v. Dumlao, 136 SCRA 475, 1985;
Technogas Phils v. CA, 268 SCRA 5,
1997).
after payment of the indemnity provided
for in Art 546 and 548, or
(2) to oblige the one who built to pay the
price of the land.
Never Let The Odds Keep You From Pursuing What You Know In Your Heart You Were Me
ant To Do.-Leroy Satchel Paige
Page 69 of 180

Civil Law Q&As (2007-2013)


Easement;
Prescription;
hectorchristopher@yahoo.com
Acquisitive
Prescription (2009)
dbaratbateladot@gmail.com
In 2006, Brand0 fenced off his property,
thereby blocking Andres access to the
national highway. Andres demanded that
No. XI. TRUE or FALSE. Answer TRUE if
part of the fence be removed to maintain
the statement is true, or FALSE if the
his
statement is false. Explain your answer in
(pathway A), but Brando refused, claiming
not more than two (2) sentences.
that there was another available pathway
(C). Acquisitive prescription of a negative
easement runs from the time the owner of
the dominant estate forbids, in a notarized
document, the owner of the servient estate
old
access
route
to
the
highway
(pathway B) for
highway. Andres
has defects, is
inconvenient to

ingress and egress to the


countered that pathway B
circuitous, and is extremely
use.

from executing an act which would be


To settle their dispute, Andres and Brando

lawful without the easement. (1%)


hired Damian, a geodetic and civil engineer,
to survey and examine the two pathways
SUGGESTED ANSWER:
True. In negative easements, acquisitive
prescription runs from the moment the
owner of the dominant estate forbade, by
an
instrument
acknowledged
before
notary public, the owner of the servient
estate from executing an act which
would be lawful without the easement
(Art. 621, NCC).
and
the
determine
prejudicial
surrounding
the
way
areas,
shortest
through
and
the
and
the
to
least
servient
estates. After the survey, the engineer
concluded that pathway B is the longer
route and will need improvements and
repairs, but will not significantly affect the
use of Brando s property. On the other
hand, pathway A that had long been in
place, is the shorter route but would
significantly affect the use of Brando s
Easement; Right of Way (2013)

property.
No.VII.In 2005, Andres built a residential
In light of the engineer s findings and the
house on a lot whose only access to the
circumstances of the case, resolve the
national highway was a pathway crossing
parties

right of way dispute. (6%)

Brando s property. Andres and others have


been using this pathway (pathway A) since
SUGGESTED ANSWER:
1980.
Never Let The Odds Keep You From Pursuing What You Know In Your Heart You Were Me
ant To Do.-Leroy Satchel Paige
Page 70 of 180

Civil Law Q&As (2007-2013)


hectorchristopher@yahoo.com
dbaratbateladot@gmail.com
Andres is not entitled to the easement of
(Pathway B). Second, the right of way
right of way for Pathway A. Pathway B
obtained (Pathway A) is not the least
must be used.
prejudicial
to
Brandos
property,
as
evidence by the reports of the geodetic
The owner of a dominant estate may
and civil engineer.
validly obtain a compulsory right of way
only
after
he
has
established
the
existence of four requisites, to wit:
When
there
adequate
is
outlet
already
from
an

the
existing
dominant
estate to the public highway, even if the
(1) The (dominant) estate is surrounded
said outlet, for one reason or another, be
by other immovables and is without
inconvenient,
adequate outlet to a public highway;
another servitude is entirely unjustified
(2)
After
payment
of
the
proper
indemnity;
proprietors own acts; and
need
to
open
up
(Costabella Corporation v. CA, G.R. No.
80511, Jan 25, 1991). The rule that the
easement
(3) The isolation was not due to the
the
of
right
of
way
shall

be
established at the point least prejudicial
to the servient estate is controlling
(Quimen v. Quimen and CA, G.R. No.
(4) The right of way claimed is at a point
least prejudicial to the servient estate,
and insofar as consistent with this rule,
where the distance from the dominant
estate to the public highway maybe the
shortest (Art 650, civil Code).
112331, May 29, 1996).
(Note: It is not clear from the problem if there
exists an easement in favor of the lot
belonging to Andres and if Brandos lot is
burdened as a servient estate by a right of
way as a servient estate. If there is such an
has
easement burdening Brandos lot, was it
consistently ruled that in case both
created as legal easement or as a voluntary
criteria cannot be complied with, the
easement. If the used pathway was only a
right of way shall be established at the
tolerance, then Brando may close it. Andres
point least prejudicial to the servient
must ask for the constitution of a legal
estate.
easement through Brandos lot by proving
However,
the
Supreme
Court
the four requisites required by Art 649 and
The first and fourth requisites are not
65, Civil Code).
complied with. First, there is another
available outlet to the national highway

Never Let The Odds Keep You From Pursuing What You Know In Your Heart You Were Me
ant To Do.-Leroy Satchel Paige
Page 71 of 180

Civil Law Q&As (2007-2013)


hectorchristopher@yahoo.com
Easement; Right of Way (2010)
dbaratbateladot@gmail.com
easement or servitude, even if the deed
of sale is silent on the matter.
No.XIII. Franz was the owner of Lot E which
was surrounded by four (4) lots one of
(3) The vendee of the property in which a
which Lot C he also owned. He promised
servitude
Ava that if she bought Lot E, he would give
close or put obstructions thereon to
her a right of way in Lot C.
prevent the dominant estate from using
Convinced, Ava bought Lot E and, as
promised, Franz gave her a right of way in
Lot C.
Ava cultivated Lot E and used the right of
way granted by Franz.
Ava later found gainful employment abroad.
or
easement
exists
cannot
it.
(4) Avas working abroad for more than
ten (10) years should not be construed as
non-user, because it cannot be implied
from the fact that she or those she left
behind to cultivate the lot no longer use
the right of way.
On her return after more than 10 years, the
right of way was no longer available to her
Note:
Since
a

right
of
way
is
a
because Franz had in the meantime sold
discontinuous easement, the period of
Lot C to Julia who had it fenced.
ten years of non-user, shall be computed
from the day it ceased to be used under
(A). Does Ava have a right to demand from
Julia the activation of her right of way?
Explain. (2.5%)
SUGGESTED ANSWER:
Act 6341 (2) CC.
(5)
Renunciation
easement
must
or
be
waiver
specific,
of
an
clear,
express and made in a public instrument
in accordance of Art 1358 of the New
Yes. Ava has the right to demand from
Civil Code.
Julia the activation of the right of way,
ALTERNATIVE ANSWER:
for the following reasons:
Yes. Ava has the right to demand from
(1) The easement of the right of way is a

Julia the activation of the right of way.


real right which attaches to, and is
A voluntary easement of right of way,
inseperable from, the estate to which it
like
belongs.
extinguished only by mutual agreement
any
other
contract,
could
be
or by renunciation of the owner of the
(2) The sale of the property includes the
dominant estate. Also, like any other
Never Let The Odds Keep You From Pursuing What You Know In Your Heart You Were Me
ant To Do.-Leroy Satchel Paige
Page 72 of 180

Civil Law Q&As (2007-2013)


contract,
an
hectorchristopher@yahoo.com
easement
is
dbaratbateladot@gmail.com
generally
her in lot C if Ava purchase lot E. The
effective between parties, their heirs and
promise was not reduced to writing (Obra
assignees, except in case where the
v. Baldria, 529 SCRA 621 [2007]). Hence,
rights and obligations arising from the
it was not or could not have been
contract are not transmissible by their
registered as to warn buyers of lot C
nature, or by stipulations or by provision
about the existence of the easement on
of law (Unisource Commercial v. Chung,
the property. Not having been annotated
593 SCRA 530 [2009]).
on the TCT to lot C, the buyer acquired
lot C free from such right of way granted
(B). Assuming Ava opts to demand a right of
to Ava.
way from any of the owners of Lots A, B,
and D, can she do that? Explain. (2.5%)
SUGGESTED ANSWER:
Hidden Treasure (2008)
Yes. Ava has the option to demand a
No. VIII. Adam, a building contractor, was

right of way on any of the remaining lots


engaged by Blas to construct a house on a
of Franz more so after Franz sold lot C to
lot which he (Blas) owns. While digging on
Julia. The essential elements of a legal
the lot in order to lay down the foudation of
right of way under Art 649 and 650 of
the house, Adam hit a very hard object. It
the New Civil Code are complied with.
turned out to be the vault of the old Banco
de las Islas Filipinas. Using a detonation
ALTERNATIVE ANSWER:
device, Adam was able to open the vault
Yes. Ava has the option to demand a
containing old notes and coins which were
right of way from the other lots. The law
in circulation during the Spanish era. While
provides that whenever a piece of land
the notes and coins are no longer legal
acquired by sale, exchange or partition is
tender, they were valued at P100 million
surrounded
the
because of their historical value and the
vendor, exchanger, or co-owner, he shall
coins silver nickel content. The following
be obliged to grant a right of way
filed legal claims over the notes and coins:
by
other

estates
of
without indemnity (Art 652, NCC).
(i). Adam, as finder;
ALTERNATIVE ANSWER:
(ii). Blas, as owner of the property where
No. There was merely a promise to Ava
they were found;
that a right of way shall be granted to
Never Let The Odds Keep You From Pursuing What You Know In Your Heart You Were Me
ant To Do.-Leroy Satchel Paige
Page 73 of 180

Civil Law Q&As (2007-2013)


hectorchristopher@yahoo.com
dbaratbateladot@gmail.com
(iii). Bank of the Philippine Islands, as
present case, Adam, as finder, and Blas,
successor-in-interest of the owner of the
as owner of the land, are entitled to
vault; and
share
50-50
in
the
treasure.
The
government can only claim if it can
(iv). The Philippine Government because of
establish that the notes and coins are of
their historical value.
interest to science or the arts, then it
must pay just price of the things found,
(A). Who owns the notes and coins? (4%)
to be divided equally between Adam and
Blas (Art. 438, Civil Code).
SUGGESTED ANSWER:
The notes and coins are no longer owned
by the Bank of the Philippine Islands,
which has either lost or abandoned the
vault and its contents, and it has not
taken any effort to search, locate or
recover the vault. In any case, since the
vault is in actual possession of Adam,
(B). Assuming that either or both Adam and
Blas are adjudged as owners, will the notes
and coins be deemed part of their absolute
community or conjugal partnership of gains
with their respective spouses? (2%)
SUGGESTED ANSWER:
BPI may attempt, in a judicial action to

If either or both Adam and Blas are


recover, to rebut the presumption of
adjudged as owners, the notes and coins
ownership in favor of Adam and Blas
shall be deemed part of their absolute
(Art. 433, Civil Code). Hidden treasure is
community or conjugal partnership of
any hidden and unknown deposit of
gains with their respective spouses (Art.
money,
117, par 4, FC).
jewelry,
or
other
precious
objects, the lawful ownership of which
does not appear. Given the age and
importance of the items found, it would
be safe to consider the vault, notes and
Mortgage; Public or Private Instrument
coins
(2013)
abandoned
by
BPI
and
its
predecessor (Art. 439, Civil Code). It
belongs to the owner of the land on
which it is found. When the discovery is
made on the property of another, or of
the State and by chance, one-half of it
shall belong to the finder who is not a
trespasser (Art. 438, Civil Code). In the

No.VI. Lito obtained


from Ferdie, payable
secure payment, Lito
mortgage on a Toyota
estate mortgage on a
piece of property.

a loan of P1,000,000
within one year. To
executed a chattel
Avanza and a real
200-square meter

Never Let The Odds Keep You From Pursuing What You Know In Your Heart You Were Me
ant To Do.-Leroy Satchel Paige
Page 74 of 180

Civil Law Q&As (2007-2013)


hectorchristopher@yahoo.com
dbaratbateladot@gmail.com
(A) Would it be legally significant - from the
exceeds Five Hundred pesos (P500.00)
point of view of validity and enforceability must appear in writing, even in private
if the loan and the mortgages were in public
one. However, the requirement is not for
or private instruments? (6%)
validity of the contract, but only for its
greater efficacy.
SUGGESTED ANSWER:
With regard to the chattel mortgage, Art.
From the point of view of validity and
1508,
enforceability,
legal
requires an affidavit of good faith stating
significance if the mortgage was in a
that the chattel mortgage is supposed to
public or private instrument. As for the
stand as security of the loan; thus, for
loan,
significance
the validity of the chattel mortgage, it
except of interest were charged on the
must be in a public document and
loan, in which case, the charging of
recorded
interest must be in writing.
Register in the Register of Deeds. A real

there
there
is
no
would
legal
be
A contract of loan is a real contract and
is perfected upon delivery of the object
of the obligation (Art 1934, Civil Code).
Thus, a contract of loan is valid and
enforceable even if it is neither in a
private nor in a public document.
As a rule, contracts shall be obligatory in
whatever form they
may
have been
entered into provided all the essential
requisites for their validity are present.
With regards to its enforceability, a
contact of loan is not among those
enumerated under Art. 1403 (2) of the
Civil Code, which are covered by the
Statute of Frauds.
It is important to note that under Art.
1358 of the Civil Code, all the other
contracts where the amount involved
the
Chattel
in
the
Mortgage
Chattel
Law,
Mortgage
estate mortgage, under the provisions of
Art. 2125 of the Civil Code, requires that
in order that a mortgage may be validly
constituted the document in which it
appears be recorded. If the instrument is

not
recorded,
the
mortgage
is
nevertheless valid and binding between
the parties. Hence, for validity of both
chattel and real estate mortgages, they
must appear in a public instrument. But
the
purpose
of
enforceability,
it
is
submitted that the form of the contract,
whether in a public or private document,
would
be
immaterial
(Mobil
Oil
v.
Diocaresa, 29 SCRA 656, 1969).
Also, under Art 1358, acts and contracts
which have for their object the creation
or
transmission
of
real
rights
over
immovable property must be in a public
Never Let The Odds Keep You From Pursuing What You Know In Your Heart You Were Me
ant To Do.-Leroy Satchel Paige

Page 75 of 180

Civil Law Q&As (2007-2013)


hectorchristopher@yahoo.com
dbaratbateladot@gmail.com
document for greater efficacy and a real
right. Possession may be the real right of
estate mortgage is a real right over
possession or jus possessiones or it can
immovable property.
be merely the right to possess or jus
possedendi, which are among the basic
rights of ownership. If the real right of
possession is possession in the concept
Occupation vs. Possession (2007)
of
owner,
limitations,
No.I. Distinguish the following concepts:
but
it
subject
may
to
ripen
certain
into
full
ownership of the thing or property right
through
(A). Occupation v. possession. (5%)
acquisitive
prescription
depending on whether it is a case of
SUGGESTED ANSWER:
ordinary or extraordinary prescription
and whether the property is movable or

Occupation
is
an
original
mode
of
immovable.
acquiring ownership (Art. 712, NCC).
Things appropriable by nature which are
without an owner, such as animals that
Ownership; Co-Ownership (2009)
are the object of hunting and fishing,
hidden
treasure
and
abandoned
No. XI. TRUE or FALSE. Answer TRUE if
movables, are acquired by occupation
the statement is true, or FALSE if the
(Art. 713, NCC). However, ownership of a
statement is false. Explain your answer in
piece of land cannot be acquired by
not more than two (2) sentences.
occupation (Art. 714, NCC).
(D). The renunciation by a co-owner of his
ALTERNATIVE ANSWER:
undivided share in the co-owned property
in lieu of the performance of his obligation
Occupation
is
a
mode
of

acquiring
to contribute to taxes and expenses for the
dominion by the seizure of corporeal
preservation
things which have no owner, with the
constitutes dacion en pago. (1%)
of
the
property
intention of acquiring the ownership
thereof.
It
is
an
original
mode
of
SUGGESTED ANSWER:
acquiring ownership upon seizure of a
True, Under the Civil Code, a co-owner
res nullius by the occupant who has the
may renounce his share in the co-owned
intention to become the owner thereof.
property in lieu of paying for his share in
Possession, on the other hand, is the
the
holding of the thing or an enjoyment of a
preservation of the co-owned property.
taxes
and

expenses
for
the
Never Let The Odds Keep You From Pursuing What You Know In Your Heart You Were Me
ant To Do.-Leroy Satchel Paige
Page 76 of 180

Civil Law Q&As (2007-2013)


In
effect,
there
is
hectorchristopher@yahoo.com
dacion
en
pago
dbaratbateladot@gmail.com
SUGGESTED ANSWER:
because the co-owner is discharging his
monetary obligation by paying it with
Yes, Cathy can lawfully ask for the
his non-monetary interest in the codemolition
owned property. The fact that he is
there are two or more heirs, the whole
giving up his entire interest simply
estate
means that he is accepting the value of
partition, owned in common by such
his interest as equivalent to his share in
heirs, subject to the payment of debts of
the taxes and expenses of preservation.
the deceased (Art. 1078, Civil Code),
of
of
Bobby s
the
house.

decedent,
is,
Where
before
Under the rules on co-ownership, "none
of
Ownership; Co-Ownership (2008)
the
co-owners
shall,
without
the
consent of the others make alterations
in the thing owned in common, even
No. VI. Alex died without a will, leaving only
though benefits for all would results
an undeveloped and untitled lot in Tagiug
therefrom." In Cruz v. Catapang, G.R. No.
City. He is survived by his wife and 4
164110, 12 Feb., 2008, the Court held
children. His wife told the children that she
that "alterations include any act of strict
is waiving her share in the property, and
dominion
allowed Bobby, the eldest son who was
construction of a house." In the present
about to get married, to construct his
case, of Alex is the real owner of the
house on  of the lot, without however
undeveloped and untitled lot in Taguig,
obtaining the consent of his siblings. After
co-ownership is created among his wife

settlement of Alex s estate and partition


and four children over said property
among the heirs, it was discovered that
upon his death. Since the construction
Bobby s house was constructed on the
of the house by Bobby was done without
portion allocated to his sister, Cathy asked
obtaining the consent of his siblings, the
Bobby to demolish his house and vacate
alteration effected is illegal. Bobby is
the portion alloted to her. In leiu of
considered to be in bad faith and as a
demolition, Bobby offered to purchase from
sanction for his conduct, he can be
Cathy the lot portion on which his house
compelled
was constructed. At that time, the house
remove
constructed was valued at P350.000.
expense.
(A). Can Cathy lawfully ask for demolition of
(B). Can Bobby legally insist on purchasing
Bobby s house? (3%)
the land? (2%)
or
by
the
ownership
Cathy
to

structure
such
demolish
at
his
as
or
own
Never Let The Odds Keep You From Pursuing What You Know In Your Heart You Were Me
ant To Do.-Leroy Satchel Paige
Page 77 of 180

Civil Law Q&As (2007-2013)


hectorchristopher@yahoo.com
SUGGESTED ANSWER:
dbaratbateladot@gmail.com
nature and object to remain at a fixed
place on a river, lake or coast." Since the
No.
Bobby
cannot
legally
insist
on
floating
platform
is
a
petroleum
purchasing the land. Being in bad faith,
operation
he has no option to pay for the price of
remain permanently where it is situated,
the lot (Art. 450, Civil Code).
even if it is tethered to a ship which is
facility,
it
is
intended
to
anchored to the seabed.
ALTERNATIVE ANSWER:
Property; Movable or Immovable (2007)
The platform is a movable property
No.II. Manila Petroleum Co. owned and

because it is attached to a movable


operated a petroleum operation facility off
property,
the coast of Manila. The facility was located
merely anchored to the seabed. The fact
on a floating platform made of wood and
that the vessel is merely anchored to the
metal,
sea
upon
which
was
permanently
bed
i.e.
only
the
vessel
shows
which
that
it
is
was
not
attached the heavy equipment for the
intended to remain at a fixed place;
petroleum operations and living quarters of
hence, it remains a movable property. If
the crew. The floating platform likewise

the intention was to make the platform


contained a garden area, where trees,
stay permanent where it was moored, it
plants
The
would not have been simply tethered to
platform was tethered to a ship, the MV
a vessel but itself anchored to the
101, which was anchored to the seabed.
seabed.
Please briefly give the reason for your
(B). Are the equipment and living quarters
answers. (10%)
movable or immovable property?
(A).Is the platform movable or immovable
SUGGESTED ANSWER:
and
flowers
were
planted.
property?
The thing and living quarters of the crew
SUGGESTED ANSWER:
are immovable property under Art. 415
The platform is an immovable property
under Art. 415 (9) NCC, which provides
that
"docks
and
structures
which,
though floating, are intended by their

(3)
NCC, classifies as an immovable
"everything attached to an immovable in
a fixed manner, in such a way that it
cannot be separated therefrom without
breaking the material or deterioration of
Never Let The Odds Keep You From Pursuing What You Know In Your Heart You Were Me
ant To Do.-Leroy Satchel Paige
Page 78 of 180

Civil Law Q&As (2007-2013)


hectorchristopher@yahoo.com
dbaratbateladot@gmail.com
the object." Both the equipment and the
The trees, plants and flowers planted in
living quarters are permanently attached
the garden area of the platform are
to
an
immovable property under Art. 415 (2)
immovable. The equipment can also be
NCC which classifies as an immovable
classified
property
the
platform
as
an
which
is
immovable
also
property
"trees,
plants
and
growing
under Art. 415 (5) NCC because such
fruits, while they are attached to the
equipment are "machinery, receptacles,
land or form an integral part of an

instruments or implements intended by


immovable,
the
facility.
owner
of
the
tenement
for
an
the
petroleum
operation
industry or works which may be carried
on in a building or on a piece of land and
which tend directly to meet the needs of
the industry or works." It is logically
assumed that the petroleum industry
may be carried on in a building or on a
piece
of
land
and
the
platform
is
analogous to a building.
ALTERNATIVE ANSWER:
The trees, plants and flowers planted in
the garden area of the platform are
movable property because they are not
permanently attached t the land and do
not
form
immovable.

an
integral
The
part
of
an
is
not
an
platform
immovable property for the same reason
ALTERNATIVE ANSWER:
already given in the Alternative Answer
The equipment and living quarters of the
to Item (a) above.
crew are movable properties since they
are attached to a platform which is also
Land Titles and Deeds
a movable property, because it is simply
attached to a
vessel is likewise a
movable property since it was merely
Acquisition
of
Lands;
Sale
of
Real
Property to an Alien (2009)
anchored on the seabed only shows that
it is not intended to remain at a fixed
No.XIX. In 1972, Luciano de la Cruz sold to

place;
Chua Chung Chun, a Chinese citizen, a
hence,
it
remains
a
movable
property.
parcel of land in Binondo. Chua died in
1990, leaving behind his wife and three
(C). Are the trees, plants and flowers
children,
immovable or movable property?
naturalized Filipino citizen. Six years after
Chuas
SUGGESTED ANSWER:
one
death,
of
whom,
the
heirs
Julian,
is
executed
a
an
extrajudicial settlement of estate, and the
parcel of land was allocated to Julian. In
Never Let The Odds Keep You From Pursuing What You Know In Your Heart You Were Me
ant To Do.-Leroy Satchel Paige
Page 79 of 180

Civil Law Q&As (2007-2013)


hectorchristopher@yahoo.com
2007, Luciano filed suit to recover the land
dbaratbateladot@gmail.com
Non-Registrable Properties (2007)
he sold to Chua, alleging that the sale was
void
because
it
contravened
the
Constitution which prohibits the sale of
private lands to aliens. Julian moved to
dismiss
the
suit
on
grounds
of pari
delicto, laches and acquisitive prescription.
Decide the case with reasons. (4%)
No.IV.
What
properties
are
not
registrable? (5%)
Supply this information.
SUGGESTED ANSWER:
The
SUGGESTED ANSWER:
(B).
following
properties

are
not
registrable:
The case must be dismissed. Julian, who
is a naturialized Filipino citizen and to
(1.) Properties of the Public dominion;
whom the property was allocated in a n
(2.) Properties for public use or public
extra-judicial partition of the estate, is
service;
now the owner of the property. The
defect in ownership of the property of
(3.)
Inalienable
Julians alien father has already been
domain;
lands
of
the
public
cured by its transfer to Julian. It has
been validated by the transfer of the
(4.) Military installations, civil and quasiproperty to a Filipino citizen. Hence,
public lands; and
there
is
no
more
violation
of

the
Constitution because the subject real
property is now owned by a Filipino
citizen (Halili v. CA, 287 SCRA 465,
[1998]). Further, after the lapse of 35
(5.) All lands not classified as alienable
and disposable.
ALTERNATIVE ANSWER:
year, laches has set in and the motion to
(1).
dismiss may be granted, for the failure of
intended
Luciano to question the ownership of
canals, rivers, torrents, ports and bridges
Chua before its transfer of ownership to
constructed by the State, banks, shores,
Julian.
roadsteads, and the like, are incapable of
Properties
private
for
of
public
public
use,
appropriation,
dominium
like
much
roads,
less
registration (Art. 420 NCC). This includes
public markets, public plazas, municipal
streets

and
public
buildings
Never Let The Odds Keep You From Pursuing What You Know In Your Heart You Were Me
ant To Do.-Leroy Satchel Paige
Page 80 of 180

Civil Law Q&As (2007-2013)


hectorchristopher@yahoo.com
dbaratbateladot@gmail.com
(Municipality of Antipolo v. Zapanta, 133
(7.) Lands reclaimed by the government
SCRA 820, 1986; Martinez v. CA, 56
from the sea, lakes, or other bodies of
SCRA 647, 1974; Navera v. Quicho, 5
water are disposed or acquisible only by
SCRA 454, 1962).
lease
and
not
otherwise,
under
the
Public Land Act.
(2.) Lands proclaimed or classified as
forest, timberlands, mineral lands and
national parks. Under Sec 2, Art XII,
Constitution of the Philippines, these
Prescription;
Acquisitive
Prescription
lands are inalienable.
(2008)
(3.) Lands that are reserved by law or
No. VII. Anthony bought a piece of untitled
Presidential proclamation for military,
agricultural land from Bert. Bert, in turn,
civic or quasi-public purpose, Under Sec
acquired the property by forging carlo s

88, Chapter XII of the Public Land Act,


signature in a deed of sale over the
such lands shall be inalienable and shall
property. Carlo had been in possession of
not be subject to occupation, entry, sale,
the property for 8 years, declared it for tax
lease or other disposition.
purposes, and religiously paid all taxes due
on the property. Anthony is not aware of
(4.) In general, all lands of the public
the defect in Bert s title, but has been in
domain that has not been classified as
actual physical possession of the property
alienable and disposable under the Public
from the time he bought it from Bert, who
Land Act.
had
never
been
in
possession
of
the
property for one year.
(5.) Lands that form part of the seabed,
riverbed or lakebed. These lands are not
(A). Can Anthony acquire ownership of the
susceptible to private appropriation.
property by acquisitive prescription? How
many more years does he have possess it to
(6.) Foreshore lands is that strip of land
acquire ownership? (2%)

that lies between the high and low water


marks
and
alternately
wet
and
dry
SUGGESTED ANSWER:
according to the flow of the tide belong
to the public domain, and can only be
Yes, Anthony can acquire ownership of
acquired by lease if not needed by the
the
government for public or quasi-public
prescription.
purposes.
Anthony is a buyer/possessor in good
property
through
In
the
acquisitive
present
case,
Never Let The Odds Keep You From Pursuing What You Know In Your Heart You Were Me
ant To Do.-Leroy Satchel Paige
Page 81 of 180

Civil Law Q&As (2007-2013)


hectorchristopher@yahoo.com
dbaratbateladot@gmail.com
faith because he was not aware of the
shall have a right to a part of the
defect in Bert s title (Art. 526, Civil
expenses of cultivation, and to a part of
Code). As such, Anthony can acquire
the net harvest of the standing crops,
ownership and other real rights over
both in proportion to the time of the
immovable
possession (Art 545, Civil Code).
property
through
open,
continuous possession of 10 years (Art.
1134, Civil Code). Anthony needs nine
(9) more years of possession, in addition
to his one (1) year of possession in good
faith.
(B).If Carlo is able to legally recover his
property,
can
he
require
Anthony
to
account for all the fruits he has harvested
from the property while in possession? (2%)
If Carlo is able to legally recover his
property, he cannot require Anthony to
account for all the fruits harvested from
the property. Anthony is entitled to the
fruits harvested in good faith before his
possession was legally interrupted (Art.
544, Civil Code).

property when Carlo recovers possession,


can Carlo appropriate them? (2%)
issuance of a writ of possession over a
foreclosed real property prescribe in five (5)
years? (5%)
Yes, it prescribes in five (5) years. If the
real property mortgaged is judicially
foreclosed,
the
action
for
judicial
foreclosure should be filed within a
period of ten (10) years. The request for
issuance of a writ of possession should
be filed upon motion of the winning
appropriate
judgment of foreclosure. The writ of
possession is an order commanding the
sheriff to place a person named therein
in possession of real property (BPI v.
Icot. G.R. No. 168081, Oct 12, 2009).
SUGGESTED ANSWER:
can
No.IX.a) Does the right to request for the
bidder within five (5) years after the
(C).If there are standing crops on the
Carlos
Property Mortgage (2012)
SUGGESTED ANSWER:
SUGGESTED ANSWER:
Yes,
Prescription; Judicially Foreclosed Real
only
a
portion of the standing crops on the
property once he recovers possession.

Anthony being a possessor in good faith,


Never Let The Odds Keep You From Pursuing What You Know In Your Heart You Were Me
ant To Do.-Leroy Satchel Paige
Page 82 of 180

Civil Law Q&As (2007-2013)


Purchaser
in
hectorchristopher@yahoo.com
Good
Faith;
Mortgaged
dbaratbateladot@gmail.com
of Deeds under Act. 3344 and obtained a
Property (2008)
tax declaration in its name.
No. XIX. Juliet offered to sell her house and
(A). Was Dehlma a purchaser in good faith?
lot, together with all the furniture and
(2%)
appliances
therein
to
Dehlma.
Before
agreeing to purchase the property, Dehlma
went to the Register of Deeds to verify
Juliet s title. She discovered that while the
property was registered in Juliet s name
under
the
amended
Land
by
the
Registration
Property
Act,

as
Registration
Decree, it property, Dehlma told Juliet to
redeem the property from Elaine, and gave
her an advance payment to be used for
purposes of realesing the mortgage on the
property. When the mortgage was released,
Juliet executed a Deed of Absolute Sale
over the property which was duly registered
with the Registry of Deeds, and a new TCT
was issued in Dehlma s name. Dehlma
immediately took possession over the house
and
lot
and
the
movables
therein.
Thereafter, Dehlma went to theAssessor s
Office to get a new tax declaration under
SUGGESTED ANSWER:
Yes, Dehlma is a purchaser in good faith.
In
the
present
case,
before
Dehlma
bought the property, she went to the
Register of Deeds to verify Juliet s title.
When she discovered that the property
was mortgaged to Elaine, she gave an
advance payment so that Juliet could
release the mortgage. It was only after
the mortgage was released and free from
the claims of other persons that Dehlma
bought the property. Thus, Dehlma is a
purchaser in good faith (Mathay v. CA,
G.R. No. 115788, 17 Sept, 1998).
(B). Who as between Dehlma and XYZ Bank
has a better right to the house and lot? (2%)
SUGGESTED ANSWER:
her name. She was surprised to find out

that the property was already declared for


Between Dehlma and XYZ Bank, Dehlma
tax purposes in the name of XYZ Bank
has a better right to the house and lot.
which had foreclosed the mortgage on the
After the release of the mortgage, the
property before it was sold to her. XYZ
Deed of Absolute Sale was registered and
Bank
the
a new title was issued in Dehlma s name.
foreclosure sale of the property. At that
Act 3344 is applicable exclusively to
time, the property was still unregistered but
instruments resulting from agreement of
XYZ Bank registered the Sheriff s Deed of
parties thereto and does not apply to
Conveyance in the day book of the Register
deeds
was
also
the
purchaser
in
of
a
sheriff
conveying
to
a

Never Let The Odds Keep You From Pursuing What You Know In Your Heart You Were Me
ant To Do.-Leroy Satchel Paige
Page 83 of 180

Civil Law Q&As (2007-2013)


hectorchristopher@yahoo.com
dbaratbateladot@gmail.com
purchaser unregistered lands sold to him
land registration and acquisition of title to
under execution (Williams v. Suer, 49
land.
Phil. ,534).
following items:
(C). Who owns the movables inside the
(A). What is the governing law? (5%)
The
manual
should
include
the
house? (2%)
SUGGESTED ANSWER:
SUGGESTED ANSWER:
The
Dehlma
owns
is
the
Land
Registration Act as amended by Property
when she acquired the house and lot
Registration Decree (Act 496 as amended
from
by PD 1529).
all
movables

law
because
Juliet,
the
governing
the
furniture
and
appliances therein were included in the
sale. As owner of the real property,
[Note: It is respectfully recommended
Dehlma also owns the movables found
that full credit be given to examinees
therein (Art. 542, Civil Code).
who did not give the exact title or
number of the law but merely stated a
description of the law.]
Registration; Governing Law (2007)
ALTERNATIVE ANSWER:
No.IV.
Property
In general, the governing law relating to
development
registration and acquisition of title to
company engaged in developing and selling
land is Act 496 of 1902 as amended by
subdivisions,
PD 1529, otherwise known as Property
Bedrock
Development
Land
Corp.

is
&
a
condominium
units
and
industrial estates. In order to replenish its
Registration Decree of June 11, 1978.
inventories, it embarked on an aggressive
land banking program. It employed "scouts"
(1.)
who roam all over the Philippines to look for
registration
and conduct investigations on prospective
Torrens System by voluntary ordinary
sites
judicial proceedings.
for
acquisition
and
development,
whether developed, semi-developed or raw
land. The management of Bedrock asks you
as the company counsel to prepare a
manual
containing
a
summary
of
the
Chapter
of
III-I

land
governs
title
original
under
the
(2.) Chapter II-II governs compulsory
registration of lands through cadastral
proceedings.
pertinent laws and regulations relating to
Never Let The Odds Keep You From Pursuing What You Know In Your Heart You Were Me
ant To Do.-Leroy Satchel Paige
Page 84 of 180

Civil Law Q&As (2007-2013)


hectorchristopher@yahoo.com
dbaratbateladot@gmail.com
(3.) Section 103 governs registration of
(c) Replacement of lost or destroyed
homestead, sales, free patent under CA
owner s duplicate certificate of title.
No. 141, as amended, otherwise known
(9.)
as the Public Land Act.
R.A.
No.
reconstitution
(4.) Section 104 governs registration of
certificates
of
land
26
of
governs
lost
or
judicial
destroyed
originals of the certificate of title.
transfers,
emancipation patents and Certificates of
(10.)
Land Ownership Award (CLOA) under
administrative reconstitution of lost or
Comprehensive Land Reform Law.
destroyed original certificates of title.
(5.) Chapter V governs the registration of

(11.)
land dealings on registered land like
registration
conveyances,
unregistered private lands.
transfers,
mortgages,
leases, powers of attorney, trusts and
similar contracts inter vivos.
R.A.
No.
Section
of
6732
113
governs
governs
instruments
the
affecting
(12.) Section 117 governs "consultas,"
where the Register of Deeds refuses to
(6.) Chapter V-II governs the registration
register a deed or when he is in doubt as
of involuntary dealings on registered
to what action to take on an instrument
land like attachments, adverse claims,
presented for registration.
enforcement of liens on registered land,
notices of lis pendens. (7.) Chapter VI
governs the registration of judgments,
orders and partitions, condemnation in
eminent domain proceedings, judicial
and extra-judicial settlement of estates.
(8.) Sections 107, 108 and 109 govern

petitions
and
registration
actions
after
original
like:
(a).Compulsory
surrender of withheld owner s duplicate
certificate of title;
(b)
Amendment
certificate of title;
Registration;
Party
Who
First
took
Possession (2013)
No.IX.Rica petitioned for the annulment of
her
ten-year
old
marriage
to
Richard.
Richard hired Atty. Cruz to represent him
in the proceedings. In payment for Atty.
Cruz s acceptance and legal fees, Richard
conveyed to Atty. Cruz a parcel of land in
Taguig that he recently purchased with his
and
alteration
of

lotto winnings. The transfer documents


were
duly
signed
and
Atty.
Cruz
Never Let The Odds Keep You From Pursuing What You Know In Your Heart You Were Me
ant To Do.-Leroy Satchel Paige
Page 85 of 180

Civil Law Q&As (2007-2013)


hectorchristopher@yahoo.com
dbaratbateladot@gmail.com
immediately took possession by fencing off
the
liquidation
of
the
absolute
the property s entire perimeter.
community or conjugal partnership of
the spouses as the case may be (Art. 50
Desperately needing money to pay for his
in relation to Art 43 of the Family Code).
mounting legal fees and his other needs
Richard purchased the land with his
and despite the transfer to Atty. Cruz,
lotto winnings during the pendency of
Richard offered the same parcel of land for
the suit for annulment and on the
sale to the spouses Garcia. After inspection
assumption
of the land, the spouses considered it a
governed by
good investment and purchased it from
community
Richard. Immediately after the sale, the
winnings from gambling or betting will
spouses
form part thereof. Also, since the land is
Garcia

commenced
the
that
the
parties
the regime
or
conjugal
of
are
absolute
partnership,
construction of a three-story building over
part
the land, but they were prevented from
conjugal
doing this by Atty. Cruz who claimed he
Rica, it may not be sold or alienated
has a better right in light of the prior
without the consent of the latter and
conveyance in his favor.
any disposition or encumbrance of the
of
property
Is Atty. Cruz s claim correct? (8%)
the
absolute
partnership
of
the

community
of
Richard
community
or
or
and
the
conjugal property without the consent of
the other spouse is void (Art 96 and Art
SUGGESTED ANSWER:
124, Family Code).
No. Atty. Cruz is not correct. At first
glance, it may appear that Atty. Cruz is
the one who has the better right because
he first took possession of the property.
Registration; Requisites; Proof (2013)
However, a lawyer is prohibited under
No.X. Manuel was born on 12 March 1940
Art
in a 1 000-square meter property where he
1491
of
the
Civil
Code
from
acquiring the property and rights which
grew
may be the object of any litigation in
cultivate the land. Michael has lived on the
which they may take part by virtue of
property since the land was opened for

their profession. While the suit is for


settlement
annulment of marriage and it may be
Commonwealth government in 1935, but
urged that the land itself is not the
for some reason never secured any title to
object of the litigation, the annulment of
the property other than a tax declaration in
marriage, if granted, will carry with it
his name. He has held the property through
up
helping
at
his
about
father,
the
time
Michael,
of
the
Never Let The Odds Keep You From Pursuing What You Know In Your Heart You Were Me
ant To Do.-Leroy Satchel Paige
Page 86 of 180

Civil Law Q&As (2007-2013)


hectorchristopher@yahoo.com
dbaratbateladot@gmail.com
the years in the concept of an owner and
ownership
since
his stay was uncontested by others. He has
earlier. However, it is only necessary
also conscientiously and continuously paid
that the land is already declared A & D
the realty taxes on the land.
land at the time for application for
registration
is
June
filed
12,
1945,
(Malabanan
or
v.
Michael died in 2000 and Manuel - as
Republic, G.R. No. 180067, June 30,
Michaels only son and heir -now wants to
2009).
secure and register title to the land in his
own name. He consults you for legal advice
Manuel could also invoke Sec 14 (2) of
as he wants to perfect his title to the land
the
and secure its registration in his name.

registration through ordinary acquisitive


same
Decree,
which
allows
prescription for thirty years, provided,
(A) What are the laws that you need to
however, that the land is patrimonial
consider in advising Manuel on how he can
in character, i.e. already declared by the
perfect his title and register the land in his
government (a) as A & D, and (b) no
name? Explain the relevance of these laws
longer needed for public use or public
to your projected course of action. (4%)
service (Malabanan, supra).
SUGGESTED ANSWER:
Manuel could also file an application for
(Note:
With
all
due
respect,
it
is
recommended that the examiner accept and
give full credit to any of the answers given in
each of the following paragraphs.)
I
would
advice
Manuel

to
file
an
of Pres. Decree No. 1529, or the Property
Registration Decree (PRD), specifically
Sec14 (1) which requires (a) that the land
applied for forms part of the alienable
and disposable (A & D) portion of the
public domain, and (b) that the applicant
been
notorious
thereof
in
open,
possession
under
bona
continuous
and
fide
incomplete
and
occupation
claim
of
of
title
imperfect
through
or
judicial
legalization under Sec. 48 (b) of CA no.
141, or the Public Land Act (PLA). But,
as
application for registration under Sec 14
has
confirmation
held

in
substantial
Malabanan,
difference
there
between
is
no
this
provision and Sec 14 (1) of the PRD.
Both refer to agricultural lands already
classified as alienable and disposable at
the time the application is filed, and
require possession and occupation since
June 12, 1945. The only difference is
that under the PRD, there already exists
a title which is to be confirmed, whereas
under the PLA, the presumption is that
land is still public land (Republic v.
Never Let The Odds Keep You From Pursuing What You Know In Your Heart You Were Me
ant To Do.-Leroy Satchel Paige
Page 87 of 180

Civil Law Q&As (2007-2013)


hectorchristopher@yahoo.com
dbaratbateladot@gmail.com
Aquino, G.R. No. L-33983, January 27,
Manuel has a the burden to overcome
1983).
the presumption of State ownership by
well-nigh
incontrovertible
evidence
Manuel may also invoke vested rights
(Ong
acquired under Rep. Act. No. 1942, dated
March 12, 2008). Accordingly, he must
June 2, 1957, which amended Sec. 48 (b)
show that ht eland is already classified
of the PLA by providing for a prescriptive
as A & D at the time the application for
period
registration is filed and that he has
of
thirty
years
for
judicial
v.
Republic,
G.R.
No.
175746,
confirmation of imperfect title. It must

been
only be demonstrated that possession
thereof in the manner required by law
and occupation commenced on January
since June 12, 1945, or earlier.
in
possession
and
occupation
24, 1947 and the 30-year period was
completed prior to the effectivity of PD
Manuel may tack his possession to that
No. 1073 on January 25, 1977. PD No.
of his predecessor-in-interest (Michael)
1073
and
by the testimony of disinterested and
1945
knowledgeable eyewitnesses. Overt acts
(Republic v. Espinosa, G.R. No. 171514,
of possession may consist in introducing
July 18, 2012).
valuable improvements like fencing the
now
occupation
requires
since
possession
June
12,
land, constructing a residential house
Another alternative is for Manuel to

thereon,
secure
planting fruit bearing trees, declaring
title
through
administrative
cultivating
the
land
and
proceedings under the homestead or free
the
patent provisions of the PLA. The title
paying realty taxes, all of which are
issued has the same efficacy and validity
corroborative proof of possession.
as
a
title
issued
through
land
for
taxation
purposes
and
judicial
proceedings, but with the limitations
To identify the land, he must submit the
that the land cannot be sold or disposed

tracing cloth plan or a duly-certified


of within five years from the issuance of
blueprint
patent
(Director of Lands v. Reyes, G.R. No. L(Sec.
118,
CA
No.
141,
as
amended).
or
whiteprint
copy
thereof
27594, November 28, 1975; Director of
Lands v. CA and Iglesia ni Cristo, G.R.
(B) What do you have to prove to secure
Manuel s
objectives
and
documentation are necessary? (4%)
No. L-56613, March 14, 1988).
what
To show the classification of the land as
A
SUGGESTED ANSWER:
&
D,
the
application

must
be
accompanied by (1) a CENRO or PENRO
Never Let The Odds Keep You From Pursuing What You Know In Your Heart You Were Me
ant To Do.-Leroy Satchel Paige
Page 88 of 180

Civil Law Q&As (2007-2013)


hectorchristopher@yahoo.com
dbaratbateladot@gmail.com
certification; and (2) a certified true
copy
of
approved
the
by
original
the
classification
DENR
Secretary
I will institute the following actions
against Atty. Tan:
(Republic v. Bantigue, G.R.No. 162322,
March
14,
2012).
A
presidential
or
legislative act may also be considered.
(a). A civil action for damage for the
fraudulent transfer of the title in his
name and to recover the value of the
property;
Remedies; Fraud; Rights of Innocent
Purchaser (2009)
No.IX. Before migrating to Canada in 1992,
the spouses Teodoro and Anita entrusted
all their legal papers and documents to
their nephew, Atty. Tan. Taking advantage
of the situation, Atty. Tan forged a deed of
sale, making it appear that he had bought
the couples property in Quezon City. In
2000, he succeeded in obtaining a TCT over
the property in his name. Subsequently,
Atty. Tan sold the same property to Luis,

who built an auto repair shop on the


property. In 2004, Luis registered the deed
of conveyance, and title over the property
was transferred in his name.
(b).
An
action
against
the
National
Treasurer for compensation from the
State Assurance Fund which is set aside
by law to pay those who lose their land
suffer damages as a consequence of the
operation of the Torrens system;
(c). A criminal action for forgery or
falsification of public document;
(d).
A
complaint
with
the
Supreme
Court/Integrated Bar of the Philippines
to
disbar
or
suspend
him
or
other
disciplinary action for violation or the
Code of Professional Ethics.
Any action against Luis will not prosper
In 2006, the spouses Teodoro and Anita
because he is an innocent purchaser for
came to the Philippines for a visit and

value. The Title to the land he bought


discovered what had happened to their
was already in the name of the person
property. They immediately hire you as
who sold the property to him, and there
lawyer. What action or actions will you
is nothing on the title which will make
institute in order to vindicate their rights?
him suspect about the fraud committed
Explain fully. (4%)
by Atty. Tan.
SUGGESTED ANSWER:
Never Let The Odds Keep You From Pursuing What You Know In Your Heart You Were Me
ant To Do.-Leroy Satchel Paige
Page 89 of 180

Civil Law Q&As (2007-2013)


hectorchristopher@yahoo.com
Contracts
dbaratbateladot@gmail.com
Rescission of Contract; Fortuitous Event
(2008)
Contract to Sell vs. Conditional Contract
of Sale (2012)
No.XVIII. AB Corp. entered into a contract
with XY Corp. whereby the former agreed to
No.X.a) A contract to sell is the same as a
construct
the
research
and
laboratory
conditional contract of sale. Do you agree?
facilities of the latter. Under the terms of
Explain your answer. (5%)
the contract, AB Corp. agreed to complete
the facility in 18 months, at the total
SUGGESTED ANSWER:
contract price of P10 million. XY Corp. paid
No. A contract to sell is a species of
conditional sale. The contract to sell
does not sell a thing or property; it sells
the right to buy property. A conditional
sale is a sale subject to the happening or
performance of a condition, such as
payment of the full purchase price, or
the performance of other prestation to
give, to do or not to do. Compliance with
the condition automatically gives the
right to the vendee to demand the
delivery of the object of the sale. In a
contract
to
sell,

however,
the
compliance with the condition does not
automatically sell the property to the
vendee. It merely gives the vendee the
50% of the total contract price, the balance
to be paid upon completion of the work. The
work stated immediately, but AB Corp. later
experienced work slippage because of labor
unrest
in
his
company.
AB
Corp. s
employees claimed that they are not being
paid on time; hence, the work slowdown. As
of the 17th month, work was only 45%
completed. AB Corp. asked for extension of
time, claiming that its labor problems is a
case of fortuitous event, but this was
denied by XY Corp. When it became certain
that the contruction could not be finished
on time, XY Corp. sent written notice
cancelling the contract, and requiring AB
Corp. to immediately vacate the premises.
right to compel the vendor to execute
(A). Can the labor unrest be considered a
the deed of absolute sale.
fortuitous event? (1%)
SUGGESTED ANSWER:
No.
The
labor
unrest
cannot
be
considered a fortuitous event under Art.
1174 of the Civil Code. A fortuitous
event should occur independent of the
Never Let The Odds Keep You From Pursuing What You Know In Your Heart You Were Me

ant To Do.-Leroy Satchel Paige


Page 90 of 180

Civil Law Q&As (2007-2013)


will
of
the
debtor
hectorchristopher@yahoo.com
or
without
his
participation or aggravation (Paras, Civil
dbaratbateladot@gmail.com
statement is false. Explain your answer in
not more than two (2) sentences.
Code Annotated, vol. IV, 2000 ed., p
159). As mentioned in the facts, labor
(A). A clause in an arbitration contract
unrest of the employees was caused by
granting one of the parties the power to
AB Corp. s failure to pay its employees
choose more arbitrators than the other
on time.
renders the arbitration contract void. (1%)
(B).
Can
XY
Corp.
unilaterrally
and
SUGGESTED ANSWER:
True. The Civil Code provides that Any
immediately cancel the contract? (2%)
clause giving one of the parties power to

SUGGESTED ANSWER:
choose more arbitrators than the other
is void and of no effect (Art 2045, NCC).
No, XY Corp. cannot unilaterally and
Obligations
immediately cancel the contract. In the
absence of any stipulation for automatic
rescission, rescission must be judicial
(Art. 1191, Civil Code).
(C).
Must
AB
Corp.
Extinguishment; Compensation (2009)
No.XV. Sarah had a deposit in a savings
return
the
50%
downpayment? (2%)
SUGGESTED ANSWER:
account with Filipino Universal Bank in the
amount
of
five
million
pesos
(P5,000,000.00). To buy a new car, she
obtained a loan from the same bank in the
amount of P1,200,000.00, payable in twelve
AB Corp. need not return the 50% down
monthly installments. Sarah issued in favor
payment because 45% of the work was
of the bank post-dated checks, each in the
already completed, otherwise, XY Corp.
amount of P100,000.00, to cover the twelve

would be unjustly enriching itself at the


monthly installment payments. On the
expense of AB Corp.
third,
fourth
and
fifth
months,
the
corresponding checks bounced.
The
Stipulation; Arbitration Clause (2009)
bank
then
declared
the
whole
obligation due, and proceeded to deduct the
amount
of
one
million
pesos
No. XI. TRUE or FALSE. Answer TRUE if
(P1,000,000.00) from Sarahs deposit after
the statement is true, or FALSE if the
notice to her that this is a form of
Never Let The Odds Keep You From Pursuing What You Know In Your Heart You Were Me
ant To Do.-Leroy Satchel Paige
Page 91 of 180

Civil Law Q&As (2007-2013)


hectorchristopher@yahoo.com
compensation allowed by law. Is the bank
dbaratbateladot@gmail.com
Extinguishment; Compensation (2008)
correct? Explain. (4%)
No. XV. Eduardo was granted a loan by
SUGGESTED ANSWER:
XYZ Bank for the purpose of improving a
No, the bank is not correct. While the
building
Bank is correct about the applicability of
Eduardo, executed the promissory note
compensation, it was not correct as to
("PN") in favor of the bank, with his friend
the amount compensated.
Recardo as co-signatory. In the PN, they
both
which
XYZ
acknowledged
leased
that
from
him.
they
are
A bank deposit is a contract of loan,
"individually and collectively" liable and
where the depositor is the creditor and
waived the need for prior demand. To
the bank the debtor. Since Sarah is also

secure the PN, Recardo executed a real


the debtor of the bank with respect to
estate mortgage on his own property. When
the loan, both are mutually principal
Eduardo defaulted on the PN, XYZ stopped
debtors and creditors of each other. Both
payment of rentals on the building on the
obligation
and
ground that legal compensation had set in.
liquidated but only up to the extent of
Since there was still a balance due on the
P300,000.00 (covering the unpaid third,
PN
fourth and fifth monthly installments).
foreclosed the real estate mortgage over
The entire one million was not yet due
Recardo s property. Recardo opposed the
because the loan has no acceleration
foreclosure on the ground that he is only a
clause in case of default. And since there
co-signatory; that no demand was made
is
upon him for payment, and assuming he is
no
are
due,
retention
commenced
or

the
rentals,
XYZ
the
half the balance of the loan. Further,
debtor, then all the requisites of legal
Recardo said that when the bank invoked
compensation are present but only up to
compensation between the reantals and the
the amount of P300,000.00. The bank,
amount of the loan, it amounted to a new
therefore, may deduct P300,000.00 from
contract or novation, and had the effect of
Sarahs
extinguishing the security since he did not
compensation.
in
due
deposit
person
applying
liable, his liability should not go beyond
bank
third
controversy
after
and
communicated
by
demandable

time
by
to
way
of
give his consent (as owner of the property
under the real estate mortgage) thereto.
(A). Can XYZ Bank validly assert legal
compensation? (2%)
Never Let The Odds Keep You From Pursuing What You Know In Your Heart You Were Me
ant To Do.-Leroy Satchel Paige
Page 92 of 180

Civil Law Q&As (2007-2013)


hectorchristopher@yahoo.com
SUGGESTED ANSWER:
dbaratbateladot@gmail.com
ground that legal compensation had set in.
Since there was still a balance due on the
Yes, XYZ Bank can validly assert legal
PN
compensation. In the present case, all of
foreclosed the real estate mortgage over
the elements of legal compensation are
Recardo s property. Recardo opposed the
present: (1) XYZ Bank is the creditor of
foreclosure on the ground that he is only a
Eduardo while Eduardo is the lessor of
co-signatory; that no demand was made
XYZ Bank; (2) both debts consist in a
upon him for payment, and assuming he is
sum of money, or if the things due are
liable, his liability should not go beyond
consumable, they be of the same kind,
half the balance of the loan. Further,
and also of the same quality if the latter
Recardo said that when the bank invoked
has been stated; (3) the two debts be
compensation between the reantals and the
due;
and
amount of the loan, it amounted to a new
demandable, and (5) over neither of them
contract or novation, and had the effect of

there be any retention or controversy,


extinguishing the security since he did not
commenced
give his consent (as owner of the property
(4)
they
by
be
liquidated
third
persons
and
communicated in due time to the debtor
after
applying
the
rentals,
XYZ
under the real estate mortgage) thereto.
(Art. 1279, Civil Code).
(C). Does Recardo have basis under the
Civil Code for claiming that the original
contract was novated? (2%)
Extinguishment; Novation (2008)
No. XV. Eduardo was granted a loan by
SUGGESTED ANSWER:
XYZ Bank for the purpose of improving a
No. Recardo has no basis for claiming
building
him.
novation of the original contract when
Eduardo, executed the promissory note

the bank invoked compensation because


("PN") in favor of the bank, with his friend
there was simply partial compensation
Recardo as co-signatory. In the PN, they
(Art. 1290, Civil Code) and this would
both
not bar the bank from recovering the
which
XYZ
acknowledged
leased
that
from
they
are
"individually and collectively" liable and
remaining balance of the obligation.
waived the need for prior demand. To
secure the PN, Recardo executed a real
ALTERNATIVE ANSWER:
estate mortgage on his own property. When
Eduardo defaulted on the PN, XYZ stopped
No. In order that an obligation may be
payment of rentals on the building on the
extinguished by another, it is imperative
Never Let The Odds Keep You From Pursuing What You Know In Your Heart You Were Me
ant To Do.-Leroy Satchel Paige
Page 93 of 180

Civil Law Q&As (2007-2013)


hectorchristopher@yahoo.com
dbaratbateladot@gmail.com
that it be so declared in unequivocal
or creditors (Philippine Airlines v. CA
terms,
and
or
that
the
old
and
new
Amelia
Tan,
G.R.
No.
L-49188,
obligations be on every point compatible
1990). Mere delivery of checks does not
with
discharge
each
other.
Novation
is
never
presumed (Art. 1292, Civil Code).
the
obligation

under
a
judgment. A check shall produce the
effect of payment only when they have
been cashed or where through the fault
Extinguishment;
Payment
of
Check
(2013)
No.VI. Lito obtained a loan of P1,000,000
from Ferdie, payable within one year. To
secure payment, Lito executed a chattel
mortgage on a Toyota Avanza and a real
estate mortgage on a 200-square meter
piece of property.
(B) Lito s failure to pay led to the extrajudicial foreclosure of the mortgaged
real
property. Within a year from foreclosure,
Lito tendered a manager s check to Ferdie
to redeem the property. Ferdie refused to
accept payment on the ground that he
wanted payment in cash: the check does
not qualify as legal tender and does not
include the interest payment. Is Ferdie s
refusal justified? (4%)
SUGGESTED ANSWER:
of the creditor they have been impaired
(Art 1249, Civil Code).
However, it is not necessary that the
right of redemption be exercised by
delivery of legal tender. A check may be
used
for
the
exercise
of
right
of
redemption, the same being a right and
not an obligation. The tender of a check
is sufficient to compel redemption but is
not in itself a payment that relieves the
redemptioner from his liability to pay
the redemption price (Biana v. Gimenez,

G.R. No. 132768, Sept 9, 2005, citing


Fortunado v. CA).
Redemption within the period allowed by
law is not a matter of intent but a
question of payment or valid tender of
full redemption prices within the said
period. Whether redemption is being
made under Art. 3135 or under the
General Banking Law, the mortgagor or
A check, whether a managers check or
his
assignee is required to tender
an ordinary check is not legal tender,
payment to make said redemption valid
and an offer of a check in payment of a
(Heirs of Quisumbing v. PNB and SLDC,
debt is not a valid tender of payment
G.R. No. 178242, Jan 20, 2009).
and may be refused receipt by the oblige
Never Let The Odds Keep You From Pursuing What You Know In Your Heart You Were Me
ant To Do.-Leroy Satchel Paige
Page 94 of 180

Civil Law Q&As (2007-2013)


hectorchristopher@yahoo.com
dbaratbateladot@gmail.com
Moreover, Ferdies refusal was justified
there has been extraordinary deflation since
on the ground that the amount tendered
1998, and therefore, Felipe should pay him
does not include interest. In order to
the value of the debt at the time it was
effect the redemption of the foreclosed
incurred. Felipe refused to pay him again,
property, the payment to the purchaser
claiming that Gustavo is estopped from
must include the following sums: (a) the
raising the issue of legal tender, having
bid price; (b) the interest on the bid
accepted the check in March, and that it
price, computed at one per centum (1%)
was Gustavo s negligence in not depositing
per month; and (c) the assessments and
the check immediately that caused the
taxes, if any, paid by the purchaser with
check to become stale.
the same rate of interest (Sec 28, 1997
Rules of Civil Procedure). Unless there is
(A). Can Gustavo now raised the issue that
an express stipulation to that effect, the
the cashier s check is not legal tender? (2%)
creditor cannot be compelled to receive
partial payment of the prestation (Art.
1248, Civil Code).
SUGGESTED ANSWER:

No. Gustavo previously accepted a check


as payment. It was his fault why the
check became stale. He is now estopped
Extinguishment;
Payment
of
Check;
check is not legal tender.
Legal Tender (2008)
No.
XVII.
Felipe
borrowed
$100
from
Gustavo in 1998, when the Phil P - US$
exchange rate was P56 - US$1. On March
1, 2008, Felipe tendered to Gustavo a
cashier s check in the amount of P4,135 in
payment of his US$ 100 debt, based on the
Phil P - US$ exchange rat at that time.
Gustavo accepted the check, but forgot to
deposit it until Sept. 12, 2008. His bank
refused to accepted the check because it
had become stale. Gustavo now wants
Felipe to pay him in cash the amount of
P5,600.
Claiming
from raising the issue that a cashier s
that
the
previous
payment was not in legal tender, and that
(B).
Can
Felipe
validly

refuse
to
pay
Gustavo again? (2%)
SUGGESTED ANSWER:
Yes, Felipe can refuse to pay Gustavo,
who allowed the check to become stale.
Although a check is not legal tender
(Belisario v. Natividad. 60 Phil 156),
there
are
instances
when
a
check
produces
example:
estoppel
he would

the effects of payment, for


(a) when the creditor is in
or he had previously promised
accept a check (Paras, Civil

Never Let The Odds Keep You From Pursuing What You Know In Your Heart You Were Me
ant To Do.-Leroy Satchel Paige
Page 95 of 180

Civil Law Q&As (2007-2013)


hectorchristopher@yahoo.com
dbaratbateladot@gmail.com
Code Annotated, Vol IV, 2000 ed., p.
secure the PN, Recardo executed a real
394); (b) when the check has lost its
estate mortgage on his own property. When
value because of the fault of the creditor
Eduardo defaulted on the PN, XYZ stopped
(Art. 1249, 2nd par.),as when he was
payment of rentals on the building on the
unreasonably delayed in presenting the
ground that legal compensation had set in.
check for payment (PNB v. Seeto, G.R.
Since there was still a balance due on the
No, L-4388, 13 August 1952).
PN
after
applying
the
rentals,
XYZ
foreclosed the real estate mortgage over
(C). Can Felipe compel Gustavo to receive
Recardo s property. Recardo opposed the
US$100 instead? (1%)
foreclosure on the ground that he is only a
co-signatory; that no demand was made
SUGGESTED ANSWER:
upon him for payment, and assuming he is
Felipe cannot compel Gustavo to receive
US$100 because under RA 529, payment

of loans should be at Philippine currency


at the rate of exchange prevailing at the
time of the stipulated date of payment.
Felipe could only compel Gustavo to
receive US$ 100 if they stipulated that
obligation be paid in foreign currency
(R.A. 4100).
liable, his liability should not go beyond
half the balance of the loan. Further,
Recardo said that when the bank invoked
compensation between the reantals and the
amount of the loan, it amounted to a new
contract or novation, and had the effect of
extinguishing the security since he did not
give his consent (as owner of the property
under the real estate mortgage) thereto.
(B). Can Recardo s property be foreclosed to
pay the full balance of the loan? (2%)
Liability; Solidary Liability (2008)
SUGGESTED ANSWER:
No. XV. Eduardo was granted a loan by
XYZ Bank for the purpose of improving a
building
which
XYZ
leased
from
him.
Eduardo, executed the promissory note
("PN") in favor of the bank, with his friend
Recardo as co-signatory. In the PN, they
both
acknowledged
that
they
are
"individually and collectively" liable and
waived the need for prior demand. To
Yes,
Recardo s
property

can
be
foreclosed to pay the full balance of the
loan because when he signed as cosignatory in the promissory note, he
acknowledged he is solidarily liable with
Eduardo.
In
solidary
obligations,
a
creditor has the right to demand full
payment of the obligation from any of
Never Let The Odds Keep You From Pursuing What You Know In Your Heart You Were Me
ant To Do.-Leroy Satchel Paige
Page 96 of 180

Civil Law Q&As (2007-2013)


hectorchristopher@yahoo.com
dbaratbateladot@gmail.com
the solidary debtors (Art. 1207, Civil
latter, is obliged to continue the same
Code).
until the termination of the affair and its
situations
giving
five
rise
to
examples
of
this
of
type
to
require
concerned
to
substitute
the
person
him,
if
the
2144, NCC).
No.V. What are obligations without an
Give
or

owner is in a position to do so (Art.


Obligations; Without Agreement (2007)
agreement"?
incidents,
obligations? (10%)
Second
example,
indebiti
may
also
a
case
give
of
solutio
rise
to
an
obligation without an agreement. This
refers to the obligation to return which
arises when something is received when
SUGGESTED ANSWER:
there is no right to demand it, and it was
"Obligations without an agreement" are
unduly delivered through mistake (Art.
obligations
2154, NCC).
that
do
not
arise
from

contract such as those arising from: 1.


delicts;
2.
quasi-delicts;
3.
solutio
indebiti; 4. negotiorum gestio; and 5. all
other obligations arising from law.
Third example, is when without the
knowledge of the person obliged to give
support, it is given by a stranger, the
latter shall have a right to claim the
same from the former, unless it appears
ALTERNATIVE ANSWER:
that he gave it out of piety and without
"Obligations without an agreement" refer
intention of being repaid (Art. 2164,
to the juridical relation of quasi-contract
NCC).
which
arise
from
certain
lawful,
voluntary and unilateral acts to the end
Fourth
example,
is
when
through
that no one shall be unjustly enriched or
accident or other causes a person is
benefited at the expense of another. (Art.

injured or becomes seriously ill, and he


2142, NCC)
is treated or helped while he is not in a
condition to give consent to a contract,
First Example of an obligation without
he shall be liable to pay for the services
an agreement is a case of negotiorum
of the physician or other person aiding
gestio,
whereby
him,
takes
charge
one
of
who
the
voluntarily
agency
or
management of the business or property
unless
the
service
has
been
rendered out of pure generosity (Art.
2167, NCC).
of another without any power from the
Never Let The Odds Keep You From Pursuing What You Know In Your Heart You Were Me
ant To Do.-Leroy Satchel Paige
Page 97 of 180

Civil Law Q&As (2007-2013)


hectorchristopher@yahoo.com
not,
dbaratbateladot@gmail.com
Fifth instance of an obligation without
ought
in
equity
and
good
an agreement is when the person obliged
conscience, to hold (Heirs of Lorenzo
to support an orphan or an insane or
Yap v. CA, 371 Phil 523, 1991). The
other indigent person unjustly refuses to
following are examples of constructive
give support to the latter, any third
trust: 1. Art. 1456 NCC which provides:
person may furnish support to the needy
"If property is acquired through mistake
individual, with right of reimbursement
or fraud, the person obtaining it is, by
from the person obliged to give support.
force of law considered a trustee of an
The provisions of this article apply when
implied trust for the benefit of the
the father or mother of a child under
person for whom the property comes." 2.
eighteen years of age unjustly refuses to
Art 1451 NCC which provides: "When

support him (Art. 2166, NCC).


land passes by succession through any
person and he causes the legal title to be
Trust
put in the name of another, a trust is
established by implication of law for the
Trust De Son Tort (2007)
benefit of the true owner." 3. Art 1454
NCC which provides: "If an absolute
No.III. Explain the following concepts and
conveyance of property is made in order
doctrines and give an example of each:
to
(A).
concept
of
trust de
son
tort (constructive trust) (5%)
secure
the
performance
of
an
obligation of the grantor toward the
grantee, a trust by virtue of law is
established. If the fulfillment of the
obligation is offered by the grantor when
SUGGESTED ANSWER:
it becomes due, he may demand the
A constructive trust is a trust NOT
reconveyance of the property to him." 4.
created by any word or phrase, either

Art 1455 NCC which provides: "When any


expressly or impliedly, evincing a direct
trustee, guardian or any person holding a
intention to create a trust, but is one
fiduciary relationship uses trust funds
that
the
for the purchase of property and causes
demands of justice. It does not come
conveyance to be made to him or to
about by agreement or intention but
third person, a trust is established by
mainly operation of law and construed as
operation of law in favor of the person to
a trust against one who, by fraud, duress
whom the funds belong."
arises
in
order
to
satisfy
or abuse of confidence, obtains or holds
the legal right to property which he
Never Let The Odds Keep You From Pursuing What You Know In Your Heart You Were Me
ant To Do.-Leroy Satchel Paige
Page 98 of 180

Civil Law Q&As (2007-2013)


hectorchristopher@yahoo.com
Sales
advanced
for
dbaratbateladot@gmail.com
her
employees (biyaheros).
She required them to surrender TCT of
Condominium
Act;
Partition
of
a
Condominium (2009)
No.XVIII.
The
Ifugao
Arms
is
a
strong earthquake occurred which left huge
cracks in the outer walls of the building. As
a result, a number of condominium units
were rendered unfit for use. May Edwin,
owner of one of the condominium units
affected, legally sue for partition by sale of
the whole project? Explain. (4%)
Yes, Edwin may legally sue for partition
of
the
whole
condominium
project under the following conditions:
(a) the damage or destruction caused by
the earthquake has rendered one-half

(1/2)
or
more
of
the
untenantable,
and
condominium
owners
(b)
units
therein
that
holding
the
an
aggregate of more than thirty percent
(30%) interests of the common areas are
opposed
to
the
and
to
execute
the
Domeng Bandong was not required to post
any security but when Eulalia discovered
that
he
incurred
shortage
in
cattle

procurement operation, he was required to


execute a Deed of Sale over a parcel of land
in favor of Eulalia. She sold the property to
her
grandneice
Jocelyn
who
thereafter
instituted an action for ejectment against
the Spouses Bandong.
To assert their right, Spouses Bandong filed
SUGGESTED ANSWER:
sale
properties
corresponding Deeds of Sale in her favor.
condominium project in Baguio City. A
by
their
restoration
of
the
an action for annulment of sale against
Eulalia and Jocelyn alleging that there was
no
sale
intended
but
only
equitable
mortgage for the purpose of securing the
shortage
incurred
by
Domeng
in

the
amount of P 70, 000.00 while employed as
"biyahero" by Eulalia. Was the Deed of Sale
between Domeng and Eulalia a contract of
sale or an equitable mortgage? Explain.
(5%)
SUGGESTED ANSWER:
condominium project (Sec 8 [b], Republic
Act No. 472 Condominium Act).
The contract between Domeng Bandong
and Eulalia was an equitable mortgage
rather than a contract of sale. The
Mortgage; Equitable Mortgage (2012)
purported deed of sale was actually
intended to merely secure the payment
No.VI. (b) Eulalia was engaged in the
of the shortage incurred by Domeng in
business of buying and selling large cattle.
the
In order to secure the financial capital, she
operations. Under Art 1602, Civil Code,
conduct
of
the
cattle-buying
Never Let The Odds Keep You From Pursuing What You Know In Your Heart You Were Me
ant To Do.-Leroy Satchel Paige
Page 99 of 180

Civil Law Q&As (2007-2013)


hectorchristopher@yahoo.com
dbaratbateladot@gmail.com
the contract shall be presumed to be an
offering P800,000 in ready cash for the
equitable mortgage when it may be fairly
land. When Roberto confirmed that he
inferred that the real intention of the
could pay in cash as soon as Sergio could
parties is simply to secure the payment
get
of a debt or the performance of any
decided to withdraw his offer to Marcelo,
other obligation. The present transaction
hoping to just explain matters to his friend.
was clearly intended to just secure the
Marcelo,
shortage incurred by Eulalia because
withdrawal was communicated to him,
Bandung remained in possession of the
taking the position that they have a firm
property inspite of the execution of the
and binding agreement that Sergio cannot
sale.
simply walk away from because he has an
the
documentation
however,
ready,
objected

Sergio
when
the
option to buy that is duly supported by a
duly accepted valuable consideration.
Option Contract; Liquor & Pulutan as
Consideration (2013)
against Sergio? (5%)
No.III.Sergio is the registered owner of a
500-square meter land. His friend, Marcelo,
who
has
long
been
(A) Does Marcelo have a cause of action
interested
in
the
property, succeeded in persuading Sergio to
SUGGESTED ANSWER:
Yes. Marcelo has a cause of action
against Sergio.
sell it to him. On June 2, 2012, they agreed
on the purchase price of P600,000 and that
Under Art. 1324, when the offerer has
Sergio would give Marcelo up to June30,
allowed the offeree a certain period to
2012 within which to raise the amount.
accept, the offer may be withdrawn at
Marcelo, in a light tone usual between
any
them, said that they should seal their
communicating such withdrawal, except
agreement through a case of Jack Daniels

when
Black and P5,000 "pulutan" money which
consideration,
he immediately handed to Sergio and which
promised.
time
the
before
option
as
acceptance
is
founded
something
by
upon
paid
or
the latter accepted. The friends then sat
down and drank the first bottle from the
An accepted unilateral promise to buy or
case of bourbon.
sell a determinate thing for a
certain
is
binding
upon
him
price
if
the
On June 15, 2013, Sergio learned of
promise is supported by a consideration
another

distinct
buyer,
Roberto,
who
was
from
the
price
(Art.
1479).
Never Let The Odds Keep You From Pursuing What You Know In Your Heart You Were Me
ant To Do.-Leroy Satchel Paige
Page 100 of 180

Civil Law Q&As (2007-2013)


hectorchristopher@yahoo.com
dbaratbateladot@gmail.com
Consideration in an option contract may
The
Statute
be anything of value,, unlike in sale
agreement for the sale of real property
where it must be the price certain in
or
money or its equivalent (San Miguel
agreement is unenforceable by action,
Properties Inc. v. Spouses Huang, G.R.
unless
No. 137290, July 31, 2000).
memorandum, thereof, be in writing,
of
of
an
the
Frauds
interest
same,
covers
therein.
or
some
an
Such
note
or
(Art. 1403 (e), Civil Code). Here, Marcelo

Here, the case of Jack Daniels Black and


and
the P5,000.00 pulutan money was a
Option
consideration to seal their agreement,
unilateral promise to buy or sell, which
an agreement that Marcelo is given until
need not be in writing to be enforceable
June 30, 2012 to buy the parcel of land.
(Sanchez v. Rigos, G.R. No. L-25494,
There is also no showing that such
June 14, 1972, citing Atkins, Kroll and
consideration will be considered part of
Co.
the
Southwestern Sugar & Molasses Co. v.
purchase
unilateral
price.
withdrawal
Thus,
of
Sergios
the
offer
Sergio
merely
Contract,
Inc.
v.
entered

which
Cua
into
refers
Hian
an
to
Tek
a
and
Atlantic Gulf & Pacific Co.).
violated the Option Contract between
him and Marcelo.
ALTERNATIVE ANSWER:
(B) Can Sergio claim that whatever they
No. Sergios claim has no legal basis.
might have agreed upon cannot be enforced
because any agreement relating to the sale
The contract of sale has already been
of real property must be supported by
partially executed which takes it outside
evidence in writing and they never reduced
the ambit of the Statute of Frauds is
their agreement to writing? (3%)
applicable only to executory contracts,
not to contracts that are totally or
SUGGESTED ANSWER:
partially performed (Carbonnel v. Poncio,
G.R. No. L-11231, May 12, 1958).
No. Sergios claim has no legal basis.
The contract at issue in the present case
is the option contract, not the contract
Right of First Refusal; Lessee; Effect

of sale for the real property. Therefore,


(2008)
Art. 1403 does not apply.
No.XVI. Dux leased his house to Iris for a
period of 2 years, at the rate of P25,000.00
Never Let The Odds Keep You From Pursuing What You Know In Your Heart You Were Me
ant To Do.-Leroy Satchel Paige
Page 101 of 180

Civil Law Q&As (2007-2013)


hectorchristopher@yahoo.com
dbaratbateladot@gmail.com
monthly, payable annually in advance. The
refusal. This makes the mother a buyer
contract stipulated that it may be renewed
in bad faith, hence giving more ground
for another 2-year period upon mutual
for
agreement of the parties. The contract also
(Equatorial
granted Iris the right of first refusal to
Theater, G.R. No. 106063, 21 Nov. 1996).
rescission
of
Realty,
the
et
sale
al.
v.
to
her
Mayfair
purchase the property at any time during
the lease, if Dux decides to sell the property
at the same price that the property is
offered for sale to a third party. Twentythree months after execution of the lea
se
contract, Dux sold breach of her right of
first refusal. Dux said there was no breach
because the property was sold to his
mother who is not a third party. Iris filed an
action to rescind the sale and to compel
Dux to sell the property to her at the same
price. Alternatively, she asked the court to
ALTERNATIVE ANSWER:

No, Iris cannot seek rescission of the


sale of the property to Duxs mother
because the sale is not one of those
rescissible contracts under Art. 1381 of
the Civil Code.
(B). Will the alternative prayer for extension
of the lease prosper? (2%)
SUGGESTED ANSWER:
extend the lease for another 2 years on the
No. The contract stipulated that it may
same terms.
be renewed for another 2-year period
(A). Can Iris seek rescission of the sale of
upon mutual agreement of the parties.
the property to Dux s mother? (3%)
Contracts
are
binding
between
the
parties; validity or compliance cannot be
SUGGESTED ANSWER:
left to the will of one of the parties (Art.
Yes, because the right of first refusal is
included in the contract signed by the
parties. Only if the lessee failed to
1308, Civil Code).
ALTERNATIVE ANSWER:
exercise the right of first refusal could
It depends. The alternative prayer for
the
subject
the extension of the lease may prosper if
property to others, under no less than
(a) there is a stipulation in the contract
the

conditions
of sale; (b) Dux s mother is aware of the
previously offered to the lessee. Granting
existing contract of lease; or (c) the lease
that the mother is not a third party, this
is recorded in the Registry of Property
would make her privy to the agreement
(Art. 1676, Civil Code).
lessor
same
lawfully
terms
sell
and
the
of Dux and Iris, aware of the right of first
Never Let The Odds Keep You From Pursuing What You Know In Your Heart You Were Me
ant To Do.-Leroy Satchel Paige
Page 102 of 180

Civil Law Q&As (2007-2013)


hectorchristopher@yahoo.com
Lease
dbaratbateladot@gmail.com
reimbursed the value of the improvements
he introduced. (4%)
Builder;
Good
Faith;
Useful
Improvements (2013)
No.IV.Anselmo is the registered owner of a
land and a house that his friend Boboy
occupied for a nominal rental and on the
condition that Boboy would vacate the
property
on
demand.
With
Anselmo s
knowledge, Boboy introduced renovations
consisting of an additional bedroom, a
covered veranda, and a concrete block
fence, at his own expense.
SUGGESTED ANSWER:
Boboys claim that he is a builder in
good faith has no basis. A builder in good
faith
is
someone
who
occupies
the
property in concept of an owner. The
provisions
on
builder-planter-sower

under the Civil Code cover cases in


which the builder, planter and sower
believe themselves to be owners of the
land, or at least, to have a claim of title
thereto.
Subsequently, Anselmo needed the property
as his residence and thus asked Boboy to
vacate and turn it over to him. Boboy,
despite an extension, failed to vacate the
property, forcing Anselmo to send him a
written demand to vacate.
As Boboy is a lessee of the property,
even if he was paying nominal rental,
Art. 1678, Civil Code, is applicable.
Under
this
makes,
provision,
in
good
if
the
lessee
faith,
useful
improvements which are suitable to the
In his own written reply, Boboy signified
use for which the lease is intended,
that he was ready to leave but Anselmo
without altering the form or substance of
must first reimburse him the value of the
the property leased, the lessor upon the
improvements
termination of the lease, shall pay the
he
introduced

on
the
property as he is a builder in good faith.
lessee
Anselmo
Boboy
improvements at that time. Should the
cannot ask for reimbursement as he is a
lessor refuse to reimburse said amount,
mere lessee. Boboy responded by removing
the
the improvements and leaving the building
improvements,
even
in its original state.
principal
may
refused,
insisting
that
one-half
lessee
thing
of
may
the
value
of
remove
the

though
the
suffer
damage
thereby.
(IVa) Resolve Boboy s claim that as a
builder
in
good
faith,
he
should
be
Never Let The Odds Keep You From Pursuing What You Know In Your Heart You Were Me
ant To Do.-Leroy Satchel Paige
Page 103 of 180

Civil Law Q&As (2007-2013)


hectorchristopher@yahoo.com
dbaratbateladot@gmail.com
(IVb) Can Boboy be held liable for damages
lease contracts between Jude and his
for
tenants? Explain your answer. (3%)
removing
the
improvements
over
Anselmo s objection? (4%)
SUGGESTED ANSWER:
SUGGESTED ANSWER:
Yes, Ildefonso must respect the lease
contracts between Jude and his tenants.
No. Boboy cannot be held liable for
While it is true that the said lease
damages.
contracts
The
lessor,
Anselmo,
refused
to
reimburse one-half of the value of the
improvements, so the lessee, Boboy, may
remove
principal
thereby.
the
same,
thing
If

in
improvements
even
may
suffer
removing
Boboy
though
the
damage
the
useful
caused
more
impairment in the property leased than
is necessary he will be liable for damages
(Art. 1678, Civil Code).
were
not
registered
and
annotated on the title to the property,
Ildefonso
is
still
not
an
innocent
purchaser for value. He ought to know
the existence of the lease because the
building was already occupied by the
tenants
at
the

time
he
bought
it.
Applying the principle of caveat emptor,
he should have checked and known the
status of the occupants of their right to
occupy the building before buying it.
Agency
Lease; Caveat Emptor (2009)
Agency; Sale of a Real Property through
No.VIII. Jude owned a building which he
had leased to several tenants. Without
informing
his
tenants,
Jude
sold
the
building to Ildefonso. Thereafter, the latter
notified all the tenants that he is the new
owner of the building. Ildefonso ordered the
tenants to vacate the premises within thirty
an Agent (2010)
No.XVI. X was the owner of an unregistered
parcel of land in Cabanatuan City. As she
was abroad, she advised her sister Y via
overseas call to sell the land and sign a
contract of sale on her behalf.
(30) days from notice because he had other
Y thus sold the land to B1 on March 31,
plans for the building. The tenants refused
2001 and executed a deed of absolute sale
to vacate, insisting that they will only do so
on behalf of X. B1 fully paid the purchase
when the term of their lease shall have
price.

expired. Is Ildefonso bound to respect the


Never Let The Odds Keep You From Pursuing What You Know In Your Heart You Were Me
ant To Do.-Leroy Satchel Paige
Page 104 of 180

Civil Law Q&As (2007-2013)


hectorchristopher@yahoo.com
dbaratbateladot@gmail.com
B2, unaware of the sale of the land to B1,
double sales of an immovable property,
signified to Y his interest to buy it but
the
asked Y for her authority from X. Without
person who is in good faith was first in
informing X that she had sold the land to
possession and in the absence thereof to
B1, Y sought X for a written authority to
the person who presents the oldest title,
sell.
provide there is good faith.
X e-mailed Y an authority to sell the land. Y
In a case, the Supreme Court has held
thereafter sold the land on May 1, 2001 to
that in a sale of real estate the execution
B2 on monthly installment basis for two
of
years, the first installment to be paid at the
tantamount to delivery of the possession
end of May 2001.
of the property sold. The ownership of
ownership
a
shall
notarial
pertain

document
to
of
the
sale
is
the land therefore pertains to the first
Who between B1 and B2 has a better right
buyer. It may also be mentioned that
over the land? Explain. (5%)
under Art 3344 no instruments or deed
establishing,
SUGGESTED ANSWER:
B-2 has a better title. This is not a case
of double sale. Since the first sale was
void. The law provides that when a sale
of a piece of land or any interest therein
is through an agent, the authority of the
acknowledging,
Hence, the sale to B1 was void.
ALTERNATIVE ANSWER:
or
registered under Act 496 shall be valid
except as between the parties. Thus, the
Deed of Sale of B-2 has no binding effect
on B-1.
Partnership
sale shall be void (Art 1874, NCC). The
written authority from the owner X.
modifying,
extinguishing right to real property not
latter shall be in writing; otherwise, the
property was sold by Y to B1 wihtout any
transmitting,
Liability; Liability of a Partner (2010)
No.XV.
A,

partnership
B,
and
to
C
operate
entered
a
into
a
restaurant
business. When the restaurant had gone
Under the facts, B-1 has a better right to
past break-even stage and started to garner
the land. Given the fact that the Deed of
considerable profits, C died. A and B
Sale in favor of B-1 and B-2 are not
continued the business without dissolving
inscribed in the Registry of Deeds, the
the partnership. They in fact opened a
case is governed by Art 1544 of the New
branch
Civil Code which provides that in case of
obligations in the process. Creditors started
of
the
restaurant,
incurring
Never Let The Odds Keep You From Pursuing What You Know In Your Heart You Were Me
ant To Do.-Leroy Satchel Paige
Page 105 of 180

Civil Law Q&As (2007-2013)


demanding
for
hectorchristopher@yahoo.com
the
payment
of
their
obligations.
dbaratbateladot@gmail.com
individual properties shall be subject
first to the payment of his separate
debts (Art 1835. NCC).
(A). Who are liable for the settlement of the
partnerships obligations? Explain? (3%)
SUGGESTED ANSWER:
Oral Partnership (2009)
The two remaining partners, A and B, are
No.I. TRUE or FALSE. Answer TRUE if the
liable. When any partner dies and the
statement
business
statement is false. Explain your answer in
is
continued
without
any
settlement of accounts as between him
is
true,
or
FALSE

if
the
not more than two (2) sentences.
or his estate, the surviving partners are
held liable for continuing the business
(C). An oral partnership is valid. (1%)
despite the death of C (Art 1841, 1785,
SUGGESTED ANSWER:
par 2, and Art 1833 of NCC).
TRUE.
Partnership
is
a
consensual
(B).What are the creditors recourse/s?
contract, hence, it is valid even though
Explain. (3%)
not in writing.
SUGGESTED ANSWER:
ALTERNATIVE ANSWER:
Creditors
actions,
can
for
file
the
instance,
an
appropriate
action
for
collection of sum of money against the
partnership at will and if there are no
sufficient funds, the creditors may go

after the private properties of A and B


(Art 816, NCC). Creditors may also sue
the estate of C. The estate is not
excused
from
the
liabilities
of
the
partnership even if C is dead already but
only up to the time that he remained a
partner (Art 1829, 1835, par 2, NCC;
Testate Estate of Mota v. Serra, 47 Phil
464 [1925]). However, the liability of Cs
TRUE. An oral is a consensual of the
partnership is valid even though not in
writing.
However,
If
it
involves
contribution of an immovable property
or a real right, an oral contract of
partnership is void. In such a case, the
contract of partnership to be valid, must
be in a public instrument ( Art. 1771
,NCC
),
and
the
inventory
of
said
property signed by the parties must be
attached to said public instrument (Art.
1773, NCC).
ALTERNATIVE ANSWER:
Never Let The Odds Keep You From Pursuing What You Know In Your Heart You Were Me
ant To Do.-Leroy Satchel Paige

Page 106 of 180

Civil Law Q&As (2007-2013)


TRUE.
hectorchristopher@yahoo.com
consensual
Yes, he is not entitled to the return of
contract, hence, it is valid even though
his contribution to the capital of the
not in writing. The oral contract of
partnership, but only to the net profits
partnership is also valid even if an
from the partnership business during the
immovable
is
life of the partnership period. If he is a
contributed thereto. While the law, in
limited partner, however, he may ask for
such a case, requires the partnership to
the
be in a public document, the law does
provided in Art 1856 and 1857, Civil
not expressly declare the contract void if
Code.
not
Partnership
is
property
executed
in
a
dbaratbateladot@gmail.com

or real
the
right
required
form
inferred that the said requirement is
prohibitory
NCC),
the
or
mandatory
said
oral
(Article
contract
of
his
contributions
as
Commodatum & Mutuum
(Article 1409 (7 ,NCC ). And there being
nothing in the law from which it can be
return
Mutuum;
Interest;
Solutio
Indebiti
(2012)
5,
of
No.VI.a)
Siga-an

granted
a
loan
to
partnership must also be valid. The
Villanueva in the amount of P 540, 000.00.
interested party may simply require the
Such agreement was not reduced to writing.
contract
public
Siga-an demanded interest which was paid
document in order to comply with the
by Villanueva in cash and checks. The total
required form (Article 1357, NCC). The
amount
purpose of the law in requiring a public
to P 1, 200, 000.00. Upon advice of her
document is simply to notify the public
lawyer, Villanueva demanded for the return
about the contribution.
of the excess amount of P 660, 000.00
to
be
made
into
a
Villanueva
paid
accumulated
which was ignored by Siga-an.
Share; Demand during the Existence of

(1)
Partnership (2012)
Explain. (3%)
No.X.b) A partner cannot demand the
SUGGESTED ANSWER:
Is
the
payment
of
interest
valid?
return of his share (contribution) during the
existence of a partnership. Do you agree?
No, Art. 1956, Civil Code, provides that
Explain your answer. (5%)
no interest shall be due unless it has
been expressly stipulated in writing.
SUGGESTED ANSWER:
(2) Is solution indebiti applicable? Explain.
(2%)
Never Let The Odds Keep You From Pursuing What You Know In Your Heart You Were Me
ant To Do.-Leroy Satchel Paige
Page 107 of 180

Civil Law Q&As (2007-2013)


hectorchristopher@yahoo.com
enforceable ( Article 1403 [2] b, NCC).The
SUGGESTED ANSWER:
validity
Yes,
Solutio
because
Indebiti
Villanueva
P600,000.00
dbaratbateladot@gmail.com
is
applicable
Overpaid
representing
of
the
be
Surety
interest
therefore, demand its return.
should
distinguished from its enforceability .
by
payment which is not due. He can,
contract
Surety (2010)
No.III. Define, Enumerate or Explain. (2%
Guaranty
each)

Guaranty (2009)
(A).
What
is
the
difference
between
"guaranty" and "suretyship"?
No.I. TRUE or FALSE. Answer TRUE if the
statement
is
true,
or
FALSE
if
the
statement is false. Explain your answer in
not more than two (2) sentences.
SUGGESTED ANSWER:
Guaranty and Suretyship distinguished
(1)The
(D). An oral promise of guaranty is valid
and binding. (1%)
obligation
in
guaranty
is
secondary; whereas, in suretyship, it is
primary.
SUGGESTED ANSWER :
(2) In guranty, the undertaking is to pay
if
the
principal

debtor
cannot
pay;
FALSE. An oral contract of guaranty,
whereas, in suretyship, the undertaking
being a special promise to answer for the
is to pay if the principal debtor does not
debt of
pay .
another,
is
unenforceable
unless
in
(3) In guranty, the guarantor is entitled
writing (Article 1403 [2] b, NCC ).
to the benefit of excussion; whereas, in
suretyship the surety
ALTERNATIVE ANSWER:
is not entitled.
TRUE. An oral promise of guaranty is
(4) Liability in guaranty depends upon
valid and binding. While the contract is
an independent agreement to pay the
valid,
unenforceable
obligations of the principal if he fails to
because it is not writing . Being a special
do so; whereas, in suretyship, the surety
promise
assumes liability as a regular party.

however
answer
,it
for
is
the
debt,
or
miscarriage of another, the Statute of
Frauds requires it to be in writing to be
Never Let The Odds Keep You From Pursuing What You Know In Your Heart You Were Me
ant To Do.-Leroy Satchel Paige
Page 108 of 180

Civil Law Q&As (2007-2013)


hectorchristopher@yahoo.com
dbaratbateladot@gmail.com
(5)The Guarantor insures the solvency
(B). Will your answer to [a] be the same if
of the principal debtor; whereas, the
the contract stipulates that upon failure of
surety insures the debt.
Rosario to redeem the ring on due date,
(6)In
a
guaranty,
subsidiarlty
suretyship,
liable;
the
the
guarantor
whereas,
surety
in
binds
is
a
himself
solidarity with the principal debtor (Art
2047, Civil Code).
herself
as
full
payment
of

the
loan?
Reasons. (3%)
No, my answer will be different. While
the contract of pledge is valid, the
stipulation authorizing the pledgee to
Pledge; Pactum Commissorium (2009)
Rosario
appropriate the entire proceeds thereof for
SUGGESTED ANSWER:
Pledge
No.XVII.
Jennifer may immediately sell the ring and
obtained
a
loan
immediately sell the thing pledged is
of
P100,000.00 from Jennifer, and pledged
her diamond ring. The contract signed by
the parties stipulated that if Rosario is
unable to redeem the ring on due date, she
will execute a document in favor of Jennifer
providing that the ring shall automatically
void under Art 2088 of the New Civil
Code, which provides that the creditor
cannot appropriate the things given by
way of pledge or mortgage, or dispose of
them xxx. Jennifer cannot immediately
sell by herself the thing pledged. It must
be foreclosed by selling it at a public
auction
be considered full payment of the loan.
in
accordance
with
the
procedure under Art 2112 of the New
Civil Code.

(A). Is the contract valid? Explain. (3%)


Torts and Damages
SUGGESTED ANSWER:
The contract is valid because Rosario
Damages (2012)
has to execute a document in favor of
Jennifer to transfer the ownership of the
No.I. a) Roberto was in Nikko Hotel when he
pledged ring to the latter. The contract
bumped into a friend who was then on her
does
way to a wedding reception being held in
not
commissorium
amount
because
to
it
pactum
does
not
said hotel. Roberto alleged that he was then
provide for the automatic appropriation
invited by his friend to join her at the
by the pledgee of the thing pledged in
wedding reception and carried the basket
case of default by the pledgor.
full of fruits which she was bringing to the
affair.
At
the
reception,
the

wedding
Never Let The Odds Keep You From Pursuing What You Know In Your Heart You Were Me
ant To Do.-Leroy Satchel Paige
Page 109 of 180

Civil Law Q&As (2007-2013)


hectorchristopher@yahoo.com
dbaratbateladot@gmail.com
coordinator of the hotel noticed him and
prosper. Otherwise, Robertos action will
asked him, allegedly in a loud voice, to
not prosper.
leave as he was not in the guest list. He
retorted that he had been invited to the
affair by his friend, who however denied
doing
so.
Deeply
embarrassed
by
the
The hotel is liable for the wrongful acts
of its employees.
COMMENT:
incident, Roberto then sued the hotel for
damages under Articles 19 and 21 of the
Civil Code. Will Robertos action prosper?
The facts of the problem are almost
similar to the facts of Nikko Hotel
Manila
Explain. (5%)
Garden
v.
Reyes,
G.R.
No.
154259, Feb 28, 2005. In the said case,
however, there is a categorical finding
SUGGESTED ANSWER:
that the hotel employee did not, exposed
No. Robertos action will not prosper.

From the facts given in the problem, the


wedding coordinator did not abuse her
right when she asked him to leave the
wedding reception because he was not in
the guest list. Hotel Nikko could not be
held liable for damages as its liable
the complainant to the ridicule, shame
or
embarrassment;
hence,
did
not
commit any abuse of right. The present
problem makes no statement of that
finding. In the contrary, the problem
states that it is a mere allegation.
spring from the liability of its employee
(Nikko Hotel Manila Garden v. Reyes,
G.R. No. 154259, Feb 28, 2005).
Damages; Moral & Exemplary (2009)
ALTERNATIVE ANSWER:
No.XIV. Rodolfo, married to Sharon, had an
It depends. While the hotel has the right
to exclude an uninvited guest from the
wedding reception, that does not give
the
hotel
the
license
to
humiliate
Roberto. If the wedding coordinator of
the hotel acted wrongfully e.g. with the
abuse of right, unfairly, or in a matter
that exposed Roberto to unnecessary
ridicule
or
shame,
his

action
will
illicit affair with his secretary, Nanette, a
19-year old girl, and begot a baby girl,
Rona. Nanette sued Rodolfo for damages:
actual, for hospital and other medical
expenses
in
delivering
the
child
by
caesarean section; moral, claiming that
Rodolfo
promised
to
marry
her,
representing that he was single when, in
fact, he was not; and exemplary, to teach a
lesson to like-minded Lotharios.
Never Let The Odds Keep You From Pursuing What You Know In Your Heart You Were Me
ant To Do.-Leroy Satchel Paige
Page 110 of 180

Civil Law Q&As (2007-2013)


hectorchristopher@yahoo.com
dbaratbateladot@gmail.com
(A). If you were the judge, would you award
Vinzons-Chato filed a Motion to Dismiss
all the claims of Nanette? Explain. (3%)
arguing that she cannot be held liable for
damages for acts she performed while in
SUGGESTED ANSWER:
the
If Rodolfo s marriage could not have
Commissioner. Is she correct? Explain. (5%)
discharge
of
her
duties
as
BIR
been possibly known to Nanette or there
is no gross negligence on the part of
Nanette, Rodolfo could be held liable for
moral damages.
Yes. As a general rule, a public officer is
not liable for acts performed in the
If there is gross negligence in a suit for
quasi-delict,
SUGGESTED ANSWER:
exemplary
could
be
awarded.
discharge of his duties. The exceptions
are when he acted with malice, bad faith,
or gross negligence in the performance
of his duty, or when his act is in

violation of a Constitutional guaranteed


right and liberties of a person under
Damages; Public Officers acting in the
Performance of their Duties (2012)
Art32 of the NCC.
The public officer is not automatically
No.II.a) Liwayway Vinzons-Chato was then
considered to have violated the rights or
the
liberties of a person simply because the
Commissioner
of
Internal
Revenue
while Fortune Tobacco Corporation is an
rule
entity
declared
engaged
in
the
manufacture
of
the
public
invalid
officer
by
issued
was
court.
The
the

different brands of cigarettes, among which


complainant must still allege and prove
are
the particular injury or prejudice he has
"Champion,"
"Hope,"
and
"More"
cigarettes.
suffered
from
the
violation
of
his
constitutional right by the issuance of
Fortune filed a complaint against VinzonsChato to recover damages for the allege
d
violation of its constitutional rights arising
from Vinzons-Chatos issuance of Revenue
Memorandum Circular No. 37-934 (which
re-classified Fortune cigarettes as locally
manufactured with foreign brands and
thereby imposed higher taxes), which the
Supreme Court later declared invalid.
the invalidated rule.
The problem does not state any fact
from which any malice, bad faith or
gross negligence on the part of VinzonsChato may be inferred, or the particular
injury or prejudice the complainant may
have suffered as a result of the violation
of his constitutional right. Hence, she
Never Let The Odds Keep You From Pursuing What You Know In Your Heart You Were Me
ant To Do.-Leroy Satchel Paige
Page 111 of 180

Civil Law Q&As (2007-2013)


cannot
be
held
hectorchristopher@yahoo.com
liable.
The
facts
presented are similar to facts of the case
dbaratbateladot@gmail.com
the driver and the defense of diligence is
not available.
of Vinzons-Chato v. Fortune, G.R. No.
(B).Would your answer be the same if
141309, Dec 23, 2008.
Rommel was in the car at the time of the
accident? Explain. (2%)
Death Indemnity (2009)
SUGGESTED ANSWER:
Yes, my answer would be the same.
No. X. Rommels private car, while being
Rommel, who was in the car, shall be
driven by the regular family driver, Amado,
liable for damages if he could have
hits a pedestrian causing the latters death.
prevented the misfortune by the use of
Rommel is not in the car when the incident
due diligence in supervising his driver
happened.
but failed to exercise it (Art. 2184, NCC).
(A). Is Rommel liable for damages to the
heirs of the deceased? Explain. (2%)
Yes, my answer will be the same except

Yes, Rommel may be held liable for


damages if he fails to prove that he
exercised the diligence of a good father
of a family (Art. 2180, par 5, NCC) in
and
supervising
his
family
driver. The owner is presumed liable
unless
he
proves
his driver.
ALTERNATIVE ANSWER:
SUGGESTED ANSWER:
selecting
In such case, his liability is solidary with
the
defense
of
diligence. If the driver was performing
his assigned task when the accident
happened, Rommel shall be solidarily
that in such case the liability of the
owner is not presumed. When the owner
is inside the vehicle, he becomes liable
only when it is shown that he could have
prevented the misfortune by the use of
due diligence (Art. 2184, NCC). For the
owner to be held liable, the burden of
proving that he could have prevented
the misfortune rests on the shoulder of
the victim.
liable with the driver.
In
case
the
driver
is

convicted
of
reckless imprudence and cannot pay the
Doctrine of Discovered Peril (Last Clear
Chance) (2007)
civil liability, Rommel is subsidiarily
liable for the damage awarded against
No.III. Explain the following concepts and
doctrines and give an example of each:
Never Let The Odds Keep You From Pursuing What You Know In Your Heart You Were Me
ant To Do.-Leroy Satchel Paige
Page 112 of 180

Civil Law Q&As (2007-2013)


hectorchristopher@yahoo.com
dbaratbateladot@gmail.com
(B). doctrine of discovered peril (last clear
typhoon knocked down the fence of the
chance) (5%)
pond and the iguana crawled out of the
gate of Primos residence. N, a neighbor
SUGGESTED ANSWER:
who was passing by, started throwing
The doctrine of last clear chance states
that where the plaintiff was guilty of
prior or antecedent negligence, but the
defendant,
who
opportunity
to
had
avoid
the
the
ultimate
impending
harm failed to do so, it is the defendant
stones at the iguana, drawing the iguana to
move toward him. N panicked and ran but
tripped on something and suffered a broken
leg.
Is anyone liable for Ns injuries? Explain.
(4%)
who is liable for all the consequences of
the accident notwithstanding the prior
negligence of the plaintiff. An example is
where a person was riding a pony on a
bridge and improperly pulled the pony to
the wrong side when he saw a car
coming. The driver of the car did not
stop or change direction, and nearly hit
the horse, and, the frightened animal
jumped to its death. The driver of the
car is guilty of negligence because he
had a fair opportunity to avoid the

SUGGESTED ANSWER:
No one is liable. The possessor of an
animal or whoever may make use of the
same is responsible for the damage it
may cause, although it may escape or be
lost. This responsibility shall cease only
in case the damage should come from
force majeure or from the fault of the
person who has suffered damage (Art
2183, NCC).
accident and failed to avail himself of
that opportunity. He is liable under the
doctrine of last clear chance (Picart v.
Smith, 37 Phil. 809, 1918).
Liability;
Special
Parental
Authority
(2010)
No.XII. On May 5, 1989, 16-year old
Liability; Owner of a Pet; Fortuitous
Rozanno, who was issued a student permit,
Event (2010)
drove to school a car, a gift from his
parents. On even date, as his class was
No.XIV. Primo owns a pet iguana which he
scheduled to go on a field trip, his teacher
keeps in a man-made pond enclosed by a
requested him to accommodate in his car,
fence situated in his residential lot. A
as he did, four (4) of his classmates
Never Let The Odds Keep You From Pursuing What You Know In Your Heart You Were Me
ant To Do.-Leroy Satchel Paige
Page 113 of 180

Civil Law Q&As (2007-2013)


hectorchristopher@yahoo.com
dbaratbateladot@gmail.com
because the van rented by the school was
(B). How about the damage to the jeepney?
too crowded. On the way to a museum
Explain. (2%)
which the students were scheduled to visit,
Rozanno made a wrong maneuver, causing
a collision with a jeepney. One of his
classmates died. He and the three (3) others
were badly injured.
SUGGESTED ANSWER:
With respect to the damages caused to
the jeepney, only Rozanno should be
held liable because his negligence or
(A). Who is liable for the death of Rozannos
classmate and the injuries suffered by
Rozanno and his 3 other classmates?
Explain. (2%)
tortuous act was the sole, proximate and
immediate cause thereof.
(C). Under the same facts, except the date
of occurrence of the incident, this time in
mid-1994, what would be your answer?
SUGGESTED ANSWER:
Explain. (2%)
At the time the incident occurred in May
1989, Rozanno was still a minor. Being a
minor, Art 218 of the Family Code
applies. Pursuant to Art 218, the school,
its administrators and teachers shall be
liable for the acts of minor Rozanno
because of the special parental authority
and responsibility that they exercise
over him. The authority applies to all
authorized activities, whether inside or
SUGGESTED ANSWER:
Since Rozanno was 16 years old in 1989,
if the incident happened sometime in
the middle of 1994, Rozanno have been
21 years old at the time. Hence, he was
already of legal age. The law reducing
the age of majority to 18 years took
effect in December 1989.

the school,
Being of legal age, articles 218, 219, and
entity or institution. The field trip on
221 of the Family Code are no longer
which occasion Rozanno drove the car,
applicable. In such case, only Rozanno
was an authorized activity, and , thus,
will be personally responsible for all the
covered by the provision. Furthermore,
consequences
the parents of Rozanno are subsidiarily
school or his parents were themselves
liable pursuant to Art 219 (FC), and
also
principally liable under Art 221 (FC), if
contributed to the happening of the
they are negligent.
incident. In that event, the school or his
outside the premises
of
negligent
of
his
and
act
such
unless
his
negligence
parents are not liable under Art 218, 218
or 221 of the Family Code, but will be
Never Let The Odds Keep You From Pursuing What You Know In Your Heart You Were Me
ant To Do.-Leroy Satchel Paige

Page 114 of 180

Civil Law Q&As (2007-2013)


hectorchristopher@yahoo.com
dbaratbateladot@gmail.com
liable under general provision on the
court to justify the damages that your client
Civil Code on quasi-delict.
claims? (8%)
SUGGESTED ANSWER:
Quasi-Delict; Claims; Requisites (2013)
No.II. A collision occurred at an intersection
involving a bicycle and a taxicab. Both the
bicycle rider (a businessman then doing his
morning
exercise)
and
the
taxi
driver
claimed that the other was at fault. Based
on the police report, the bicycle crossed the
intersection first but the taxicab, crossing
at a fast clip from the bicycle s left, could
I will the base the claim of my client on
quasi-delict under Art 2176 of the Civil
Code of the Philippines. The requisites
for a claim under quasi-delict to prosper
are as follows:
(1) Act or omission, there being fault or
negligence;
(2) Damage or injury; and
not brake in time and hit the bicycle s rear
wheel, toppling it and throwing the bicycle
(3)
rider into the sidewalk 5 meters away.
damage and the act or omission.
The bicycle rider suffered a fractured right
The case clearly involves quasi-delict
knee, sustained when he fell on his right

where
side on the concrete side walk. He was
suffered
hospitalized
negligence
and
was
subsequently
operated on, rendering him immobile for 3
Causal
my
connection
client,
injury
of
as
the
the
a
between
bicycle
result
the
rider,
of
the
over-speeding
taxi
driver, without fault on my clients part.
weeks and requiring physical rehabilitation
for another 3 months. In his complaint for
To prove actual damages aside from the
damages, the rider prayed for the award

testimony of client, I will present his


ofP1,000,000
hospital and medical bills. Receipts paid
moral
actual
damages,P200,000
damages, P200,000
damages, P1
00,000
exemplary
nominal
damages
on
the
rehabilitation
will
also
be
presented. [The sentence in red should
be replaced with the following sentence
and P50,000 attorney s fees.
because he is a businessman and not an
Assuming the police report to be correct
employee. Furthermore, I will present
and as the lawyer for the bicycle rider, what
income tax returns, contracts and other
evidence
testimonial)
documents to prove unrealized profits as
and legal arguments will you present in
a result of this temporary injury.] I will

(documentary
and
Never Let The Odds Keep You From Pursuing What You Know In Your Heart You Were Me
ant To Do.-Leroy Satchel Paige
Page 115 of 180

Civil Law Q&As (2007-2013)


hectorchristopher@yahoo.com
dbaratbateladot@gmail.com
also call the attending physician to
fraudulent,
reckless,
testify as to the extent of the injuries
malevolent manner. While the amount of
suffered by my client, and to corroborate
exemplary damages may not be proved,
the contents of the medical documents.
the
plaintiff
entitled
to
must
moral
oppressive,
show
or
that
he
or
is
compensatory
Based on Art. 2202, in quasi-delicts, the
damages.
defendant shall be liable for all damages
present the police report showing the
which
probable

circumstance under which the accident


consequences of the act or omission
took place, taking into account the
complained of. It is not necessary that
actions of the parties. I will ask the
the damages have been foreseen or could
officials who responded to the accident
have been foreseen by the defendant.
to testify as to the conduct of the
Unlike
are
the
actual
natural
damages,
and
no
proof
of
pecuniary loss is necessary in order that
moral, nominal, temperate liquidated or
In
support
of
this,
I
will
parties at the time of the accident in
order to determine whether defendant
was guilty of gross negligence.
exemplary damages may be adjudicated.
Finally, attorneys fees may be recovered

The assessment is left to the discretion


when exemplary damages are awarded
of the Court (Art. 2216, Civil Code).
(Art 2208, Civil Code).
There
must
be
proof
pecuniary
estimation, however.
Moral damages can be recovered by my
Quasi Tort (2010)
client under Articles 2219 and 2200.
Moral damages may be recovered in case
No.III. Define, Enumerate or Explain. (2%
of
each)
a
quasi-delict
causing
physical
injuries. Additionally, it must be proved
that such damages were the proximate
result of the act complained of. Medical
certificates will be presented, along with
the testimony from my client and other
eyewitness accounts, in order to support
the award for moral damages.
defendant
acted
in
liable under quasi torts and what are the
defenses available to them?
Note: It is recommended that the examiner
exercise leniency and liberality in grading
the answers given to this question. The term

Exemplary damages may be granted if


the
(B). Define quasi tort. Who are the persons
wanton,
quasi-tort is not part of legal developments in
civil law. In Philippine legal tradition, quasiNever Let The Odds Keep You From Pursuing What You Know In Your Heart You Were Me
ant To Do.-Leroy Satchel Paige
Page 116 of 180

Civil Law Q&As (2007-2013)


hectorchristopher@yahoo.com
dbaratbateladot@gmail.com
delict has been treated as the closest civil
who can be held liable and their defenses
law equivalent of the common law tort. In
would also apply.
fact,
in
a
decisions,
number
the
two
of
Supreme
terms
have
Court
been
Those liable for quasi-delict include:
considered synonymous. In reality, however,
(1)
the common law tort is much broader in
causing damage to another through fault
scope than the civil law quasi-delict. In
or negligence ( Article 2176 NCC ); and
recent developments in common law, the
(2)
concept of quasi-torts can be considered as
Article 2180 (NCC ).

Those
tortfeasor
Persons
or
vicariously
the
person
liable
under
the closest common law equivalent of the
civil law concept of quasi-delict. This is
because it is argued that the growing
The defenses available include:
recognition of quasi-torts as a source of
(a) That the defendant was not negligent
obligation is hinged on the acceptance at
or that he exercised due diligence (
common law of the civil law principles of
Article 2176 NCC );
quasi-delict.
(b)
SUGGESTED ANSWER:
That
negligent
Quasi -tort is a legal concept upholding
the doctrine that some legal duty exists
although
his
the
defendant
negligence
is
not

is
the
proximate cause of the injury ( Article
2179 NCC );
that cannot be classified strictly as a
(c) That the plaintiff s own negligence
personal duty (thus resulting in a tort),
was the immediate and proximate cause
nor as a contractual duty but rather
of his injury ( Article 2179 NCC );
some other kind of duty recognizable by
the law. Tort or Quasi-tort is an
(d ) That the person vicariously liable
Law
has observed all the diligence of a good
concept, while Delict or Quasi-Delict
father of a family to prevent damage (
is
Article 2180 NCC ); and
Anglo
a
American
Civil
Law
or
Common
concept
(Wikipedia
encyclopedia).
(e)
That
the
cause

of
action
prescribed after the lapse
ALTERNATIVE ANSWER:
has
s (Article
2179 NCC ).
Quasi
-tort
is
considered
as
the
equivalent of quasi-delict. Hence the
The
fact
that
the
plaintiff
had
rules of the latter pertaining to persons
committed contributory negligence is a
partial defense (Art 2179, NCC).
Never Let The Odds Keep You From Pursuing What You Know In Your Heart You Were Me
ant To Do.-Leroy Satchel Paige
Page 117 of 180

Civil Law Q&As (2007-2013)


hectorchristopher@yahoo.com
MULTIPLE CHOICE
per
Taxation
Law
and
not
by
right
of
representation (Art 975, Civil Code)
QUESTIONS
2013
capita,
dbaratbateladot@gmail.com
Exam
MCQ (October 13, 2013)
I. (2) How much is Dante s share in the net
estate? (1%)
(A) P150,000.
(B) P200,000.
I. Armand died intestate. His full-blood
(C) P300,000.
brothers, Bobby and Conrad, and half(D) P400,000.
blood brothers, Danny, Edward and Floro,
(E) None of the above.
all predeceased him. The following are the
surviving relatives:
SUGGESTED ANSWER:
1. Benny and Bonnie, legitimate children of
E. None of the above.

Bobby;
There is no showing that Danny is an
2. Cesar, legitimate child of Conrad;
illegitimate
3. Dante, illegitimate child of Danny;
Armand. In the absence of proof to the
4. Ernie, adopted child of Edward; and
contrary, the law presumes that the
5. Felix, grandson of Floro.
The
net
value
of
Armand s
half-blood
brother
of
relationship is legitimate. Thus, Dante,
estate
is
Pl,200,000.
an illegitimate child of Danny, is barred
from inheriting from Armand pursuant
to
the
iron
curtain
rule
which
I. (1) How much do Benny and Bonnie
disqualifies an illegitimate child from
stand to inherit by right of representation?

inheriting
(1%)
legitimate children and relatives of his
(A) P200,000
father or mother, and vice versa (Art
(B) P300,000
992, Civil Code).
ab
intestao
from
the
(C) P400,000
(D) P150,000
I. (3) How much is Ernie s share in the net
(E) None of the above.
estate . (1%)
(A) P 0.
SUGGESTED ANSWER:
(B) P400,000.
(E) None of the above.
(C) P150,000.
If all the brothers/sisters are disqualified
(D) P200,000.
to inherit, the nephews/nieces inherit
(E) None of the above.
SUGGESTED ANSWER:
Never Let The Odds Keep You From Pursuing What You Know In Your Heart You Were Me
ant To Do.-Leroy Satchel Paige
Page 118 of 180

Civil Law Q&As (2007-2013)


hectorchristopher@yahoo.com
(A) 0 or (E) None of the above.
The
legal
relationship
dbaratbateladot@gmail.com
Should the share of insolvent debtor C be
created
by
divided
only
between
the
two
other
adoption is strictly between the adopter
remaining debtors, A and B? (1%)
and the adopted. It does not extend to
(A) Yes. Remission of D s share carries with
the relatives of either party (Sayson v.
it total extinguishment of his obligation to
CA, G.R. Nos. 89224-25, Jan 23, 1992).
the benefit of the solidary debtors.
(Note: E. None of the above is another
(B) Yes. The Civil Code recognizes remission
answer because Ernie has no share at
as a mode of extinguishing an obligation.
all in the net estate).
This clearly applies to D.
(C) No. The rule is that gratuitous acts

I. (4) How much is Felix s share in the net


should be restrictively construed, allowing
estate? (1%)
only the least transmission of rights.
(A) P400,000.
(D) No, as the release of the share of one
(B) P150,000.
debtor would then increase the burden of
(C) P300,000.
the other debtors without their consent.
(D) P0.
(E) None of the above.
SUGGESTED ANSWER:
(D). No, as the release of the share of one
SUGGESTED ANSWER:
debtor would then increase the burden of
(D). 0. Or (E) None of the above.
the other debtors without their consent.
In the collateral line, representation is
When one of the solidary debtors cannot,
granted only to children of brother or
because of his insolvency, reimburse his
sisters,
share
Felix
is
a
grandson
of
a
to

the
debtor
paying
the
predeceased brother.
obligation, such share shall be borne by
(Note: E. None of the above: is another
all his co-debtors, in proportion to the
answer because Felix has no share at
debt of each (Art 1217, Civil Code).
all in the net estate)
Additionally, D was released only from
his share of P10,000.00 not from the
II. A, B, C and D are the solidary debtors of
solidary tie that binds him to A, B and C.
X for P40,000. X released D from the
payment of his share of PI 0,000. When the
III. Amador obtained a loan of P300,000
obligation became due and demandable, C
from Basilio payable on March25, 2012. As
turned out to be insolvent.
security for the payment of his loan,
Amador constituted a mortgage on his
residential house and lot in Basilio s favor.
Never Let The Odds Keep You From Pursuing What You Know In Your Heart You Were Me
ant To Do.-Leroy Satchel Paige
Page 119 of 180

Civil Law Q&As (2007-2013)


hectorchristopher@yahoo.com
dbaratbateladot@gmail.com
Cacho, a good friend of Amador, guaranteed
Basilio
(the
creditor)
must
first
be
and obligated himself to pay Basilio, in case
exhausted)
Amador fails to pay his loan at maturity.
III. (2) If Amador sells his residential house
and lot to Diego, can Basilio foreclose the
III. (1) If Amador fails to pay Basilio his loan
real estate mortgage? (1%)
on March 25, 2012, can Basilio compel
Cacho to pay? (1%)
(A) Yes, Basilio can foreclose the real estate
mortgage because real estate mortgage
(A) No, Basilio cannot compel Cacho to pay
creates a real right that attaches to the
because as guarantor, Cacho can invoke
property.
the principle of excussion, i.e., all the
(B) Yes, Basilio can foreclose the real
assets of Basilio must first be exhausted.
estate mortgage. It is binding upon Diego
(B) No, Basilio cannot compel Cacho to
as the mortgage is embodied in a public
pay because Basilio has not exhausted

instrument.
the available remedies against Amador.
(C) No, Basilio cannot foreclose the real
(C) Yes, Basilio can compel Cacho to pay
estate
because the nature of Cacho s undertaking
ownership on the buyer, Diego, who
indicates
must therefore consent.
that
he
has
bound
himself
mortgage.
The
sale
confers
solidarily with Amador.
(D) No, Basilio cannot foreclose the real
(D) Yes, Basilio can compel Cacho who
estate mortgage. To deprive the new owner
bound himself to unconditionally pay in
of ownership and possession is unjust and
case Amador fails to pay; thus the benefit of
inequitable.
excussion will not apply.
SUGGESTED ANSWER:
SUGGESTED ANSWER:
(B) Yes, Basilio can foreclose the real
(B) No, Basilio cannot compel Cacho to

estate mortgage. It is binding upon Diego


pay because Basilio has not exhausted
as the mortgage is embodied in a public
the available remedies against Amador.
instrument.
The guarantor cannot be compelled to
Since
pay the creditor unless the latter has
instrument, there is constructive notice
exhausted all the property of the debtor
to
and has resorted to all the legal remedies
mortgaged property.
the
Diego,
mortgage
who
is
is
the
in
a
buyer
public
if
the
against the debtor (Art. 2058, Civil Code)
(Note: A is not the correct answer
ALTERNATIVE ANSWER:
because it states that all the assets of
Never Let The Odds Keep You From Pursuing What You Know In Your Heart You Were Me
ant To Do.-Leroy Satchel Paige

Page 120 of 180

Civil Law Q&As (2007-2013)


hectorchristopher@yahoo.com
dbaratbateladot@gmail.com
(C) No, Basilio cannot foreclose the real
(D) No, Jose s refusal is not justified. The
estate
confers
expenses he incurred are useful for the
ownership on the buyer, Diego, who
preservation of the thing loaned. It is
must therefore consent.
Jose s obligation to shoulder these useful
The mortgage is not registered, thus,
expenses.
mortgage.
The
sale
cannot be binding against third persons
(Art. 2125, Civil Code)
SUGGESTED ANSWER:
(D) No, Jose s refusal is not justified. The
IV. Cruz lent Jose his car until Jose
expenses he incurred are useful for the
finished his Bar exams. Soon after Cruz
preservation of the thing loaned. It is
delivered
Jose s obligation to shoulder these useful
the
car,
Jose
brought

it
to
Mitsubishi Cubao for maintenance check
expenses.
up and incurred costs of P8,000. Seeing the
In commodatum, the bailee is obliged to
car s peeling and faded paint, Jose also had
pay for the ordinary expenses for the use
the car repainted for P10,000. Answer the
and preservation of the thing loaned (Art
two
1941, Civil Code).
questions
below
based
on
these
common facts.
The
bailee,
Jose,
has
no
right
of
retention on the ground that the bailor
IV. (1) After the bar exams, Cruz asked for
owes him something, even if it may be
the return of his car. Jose said he would
by reason of expenses. He can only
return it as soon as Cruz has reimbursed

retain it if he suffers damages by reason


him for the car maintenance and repainting
of a flaw or defect in the thing loaned of
costs of P 18,000.
which the bailor knows (Art 1951, Civil
Is Jose s refusal justified? (1%)
Code).
(A) No, Jose s refusal is not justified. In this
kind of contract, Jose is obliged to pay for
IV. (2) During the bar exam month, Jose
all
lent the car to his girlfriend, Jolie, who
the
expenses
incurred
for
the
preservation of the thing loaned.
parked the car at the Mall of Asia s open
(B) Yes, Jose s refusal is justified. He is
parking lot, with the ignition key inside the
obliged to pay for all the ordinary and
car. Car thieves broke into and took the
extraordinary
car.
expenses,
but
subject
to
reimbursement from Cruz.
(C) Yes, Jose s refusal is justified. The

Is Jose liable to Cruz for the loss of the car


principle of unjust enrichment warrants the
due to Jolie s negligence? (1%)
reimbursement of Jose s expenses.
Never Let The Odds Keep You From Pursuing What You Know In Your Heart You Were Me
ant To Do.-Leroy Satchel Paige
Page 121 of 180

Civil Law Q&As (2007-2013)


hectorchristopher@yahoo.com
dbaratbateladot@gmail.com
(A) No, Jose is not liable to Cruz as the loss
Securities
and
was not due to his fault or negligence.
designated L and 0 as managing partners; L
(B) No, Jose is not liable to Cruz. In the
was liable only to the extent of his capital
absence of any prohibition, Jose could lend
contribution; and P was not liable for
the car to Jolie. Since the loss was due to
losses.
force majeure, neither Jose nor Jolie is
In 2006, the partnership earned a net profit
liable.
of P800,000. In the same year, P engaged in
(C) Yes, Jose is liable to Cruz. Since Jose
a different business with the consent of all
lent the car to Jolie without Cruz s
the
consent, Jose must bear the consequent
partnership
loss of the car.
of P500,000. In 2008,the partners dissolved
(D) Yes, Jose is liable to Cruz. The contract
the partnership. The proceeds of the sale of
between them is personal in nature. Jose
partnership

can neither lend nor lease the car to a third


settle its obligation. After liquidation, the
person.
partnership
partners.
Exchange
However,
incurred
assets
had
were
an
Commission,
in
2007,
a
net
loss
insufficient
unpaid
the
to
liability
ofP300,000.
SUGGESTED ANSWER:
(C) Yes, Jose is liable to Cruz. Since Jose
V. (l) Assuming that the just and equitable
lent the car to Jolie without Cruz s
share of the industrial partner, P, in the
consent, Jose must bear the consequent
profit in 2006 amounted to P1 00,000, how
loss of the car.
much is the share of 0, a limited partner, in

The bailee is liable for the loss of the


the P800,000 net profit? (1%)
thing, even if it should be through a
(A) P160,000.
fortuitous event if he lends or leases the
(B) P175,000.
thing to a third person, who is not a
(C) P280,000.
member of his household (Art 1942, Civil
(D) P200,000.
Code).
(E) None of the above.
V. In 2005, L, M, N, 0 and P formed a
SUGGESTED ANSWER:
partnership. L, M and N were capitalist
(C) P280,000.
partners who contributed P500,000 each,
First, deduct the share of P from the
while 0, a limited partner, contributed P1
profits.
,000,000. P joined as an industrial partner,
P700,000. Next, get the share of O by
contributing only his services. The Articles
following the proportion that the shares
of
of L, M, N, O is 1:1:1:2, respectively.
Partnership,
registered
with
the

P800,000
less
P100,000
is
Never Let The Odds Keep You From Pursuing What You Know In Your Heart You Were Me
ant To Do.-Leroy Satchel Paige
Page 122 of 180

Civil Law Q&As (2007-2013)


hectorchristopher@yahoo.com
dbaratbateladot@gmail.com
(B) No. P is not liable because there is a
V. (2) In 2007, how much is the share of 0,
valid
a
losses. Since the other partners allowed
limited
partner,
in
the
net
loss
stipulation
exempting
him
from
of P500,000? (1%)
him to engage in an outside business
(A) P 0.
activity, the stipulation absolving P from
(B) P1 00,000.
liability is valid. For 0, it is basic that a
(C) P125,000.
limited partner is liable only up to the
(D) P200,000.
extent of his capital contribution.
(E) None of the above.
(C) Yes. The stipulations exempting P and L
from losses are not binding upon the

SUGGESTED ANSWER:
creditors. 0 is likewise liable because the
(D) P200,000
partnership was not formed in accordance
A limited partner shall not become liable
with
a s a general partner unless, in addition
partnership.
to the exercise of his rights and powers
(D) No. The Civil Code allows the partners
as a limited partner, he takes part in the
to stipulate that a partner shall not be
control of the business (Art 1948, Civil
liable for losses. The registration of the
Code). In the absence of stipulation as to
Articles of Partnership embodying such
profits and losses, the share of each
stipulations serves as constructive notice to
partner
the partnership creditors.(E) None of the
in
the
losses
shall
be
the
requirements
of
a
limited

proportionate to what he may have


above is completely accurate.
contributed (Art 1797).
(E) None of the above is completely
accurate.
V. (3) Can the partnership creditors hold L,
0 and Pliable after all the assets of the
SUGGESTED ANSWER:
partnership are exhausted? (1%)
(E) None of the above is completely
(A) Yes. The stipulation exempting P from
accurate.
losses is valid only among the partners. L is
VI. Gary is a tobacco trader and also a
liable because the agreement limiting his
lending investor. He sold tobacco leaves to
liability to his capital contribution is not
Homer
valid insofar as the creditors are concerned.
although the period for delivery was not
Having taken part in the management of
guaranteed. Despite Gary s efforts to deliver
the partnership, 0 is liable as capitalist
on
partner.
government red tape hindered his efforts
for
time,
delivery
within
transportation

a
month,
problems
and
and he could only deliver after 30 days.
Never Let The Odds Keep You From Pursuing What You Know In Your Heart You Were Me
ant To Do.-Leroy Satchel Paige
Page 123 of 180

Civil Law Q&As (2007-2013)


hectorchristopher@yahoo.com
dbaratbateladot@gmail.com
Homer refused to accept the late delivery
(D) No. Homer was not justified in refusing
and to pay on the ground that the agreed
to accept the tobacco leaves. There was no
term had not been complied with.
term in the contract but a mixed condition.
As
lending
investor,
Gary
granted
a
The fulfillment of the condition did not
Pl,000,000 loan to Isaac to be paid within
depend purely on Gary s will but on other
two years from execution of the contract. As
factors, e.g., the shipping company and the
security for the loan, Isaac promised to
government. Homer should comply with his
deliver to Gary his Toyota Innova within
obligation.
seven (7) days, but Isaac failed to do so.
Gary
was
thus
compelled
to
demand

SUGGESTED ANSWER:
payment for the loan before the end of the
(B)
No.
Homer
was
not
justified
in
agreed two-year term.
refusing to accept the tobacco leaves. He
consented to the terms and conditions
VI. (l) Was Homer justified in refusing to
of
accept the tobacco leaves? (1%)
Obligations arising from contract have
(A) Yes. Homer was justified in refusing to
the force of law between the contracting
accept the tobacco leaves. The delivery was
parties.
to be made within a month. Gary s promise
It is clear under the facts that the period
of delivery on a "best effort" basis made the
of delivery of the tobacco leaves was not
delivery uncertain. The term, therefore, was
guaranteed.
ambiguous.
factors which may prevent him from
(B)
No.

abide
anticipated
by
it.
other
True enough, transportation problems
consented to the terms and conditions
and government red
of
slight delay was, thus, excusable.
must
justified
Gary
must
refusing to accept the tobacco leaves. He
and
not
and
making the delivery within a month.
sale
was
sale
in
the
Homer
the
abide
by
it.
tape
did. Such

Obligations arising from contract have


Obligations arising from contract have
the force of law between the contracting
the force of law between the contracting
parties.
parties and should be complied with in
(C) Yes. Homer was justified in his refusal
good faith (Art. 1160, Civil Code)
to
accept
the
delivery.
The
contract
contemplates an obligation with a term.
VI. (2) Can Gary compel Isaac to pay his
Since the delivery was made after 30 days,
loan even before the end of the two-year
contrary to the terms agreed upon, Gary
period? (1%)
could not insist that Homer accept the
tobacco leaves.
Never Let The Odds Keep You From Pursuing What You Know In Your Heart You Were Me
ant To Do.-Leroy Satchel Paige
Page 124 of 180

Civil Law Q&As (2007-2013)


(A)
Yes,
Gary
hectorchristopher@yahoo.com
to
Under Art 1198 (2) of the Civil Code, the
Nondebtor shall lose every right to make use
compliance with the promised guaranty
of the period when he does not furnish
or
to
immediately
security
can
pay
compel
the
renders
Isaac
dbaratbateladot@gmail.com
loan.
the
obligation
immediately demandable. Isaac lost his
the
creditor
the
guaranties
or
securities which he has promised.

right to make use of the period.


(B)
Yes,
to
VII. Lito was a commercial pilot who flew for
immediately pay the loan. The delivery of
Pacific-Micronesian Air. In 1998, he was
the Toyota Innova is a condition for the
the co-pilot of the airline s Flight MA916
loan. Isaac s failure to deliver the car
that mysteriously disappeared two hours
violated the condition upon which the loan
after
was granted. It is but fair for Gary to
presumably over the Pacific Ocean. No trace
demand immediate payment.
of the plane and its 105 passengers and
(C)
compel Isaac to
crew was ever found despite diligent search;
immediately pay the loan. The delivery of
Lito himself was never heard of again. Lito
the car as security for the loan is an
left behind his wife, Lita, and their two
accessory contract; the principal contract is
children.
No,
Gary
Gary
can

compel
cannot
Isaac
take-off
from
Agana,
Guam,
still the P 1,000,000 loan. Thus, Isaac can
still make use of the period.
In 2008, Lita met and married Jaime. They
(D)
now have a child of their own.
No,
Gary cannot
compel Isaac to
immediately pay the loan. Equity dictates
While on a tour with her former high school
that Gary should have granted a reasonable
classmates in a remote province of China in
extension of time for Isaac to deliver his
2010, Lita was surprised to see Lito or
Toyota Innova. It would be unfair and
somebody who looked exactly like him, but
burdensome
pay
she was sure it was Lito because of the
the
extreme surprise that registered in his face
for
the P1,000,000
Isaac

simply
to
because
promised security was not delivered.
when he also saw her. Shocked, she
immediately fled to her hotel and post haste
SUGGESTED ANSWER:
(A)
Yes,
Gary
immediately
can
pay
returned to the country the next day. Lita
compel
the
Isaac
loan.
to
Nonnow comes to you for legal advice. She asks
you the following questions:
compliance with the promised guaranty
or
security
renders
the
obligation
immediately demandable. Isaac lost his
VII. (l) If Lito is alive, what is the status of
his marriage to Lita? (1%)
right to make use of the period.
Never Let The Odds Keep You From Pursuing What You Know In Your Heart You Were Me
ant To Do.-Leroy Satchel Paige

Page 125 of 180

Civil Law Q&As (2007-2013)


hectorchristopher@yahoo.com
dbaratbateladot@gmail.com
(A) The marriage subsists because the
(B) The marriage is valid. After an absence
marital bond has not been terminated by
of more than 10 years, Lito is already
death.
presumed dead for all purposes.
(B) The marriage was terminated when Lita
(C)
married Jaime.
absence, however lengthy, is insufficient to
(C) The marriage subsists because Lita s
authorize Lita to contract a subsequent
marriage to Jaime is void.
marriage.
(D) The marriage is terminated because Lito
(D) The marriage is void. If Lito is indeed
is presumed dead after his plane has been
alive, his marriage to Lita
missing for more than 4 years.
dissolved and they can resume their marital
(E) The marriage can be formally declared
relations at any time.
The marriage
is
void.
Lito s mere
was never

terminated if Lito would not resurface.


SUGGESTED ANSWER:
SUGGESTED ANSWER:
Any answer is correct.
(C) The marriage subsists because Lita s
Under Art 390 of the Civil Code, after an
marriage to Jaime is void.
absence
For
the
purpose
years,
it
being
unknown whether or not the absentee
subsequent marriage under Art 41 of the
still lives, he shall be presumed dead doe
Family Code, the spouse present must
all
institute
succession.
summary
contracting
seven
the
a
of
of
proceeding
as
purposes,

except
This
for
provision
those
was
of
not
provided in the Family Code for the
repealed by the present Family Code.
declaration of presumptive death of the
Applying this to the problem, (A) may be
absentee, without prejudice to the effect
correct. (B) may also be correct. (C) and
of
(D) may also be correct under Art 41 of
the
reappearance
of
the
absent
spouse.
the Family Code.
VIII.Which
VII. (2) If Lito is alive, what is the status of
of
the
following
actions
or
defenses are meritorious: (1%)
Lita s marriage to Jaime? (1%)

(A) An action for recovery of down


(A) The marriage is valid because Lita s
payment paid under a rescinded oral sale
marriage to Lito was terminated upon Lito s
of real property.
disappearance for more than seven years.
(B) A defense in an action for ejectment that
the lessor verbally promised to extend or
renew the lease.
Never Let The Odds Keep You From Pursuing What You Know In Your Heart You Were Me
ant To Do.-Leroy Satchel Paige
Page 126 of 180

Civil Law Q&As (2007-2013)


hectorchristopher@yahoo.com
dbaratbateladot@gmail.com
(C) An action for payment of sum of money
filed against one who orally promised to
ALTERNATIVE ANSWER:
answer another s debt in case the latter
(E) None of the above.
defaults.
(a) The recovery of the down payments
(D) A defense in an action for damages that
should be made in the same action for
the debtor has sufficient, but unliquidated
rescission. Otherwise, it would be a
assets to satisfy the credit acquired when it
ground for dismissal under Rule 2, Sec 4
becomes due.
of Rules of Court.
(E) None of the above.
(b) Lease of a real property is covered by
the Statute of Frauds. Furthermore, it
SUGGESTED ANSWER:
also consists of interest in real property.
(A) An action for recovery of down
Hence, it must be in writing. (Art 1403,
payment paid under a rescinded oral sale
Civil Code)
of real property.
(c) A contract of guaranty is a promise to
An oral sale of real property is an
answer for the debt of another and

unenforceable
the
hence, it is also covered by the Statute
Statute of Frauds. Since, in the problem,
of Frauds. It must be in writing before it
the vendee paid down payment, it takes
can be enforced in a court action. (Art
it out of the ambit of Statute of Frauds.
1403, Civil Code)
The rescission here must be in the sense
(d)
of resolution of the reciprocal obligation
unliquidated assets does not excuse him
arising from the contract of sale. If
from paying his debt.
rescinded (resolved) by the vendee on
(e) In the technical meaning of rescission
account of the vendors failure to deliver
under Art 1191 of the Civil Code will be
the thing sold, the parties will go back to
adhered to, then there is no absolutely
their status prior to the contract. If the
correct answer. Hence, letter E is also a
vendor
possible answer.
refuses
contract
to
return
under

the
down
The
fact
that
a
debtor
has
payment, then the vendee can file an
action to recover the down payment.
IX. Betty entrusted to her agent, Aida,
If, on the other hand, the vendor and the
several pieces of jewelry to be sold on
vendee mutually agree to rescind i.e.
commission with the express obligation to
cancel the contract, the vendee likewise
turn over to Betty the proceeds of the sale,
can file an action for the recovery of the
or to return the jewelries if not sold in a
down payment on the basis of solution
month s
indebiti.
jewelries,
time.
Instead
of
selling
the
Aida pawned them with the
Never Let The Odds Keep You From Pursuing What You Know In Your Heart You Were Me
ant To Do.-Leroy Satchel Paige

Page 127 of 180

Civil Law Q&As (2007-2013)


Tambunting
hectorchristopher@yahoo.com
Pawnshop,
and
used
dbaratbateladot@gmail.com
the
(C) I will rule in favor of Tambunting. Its
money for herself. Aida failed to redeem the
good faith takes precedence over the right
pawned jewelries and after a month, Betty
of Betty to recover the jewelries.
discovered what Aida had done. Betty
(D) I will rule in favor of Tambunting. Good
brought criminal charges which resulted in
faith is always presumed. Tambunting s
Aida s conviction for estafa.
lawful acquisition in the ordinary course of
business coupled with good faith gives it
Betty thereafter filed an action against
legal right over the jewelries.
Tambunting Pawnshop for the recovery of
the
jewelries.
Tambunting
raised
the
SUGGESTED ANSWER:
defense of ownership, additionally arguing
(A) I will rule in favor of Betty. My ruling

that it is duly licensed to engage in the


is based on the Civil Code provision that
pawnshop and lending business, and that it
one who has lost any movable or has
accepted the mortgage of the jewelry in
been unlawfully deprived thereof may
good faith and in the regular course of its
recover it from the person in possession
business.
of the same. Tam bunting s claim of good
faith is inconsequential.
If you were the judge, how will you decide
the case? (1%)
Although possession of movable property
acquired in good faith is equivalent to a
(A) I will rule in favor of Betty. My ruling
title, nevertheless, one who has lost any
is based on the Civil Code provision that
movable or has been unlawfully deprived
one who has lost any movable or has
thereof may recover it from the person
been unlawfully deprived thereof may
in possession of the same. Betty has
recover it from the person in possession
been deprived unlawfully of her jewelries
of the same. Tam bunting s claim of good
by the estafa committed by Aida. The
faith is inconsequential.
pledge of the said jewelries by Aida to
(B)
I
will

rule
Betty.
Tambunting pawnshop is void because
Tambunting s claim of good faith pales into
the pledgor is not the owner (Art 2085
insignificance
unlawful
(2), Civil Code). Tambuntings claim of
deprivation of the jewelries. However, equity
good faith is inconsequential, because,
dictates
be
aside from good faith, Tambunting must
reimbursed for the pawn value of the
prove also that it acquired the jewelries
jewelries.
at a public sale in order to be able to
that
in
in
light
favor
of
the
Tambunting
of
must
retain the jewelries until reimbursed by
Never Let The Odds Keep You From Pursuing What You Know In Your Heart You Were Me
ant To Do.-Leroy Satchel Paige
Page 128 of 180

Civil Law Q&As (2007-2013)


hectorchristopher@yahoo.com
dbaratbateladot@gmail.com
Betty the amount of loan including
If you were the judge, would you grant
interest (Art 559, Civil Code).
Arlene s motion? (1%)
The only exception the law allows is
(A) Yes, I will grant the motion because the
when there is acquisition in good faith of
lease contract between Arlene and Janet
the possessor at a public sale, in which
was not in writing, hence, Janet may not
case, the owner cannot obtain its return
enforce any right arising from the same
without reimbursing the price (Dizon v.
contract.
Suntay, 47 SCRA 160, Sept 29, 1972).
(B) No, I will not grant the motion because
to allow Arlene to retain the advance
X. Arlene owns a row of apartment houses
payments
would
amount
to
unjust
in Kamuning, Quezon City. She agreed to
enrichment.
lease Apartment No. 1 to Janet for a period
(C) Yes, I will grant the motion because the
of 18 months at the rate of P10,000 per

action for recovery is premature; Janet


month. The lease was not covered by any
should first secure a judicial rescission of
contract. Janet promptly gave Arlene two
the contract of lease.
(2) months deposit and 18 checks covering
(D) No. I will not grant the motion
the rental payment for 18 months. This
because the cause of action does not
show of good faith prompted Arlene to
seek to enforce any right under the
promise Janet that should Arlene decide to
contract of lease.
sell the property, she would give Janet the
right of first refusal.
SUGGESTED ANSWER:
X. (1) Not long after Janet moved in, she
(D) No. I will not grant the motion
received news that her application for a
because the cause of action does not
Master
seek to enforce any right under the
of
Laws
scholarship
at
King s
College in London had been approved.
contract of lease.
Since her acceptance of the scholarship

Janet is not asking for the continued use


entailed a transfer of residence, Janet
of the leased premises. Moreover, the
asked Arlene to return the advance rental
contract
payments
Statute of Frauds as the same has
she
made.
Arlene
refused,
prompting Janet to file an action to recover
is
aside
the
ambit
of
the
already been partially performed.
the payments. Arlene filed a motion to
dismiss, claiming that the lease on which
X. (2)Assume that Janet decided not to
the action is based, is unenforceable.
accept
the
scholarship
and
continued
leasing Apartment No. 1. Midway through
Never Let The Odds Keep You From Pursuing What You Know In Your Heart You Were Me
ant To Do.-Leroy Satchel Paige

Page 129 of 180

Civil Law Q&As (2007-2013)


hectorchristopher@yahoo.com
dbaratbateladot@gmail.com
the lease period, Arlene decided to sell
Frauds under Art 1403 (2)(e) of the Civil
Apartment No. 1 to Jun in breach of her
Code. It must be in writing in order to be
promise to Janet to grant her the right of
enforceable.
first refusal. Thus, Janet filed an action
seeking the recognition of her right of first
refusal, the payment of damages for the
violation of this right, and the rescission of
the sale between Arlene and Jun.
2012
Taxation
Law
Exam
MCQ (October 14, 2012)
1. Which of the following is NOT included in
Is Janet s action meritorious? (1%)
the attributes of juridical capacity?
a) Juridical capacity is inherent in
(A) Yes, under the Civil Code, a promise to
buy
and
sell
a
determinate
thing
every natural person, and therefore
is
it is not acquired.
reciprocally demandable.

b) Juridical capacity is lost only


(B) No, the promise to buy and sell a
through death.
determinate thing was not supported by a
c) Juridical capacity is the fitness to
consideration.
be the subject of legal relations.
(C) Yes, Janet s right of first refusal was
d) Juridical capacity cannot exist
clearly violated when the property was not
without capacity to act.
offered for sale to her before it was sold to
Jun.
(D) No, a right of first refusal involves an
interest over real property that must be
embodied in a written contract to be
enforceable.
(E) None of the above.
SUGGESTED ANSWER:
(D) No, a right of first refusal involves an
interest over real property that must be
embodied in a written contract to be
enforceable.
The right of first refusal involves a
transfer of interest in the real property.
2.
Which
of
the
following
is
NOT
a
restriction on ones capacity to act?
a) Minority
b) Marriage
c) Deaf-mute
d) Civil Interdiction
SUGGESTED ANSWER:

This question should be disregarded.


(NOTE: There is no correct answer among
the choices given. All choices are restrictions
on ones capacity to act. While Marriage is
the only one not mentioned in Articles 38
and 39 of the NCC as a restriction on
As such, it is covered by the Statute of
Never Let The Odds Keep You From Pursuing What You Know In Your Heart You Were Me
ant To Do.-Leroy Satchel Paige
Page 130 of 180

Civil Law Q&As (2007-2013)


hectorchristopher@yahoo.com
dbaratbateladot@gmail.com
capacity to act, it restricts the capacity of a
b) Yes, the will is not valid under
married person in cases of adoption.)
Philippine law.
c) No, attestation clause is not an
3. This attribute or incident of a case
act of the testator.
determine whether it is a conflict-of-laws
d)
case or one covered by domestic law.
Spanish law.
No,
the
governing
law
is
a) Cause of action
b) Foreign element
Note: The facts do not state the Law
c) Jurisdiction
observed by the testator in executing his
d) Forum non conveniens
will. He could have observed Spanish Law
or Philippine Law (see comment of Tolentino
4. The capacity of an heir to succeed shall
to Art. 815 NCC in 3Tolentino117, 1992). If
be governed by the:
he observed Spanish Law, the opposition is
a) national law of the decedents

not correct because the will is valid under


heirs
Spanish Law, hence choice (d) is the correct
b) law of the country where the
answer. If he observed Philippine Law, the
decedent was a resident at the time
opposition
of his death
Philippine Law does not require the testator
c) national law of the person who
to sign the Attestation Clause of his will,
died
said clause not being his act. In such case,
d) law of the country where the
choice (c) is the correct answer).
properties
of
the
decedent
is
still
not
correct
because
are
located.
6. Ramon, a Filipino, executed a will in
Manila, where he left his house and located
5. Atty. BUKO, a Filipino, executed a will
in BP Homes Paraaque in favor of his

while he was in Spain. The attestation


Filipino
clause of the said will does not contain
children RJ and Ramona, both Turkish
Bukos signature. It is valid under Spanish
nationals, are disputing the bequest to
law. At its probate in Manila, it is being
Ramgen. They plotted to kill Ramgen.
opposed on the ground that the attestation
Ramon learned of the plot, so he tore his
clause does not contain BUKOs signature.
will in two pieces out of anger. Which
Is the opposition correct? Choose the best
statement is most accurate?
answer..
a) Yes, because it is a fatal defect.
son,
Ramgen.
Ramons
other
a) The mere act of Ramon Sr. is
immaterial because the will is still
readable.
Never Let The Odds Keep You From Pursuing What You Know In Your Heart You Were Me
ant To Do.-Leroy Satchel Paige
Page 131 of 180

Civil Law Q&As (2007-2013)


hectorchristopher@yahoo.com
dbaratbateladot@gmail.com
b) The mere act of tearing the will
amounts to revocation.
9. A Japanese national and a Filipino
c) The tearing of the will may
national entered into a contract for services
amount to revocation if coupled
in Thailand. The services will be rendered
with intent of revoking it.
in Singapore. In case of breach, what law
d) The act of tearing the will is
will govern?
material.
a) Thailand law
b) Philippine law
7. Even if the applicable law is a foreign
c) Singapore law
law, a count in the Philippines may be
d) Japanese law
constrained to apply Philippine law under
any of the following instances, except:
10. Pedro (Filipino) and his wife Jane
a) when the foreign law, judgment or
(American) executed a joint will in Canada,
contract is contrary to a sound and
where such joint will is valid. In case the
important
joint will is probated in Japan, what law
public
policy

of
the
forum;
will govern the formalities of the joint will?
b) when the property subject of
a) American law
the case is located outside of the
b) Philippine law
Philippines;
c) Canadian law
c)
when
the
foreign
law
or
d) Japanese law
judgment is penal in nature;
d)
when
the
foreign
law
is
procedural in nature.
11. A French national revokes his will in
Japan where he is domiciled. He then
changed his domicile to the Philippines
8. If a will is executed by a testator who was
where he died. The revocation of his will in
born

became
Japan is valid under Japanese law but
naturalized Japanese citizen at the time of
invalid under Philippine law. The affected
his
heir is a Malaysian national residing in the
a
Filipino
death,
testamentary
what
citizen
law
provisions
but
will
if
govern
the
will
its
is
Philippines. What law will apply?
executed in China and the property being
a) Japanese law
disposed is located in Indonesia?
b) Philippine law
a) Chinese law
c) French law
b) Philippine law
d) Malaysian law
c) Indonesia law
d) Japanese law

Never Let The Odds Keep You From Pursuing What You Know In Your Heart You Were Me
ant To Do.-Leroy Satchel Paige
Page 132 of 180

Civil Law Q&As (2007-2013)


hectorchristopher@yahoo.com
dbaratbateladot@gmail.com
12. In the absence of contrary stipulation in
obligation, but Bill fails or refuses to pay,
a marriage settlement, property relations of
what law will govern?
Filipino spouses shall be governed by --a) American law
a) Philippines laws
b) Philippine law
b) Law of the place where the
c) Australian law
spouses reside
d) Japanese law
c) Law of the place where the
(Facts for item numbers 15-18)
properties are situated
In 1989, Charice (Filipina) and Justine
d) Law of the place where they were
(American), were married in the Philippines.
married.
In 1990, they separated and Justine went
to Las Vegas where he obtained a divorce in
13. The will of a Filipino executed in a
the same year. He then married another
foreign country --Filipina, Lea, in Canada on January 1,
a)
cannot
be

probated
in
the
Philippines;
b)
may
1992. They had two (2) sons, James and
John (who were both born in 1992). In
the
1993, after failing to hear from Justine,
Philippines provided that properties
Charice married Bugoy (a Filipino), by
in the estate are located in the
whom she had a daughter, Regine. In 2009,
Philippines;
Regine married James (son of Justine with
c) cannot be probated before the
Lea) in California, where such marriage is
death of the testator;
valid.
d)
may
be
be
Philippines
probated
probated
provided
in
in
it
the

was
15. What is the current status of the
executed in accordance with the
marriage of Charice and Justine under
laws of the place where the will
Philippine laws?
was executed.
a) Valid
b) Void
14. Pedro (Filipino and Bill (American)
c) Voidable
entered
d) Dissolved
into
a
contract
in
Australia,
whereby it was agreed that Pedro will build
(Note: While Art 26 of the FC does not
a commercial building for Bill in the
categorically provide that the first marriage
Philippines,
the
is dissolved by the divorce obtained by the
construction, Bill will transfer and convey
foreign spouse abroad, but provides that
his cattle ranch located in Japan in favor of
such divorce merely gives the Filipino spouse
Pedro.
the capacity to contract a second marriage, it

In
and
case
in
payment
Pedro
for
performs
his
Never Let The Odds Keep You From Pursuing What You Know In Your Heart You Were Me
ant To Do.-Leroy Satchel Paige
Page 133 of 180

Civil Law Q&As (2007-2013)


hectorchristopher@yahoo.com
dbaratbateladot@gmail.com
is believed that the dissolution of the first
examinations unless he marries Princess.
marriage us the necessary consequence of
As a consequence of the threat, Ricky
the foreign divorce.)
married Princess. Can the marriage be
annulled on the ground of intimidation
16. What id the status of the marriage
under Article 45 of the Family Code?
between
Choose the best answer.
Charice
and
Bugoy
under
Philippine laws?
a) Yes, because without the threat,
a) Valid
Ricky
would
not
have
married
b) Void
Princess.
c) Voidable
b) Yes, because the threat to enforce
d) Unenforceable

the claim of Princess vitiates the


consent of Ricky in contracting the
17. What is the status of the marriage
marriage.
between
c) No, because the threat made by
Charice
and
Bugoy
under
Philippine laws?
Marforth is just and legal.
a) Valid
d) No, because Marforth is not a
b) Void
party to the contract of marriage
c) Voidable
between Princess and Ricky.
d) Unenforceable
20. Audrey, single, bought a parcel of land
18. What is the status of the marriage
in Malolos City from Franco for P 1Million.
between
A contract was executed between them
Regine
and
James
under
Philippine laws?
which already vested upon Audrey full
a) Valid

ownership
of
the
property,
although
b) Void
payable in monthly installments for a
c) Voidable
period of four (4) years. One (1) year after
d) Unenforceable
the execution of the contract, Audrey got
married to Arnel. They executed a marriage
19. Ricky and Princess were sweethearts.
settlement whereby they agreed that their
Princess became pregnant. Knowing that
properties shall be governed by the regime
Ricky is preparing for the examinations,
of conjugal partnership of gains. Thereafter,
Marforth, a lawyer and cousin of Princess,
subsequent installments were paid from the
threatened
conjugal partnership funds. Is the land
Ricky
with
the
filing
of
a
complaint for immorality in the Supreme
conjugal or paraphernal?
Court, thus preventing him from taking

Never Let The Odds Keep You From Pursuing What You Know In Your Heart You Were Me
ant To Do.-Leroy Satchel Paige
Page 134 of 180

Civil Law Q&As (2007-2013)


hectorchristopher@yahoo.com
dbaratbateladot@gmail.com
a) The land is conjugal because the
the laws of such consular official. Under
installments were paid from the
Philippine law, what is the status of the
conjugal partnership funds.
marriage of Agay and Topacio? Choose the
b)
best answer.
The
land
is
paraphernal
because ownership thereof was
a)
Void,
because
acquired before the marriage.
official
c) The land is both conjugal and
solemnize
paraphernal funds of installments
Filipinos.
were paid from both the personal
b) Valid, because according to the
funds of Audrey and the conjugal
laws of Australia, such consular
partnership funds.

official has authority to celebrate the


d) The land is paraphernal because
marriage.
it was Audrey who purchased the
c) Voidable, because there is an
same.
irregularity in the authority of the
only
consular
21.
Ernesto
donated
worth P 32,000
mobile
between
solemnize
marriages.
delivered the unit to Hubert who accepted.
recognized as valid in the place
Which statement is most accurate?
where it was celebrated.
donation
orally
to
to
d) Valid, because such marriage is
The
Hubert
phone
authority
marriages

official
consular
and
a)
to
a
has
the
is
void
and
Ernesto may get mobile phone
(Note: The issues in the problem is whether
back.
or not the fact that one of the parties to the
b) The donation is void but Ernesto
marriage was an alien constituted absence
cannot get the mobile phone back.
of authority or mere irregularity of authority.
c) The donation is voidable and may
The problem only give the choice, letter (a),
be anulled.
in case it is interpreted as absence of
d) The donation is valid.
authority. The problem does not give a
choice in case it is interpreted as an
22. Agay, a Filipino citizen and Topacio, an
irregularity thereby making all the other
Australian
answers wrong).

consular
Australia.
citizen,
office
got
of
According
married
the
to
in
Philippines
the
laws
the
in
of
Australia, a marriage solemnized by a
23. Separation of property between spouses
consular official is valid, provided that such
during the marriage may take place only:
marriage is celebrated in accordance with
a) by agreement of the spouses.
Never Let The Odds Keep You From Pursuing What You Know In Your Heart You Were Me
ant To Do.-Leroy Satchel Paige
Page 135 of 180

Civil Law Q&As (2007-2013)


hectorchristopher@yahoo.com
dbaratbateladot@gmail.com
b) If one of the spouses has given
b) No trial shall be held without the
ground for legal separation.
6-month cooling off period being
c) Upon order of the court.
observed.
d) If one spouse has abandoned the
c) The spouses will be entitled to live
other.
separately upon the start of the
trial.
24.
The
husband
may
impugn
the
d) The prosecuting attorney has
legitimacy of his child but not on the
to conduct his own investigation.
ground that:
a)
the
wife
is
suspected
of
27.

A
husband
infidelity.
hidden
treasure
b) the husband had a serious illness
property
that prevented him from engaging in
discovered treasure?
of
his
The
by
chance
on
the
wife.
paraphernal
Who
owns
a)
c) they were living apart.
husband (finder)
d) he is physically incapable of
conjugal partnership.
sexual intercourse.
b) The half pertaining to the wife (as
25. A marriage is void if:
a)
solemnized
pertaining

belongs
to
the
sexual intercourse.
owner)
half
discovered
the
belongs to the
to
the
conjugal
partnership.
marriage
c) One half shall belong to the
license issued without complying
husband as finder and the other
with the required 10-day posting.
half shall belong to the wife as
b) solemnized by a minister whom
owner of the property.
the
d) a and b
parties
with
a
believe to
have the
authority.
c) between parties both 23 years of
28. Which of the following marriages is void

age but without parental advice.


for reasons of public policy?
d) none of the above
a) Between brothers and sisters,
whether of the full or half blood.
26.
In
legal
separation,
which is not
correct?
b) Between step-parents and step
children.
a) The aggrieved spouse may file the
c)
Between
parents-in-law
action within five (5) years from the
children-in-law.
time of the occurrence of the cause.
d) b and c
and
Never Let The Odds Keep You From Pursuing What You Know In Your Heart You Were Me
ant To Do.-Leroy Satchel Paige
Page 136 of 180

Civil Law Q&As (2007-2013)


hectorchristopher@yahoo.com
dbaratbateladot@gmail.com
29. The following constitute the different
a)
Children
conceived
or
born
circumstances or case of fraud which will
outside a valid marriage.
serves as ground for the annulment of a
b) Children born under a valid
marriage, except?
marriage, which was later declared
a) Non-disclosure of the previous
void because of the psychological
conviction by final judgment of the
incapacity of either or both of the
other party of a crime involving
spouses.
moral turpitude.
c) Children conceived and born
b)
Concealment
of
a
sexuallyoutside a valid marriage.
transmissible disease, regardless of

d) Children born under a valid


its nature, existing at the time of the
marriage,
marriage.
obtained a legal separation.
but
the
parents
later
c) Concealment of drug addiction,
habitual alcoholism, homosexuality
32. An illegitimate child may use the
or lesbianism existing at the time of
surname of his father when his filiation is
marriage.
established
d) Concealment by the wife or the
instances, except:
in
any
of
the
following
husband of the fact of sexual
a) Filiation has been recognized by
relations prior to the marriage.
the father through the record of
birth appearing in the civil register
30. Which of the following is not a requisite
b) Admission of filiation by the
for a valid donation propter nuptias?

father in a public document.


a) The donation must be made
c) Private handwritten instrument is
before
made by the father acknowledging
the
celebration
of
the
marriage.
b)
The
his filiation.
donation
shall
be
automatically revoked in case of
d) Affidavit by the mother stating
the name of his true father.
non-celebration of the marriage.
c) The donation must be made in
33. Under RA 8043, an adopter is required
consideration of the marriage.
to be at least ____ years old and ____ years
d) The donation must be made in
older than the child to be adopted at the
favor of one or both of the future
time of the application unless the adopter is
spouses.
the parent by nature of the child.
a) 30 and 15
31. Who are illegitimate children?
b) 27 and 16

Never Let The Odds Keep You From Pursuing What You Know In Your Heart You Were Me
ant To Do.-Leroy Satchel Paige
Page 137 of 180

Civil Law Q&As (2007-2013)


hectorchristopher@yahoo.com
dbaratbateladot@gmail.com
c) 50 and 10
d) Subjects the child or allows him
d) 18 and 15
to
be
subjected
to
acts
of
lasciviousness.
34. Under RA 8043, a child qualified to be
adopted is any person below _____ years
37. Which of the following statements
old.
is wrong?
a) 18
a) The possessor in bad faith shall
b) 21
reimburse the fruits received and
c) 15
those which the legitimate possessor
d) 16
could have received.
b) The possessor in bad faith has
35. Which of the following DOES NOT
right
of
reimbursement
result in permanent termination of parental

necessary expenses and those for


authority?
the
production,
gathering
for
and
a) Death of the parents.
preservation of the fruits.
b) Death of the child.
c) The possessor in bad faith is not
c) Emancipation of the child.
entitled to a refund of ornamental
d) Conviction of the parents of a
expenses.
crime which carries with it the
d) The possessor in bad faith is
penalty of civil interdiction.
entitled to a refund of useful
expenses.
36. The court, in an action filed for the
purpose, may suspend parental authority if
38.
the parent or the person exercising parental
completes the statement The expenses
authority commits any of the following
incurred in improvements for the luxury or
acts, except:
mere pleasure shall not be refunded to thew
a) Treats the child with excessive
Which
phrase

most
accurately
possessor in bad faith:
harshness or cruelty.
a) but he may remove the objects
b) Gives the child corrupting orders,
for which such expenses have
counsel or example.
been incurred, provided that the
c) Compels the child to take up a
thing suffers no injury thereby,
course in college against his/her
and that the lawful possessor does
will.
not prefer to retain them.
Never Let The Odds Keep You From Pursuing What You Know In Your Heart You Were Me
ant To Do.-Leroy Satchel Paige
Page 138 of 180

Civil Law Q&As (2007-2013)


hectorchristopher@yahoo.com
dbaratbateladot@gmail.com
b) and he may not remove the
of the dominion over the property as owner.
objects for which such expenses
What action is being referred to?
have been incurred.
a) Accion publiciana
c) and he may not remove the
b) Accion reinvindicatoria
objects for which such expenses
c) Accion interdictal
have been incurred, unless he pays
d) Quieting of Title
the value they may have at the time
he entered into possession.
42. A summary action to recover physical
d) but he may remove the objects for
or material possession only and must be
which such expenses have been
brought within one (1) year from the time
incurred.
the cause of action arises. What action is
being referred to?
39. The following are the limitations on the
a) Accion publiciana
right of ownership imposed by the owner
b) Accion reinvindicatoria
himself, except:
c) Accion interdictal

a) Will/Succession
d) Quieting of Title
b) Mortgage
c) Pledge
43. The following things are property of
d) Lease
public dominion, except:
a) ports and bridges constructed by
40. A plenary action for the recovery of the
the State.
possession
b) vehicles and weapons of the
of
real
allegation and proof
estate,
upon
mere
of a better right
Armed Forces of the Philippines.
thereto, and without allegation of proof of
c) rivers.
title. This action can only be brought after
d) lands reclaimed by the state from
the expiration of one (1) year. What action
the sea.
is being referred to?
a) Accion publiciana
SUGGESTED ANSWER:
b) Accion reinvindicatoria
This
c) Accion interdictal

because there is no correct answer.


d) Quieting of Title
(Note: At first glance, one gets the impression
question
should
be
disregarded
that vehicles and weapons of the AFP are
41. Action to recover real property based on
not property of the public domain. But they
ownership. Here, the object is the recovery
are actually property of the public dominion
under the second paragraph of Art 420 of
Never Let The Odds Keep You From Pursuing What You Know In Your Heart You Were Me
ant To Do.-Leroy Satchel Paige
Page 139 of 180

Civil Law Q&As (2007-2013)


hectorchristopher@yahoo.com
dbaratbateladot@gmail.com
the NCC. Property of the state which are not
a) the donee accepts the donation.
for public use but are intended for some
b) the donor executes the deed of
public service are properties of the public
donation.
dominion. While the vehicles and weapons of
c) the donor knows of the donees
the AFP are not for public use, they are used
acceptance even if the latter has
for the defense of the State which is a public
not received the copy of the deed
service.)
of donation.
d) the donee confirms that the donor
44. Which of the following statements
has learned the formers acceptance.
is wrong?
a) patrimonial property of the
47. The following are the elements of an
state, when no longer intended
obligation, except:
for
public
use
or
for
public

a) Juridical/Legal Tie
service, shall become property of
b) Active subject
public dominion.
c) Passive subject
b) all property of the State, which is
d) Consideration
not
of
public
dominion,
is
patrimonial property.
48. It is a conduct that may consist of
c) The property of provinces, cities
giving, doing, or not doing something.
and municipalities is divided into
a) Obligation
property
b) Juridical necessity
for
public
use
and
patrimonial property.
d)
Property
is
c) Prestation
either
of

public
dominion or of private ownership.
d) Contract
49.
It
is
a
juridical
relation
arising
from lawful, voluntary and unilateral acts
45.
The following
cannot
ask for
the
reduction of inofficious donation, except:
a) Creditors of the deceased
based on the principle that no one should
unjustly enrich himself at the expense of
another.
b) Devisees or legatees
a) Quasi-contract
c) Compulsory heirs of the donor
b) Quasi-delict
d)
c) Cotract
The surviving spouse of the
donee.
d) Delict
46. Donation is perfected from the moment
50. The following are the elements of quasi-

--delict, except:
Never Let The Odds Keep You From Pursuing What You Know In Your Heart You Were Me
ant To Do.-Leroy Satchel Paige
Page 140 of 180

Civil Law Q&As (2007-2013)


hectorchristopher@yahoo.com
dbaratbateladot@gmail.com
a) Act or omission
54. It is an international evasion of the
b) Fault/negligence
faithful performance of the obligation.
c) Damage/injury
a) Negligence
d) Pre-existing contract
b) Fraud
c) Delay
51. A debtor is liable for damages in case of
delay
if
he
is
guilty
of
any
of
d) Mistake
the
following, except:
55. The following are the requisites of
a) default (mora)
fortuitous event, except:
b) mistake
a) Cause is independent of the will
c) negligence (culpa)
of the debtor.

d) breach through contravention of


b)
the tenor thereof
is unforeseeable/unavoidable.
The
event
c) Occurrence renders it absolutely
52. This term refers to a delay on the part
impossible for the debtor to fulfill his
of both the debtor and creditor in reciprocal
obligation
obligations.
impossibility must be absolute not
in
a
normal
manner;
a) Mora accipiendi
partial, otherwise not force majeure.
b) Mora solvendi
d)
c) Compensation morae
aggravation of the injury to the
d) Solution indibiti
creditor.
Debtor
contributed
to
the
53. The following are the requisites of mora
56. A debtor may still be held liable for loss

solvendi, except:
or damages even if it was caused by a
a) Obligation pertains to the debtor
fortuitous event in any of the following
and
instances, except:
is
determinate,
due,
demandable, and liquidated.
a) The debtor is guilty of dolo, malice
b) Obligation was performed on its
or bad faith, has promised the same
maturity date.
thing to two or more persons who do
c) There is judicial or extrajudicial
not have the same interest.
demand by the creditor.
b) The debtor contributed to the
d) Failure of the debtor to comply
loss.
with such demand.
c) The thing to be delivered is
generic.
Never Let The Odds Keep You From Pursuing What You Know In Your Heart You Were Me
ant To Do.-Leroy Satchel Paige
Page 141 of 180

Civil Law Q&As (2007-2013)


hectorchristopher@yahoo.com
dbaratbateladot@gmail.com
d) The creditor is guilty of fraud,
c) No, because a motion to dismiss
negligence
is a prohibited pleading.
or
contravened
delay
the
or
tenor
if
of
he
the
d) Yes, because Fermin and Toti
obligation.
should also pay their share of the
obligation.
57.
Buko,
themselves
Fermin
solidarily
and
59. Buko, Fermin and Toti are solidarily
amount of P 5,000.00. Suppose Buko paid
debtors of Ayee. Twelve (12) years after the
the obligation, what is his right as against
obligation became due and demandable,

his co-debtors?
Buko paid Ayee and later on asked for
Buko
pay
bound
the
a)
to
Toti
cas
Ayee
ask
for
reimbursement from Fermin and
reimbursement
of
Fermins
and
Totis
shares. Is Buko correct? Why?
Toti.
a) No, because the obligation has
b) Buko can sue Fermin and Toti for
already prescribed.
damages.
b) Yes, because the obligation is
c) Buko can sue for rescission.
solidary.
d) Buko can claim a refund from
c) No, because in solidary obligation
Ayee.

any one of the solidary debtors can


pay the entire debt.
58.
Buko,
Fermin
and
Toti
bound
d) Yes, because Fermin and Toti will
themselves solidarily to pay Ayee the sum
be unduly enriched at the expense
of P 10,000.00. When the obligation became
of Buko.
due and demandable, Ayee sued Buko for
the
payment
of
the P 10,000.00.
Buko
60. Buko, Fermin and Toti are solidary
moved to dismiss on the ground that there
debtors
under
a
loan
obligation
was failure to implead Fermin and Toti who
of P 300,000.00 which has fallen due. The
are indispensable parties. Will the motion
creditor has, however, condoned Fermins
to dismiss prosper? Why?

entire share in the debt. Since Toti has


a) Yes, because Fermin and Toti
become insolvent, the creditor makes a
should have been impleaded as their
demand on Buko to pay the debt. How
obligation is solidary.
much, if any, may Buko be compelled to
b) No, because the creditor may
pay?
proceed against any one of the
a) P 200.000.00
solidary debtors or some or all of
b) P 300,000.00
them simultaneously.
c) P 100,000.00
Never Let The Odds Keep You From Pursuing What You Know In Your Heart You Were Me
ant To Do.-Leroy Satchel Paige
Page 142 of 180

Civil Law Q&As (2007-2013)


hectorchristopher@yahoo.com
d) P 150,000.00
dbaratbateladot@gmail.com
d) That they be liquidated and
demandable.
61. Dina bought a car from Jai and
delivered a check in payment of the same.
63. Which of the following statements
Has Dina paid the obligation? Why?
is correct?
a) No, not yet. The delivery of
a) All contracts are perfected by
promissory
mere consent.
notes
payable
to
order, or bills of exchange or
b) All contracts are perfected by
other mercantile documents shall
delivery of the object.
produce the effect of payment
c) All contracts are required to be in
only when they have been cashed,
writing.
or when through the fault of the
d) All contracts are required to
creditor they have been impaired.
have a valid consideration.
b) Yes, because a check is a valid
legal tender of payment.

64. It is a principle which holds that parties


c) It depends. If the check is a
are bound not only by what has been
managers check or cashiers check
expressly provided for in the contract but
it will produce the effect of payment.
also to the natural consequences that flow
If
out of such agreement.
its
an
ordinary
check,
no
payment.
a) Obligatory force of contracts
d) Yes, because a check is as good
b) Mutuality of contracts
as cash.
c) Autonomy of contracts
62. The following are the requisites of legal
d) Relativity of contracts
compensation, except:
a) That each of the obligors is bound
65. It is a principle which holds that
principally and that he be the same
contracts must be binding to both parties
time a principal creditor of the
and its validity and effectivity can never be
other.

left to the will of one of the parties.


b) That both debts consist in a sum
a) Obligatory force of contracts
of money, or if the things due are
b) Mutuality of contracts
consumable, they be the same kind,
c) Autonomy of contracts
and also of the same quality if the
d) Relativity of contracts
latter has been stated.
c) That the two (2) debts are not
66. It refers to the rule that a contract
yet due.
is binding
not
only between
Never Let The Odds Keep You From Pursuing What You Know In Your Heart You Were Me
ant To Do.-Leroy Satchel Paige
Page 143 of 180

Civil Law Q&As (2007-2013)


hectorchristopher@yahoo.com
dbaratbateladot@gmail.com
parties but extends to the heirs, successors
b) Acceptance of the offer by the
in
offeree.
interest,
and
assignees
of
the
parties, provided that the contract involved
c) Qualified/conditional acceptance
transmissible rights by their nature, or by
of the offer, which becomes counterstipulation or by law.
offer.
a) Obligatory force of contracts
d)
Subject
matter
becomes
b) Mutuality of contracts
illegal/impossible before acceptance
c) Autonomy of contracts
is communicated.
d) Relativity of contracts
70. Which of the following statements
67. It is rule which holds that the freedom
is correct?

of the parties to contract includes the


a) Offers in interrelated contracts
freedom
are perfected upon consent.
to
stipulate,
provided
the
stipulations are not contrary to law, morals,
b) Offers in interrelated contracts
good customs, public order or public policy.
require a single acceptance.
a) Obligatory force of contracts
c) Business
b) Mutuality of contracts
are definite
c) Autonomy of contracts
specific acceptance.
d) Relativity of contracts
d) Advertisements
are only
advertisements
offers
that
for
invitations
require
Bidders
to
make
68. The following are the ways by which
proposals and the advertiser is

innominate
not
contracts
are
regulated, except:
By
the
to
accept
the
highest/lowest bidder, unless it
a) By the stipulation of the parties.
b)
bound
appears otherwise.
general principles of
quasi-contracts and delicts
71.
c) By the rules governing the most
contracts (Contracts which must appear in
analogous nominate contracts.
writing), except:
d) By the customs of the place.
The
following
a) Donations of real estate or of
69. An offer becomes ineffective on any of
movables
the following grounds, except:
exceeds P 5,000.00.
a)

Death,
civil
interdiction,
are solemn
if
the
value
b) Stipulation to pay interest in
insanity/insolvency of either party
loans.
before acceptance is conveyed.
c) Sale of land through an agent
(authority must be in writing).
Never Let The Odds Keep You From Pursuing What You Know In Your Heart You Were Me
ant To Do.-Leroy Satchel Paige
Page 144 of 180

Civil Law Q&As (2007-2013)


d)
hectorchristopher@yahoo.com
Construction
contract
of
a
building.
dbaratbateladot@gmail.com
74. The following are the characteristics of
a voidable contract, except:
a) Effective until set aside.
72.
The
following
are
rescissible
b) May be assailed/attacked only in
contracts, except:
a)
an action for that purpose.
Entered
into
by
guardian
c) Can be confirmed or ratified.
whenever ward suffers damage more
d) Can be assailed only by either
than  of value of property.
party.
b) Agreed upon in representation of
absentees, if absentee suffers lesion

75.
by more than  of value of property.
contracts, except:
c)
Contracts
committed
on
where
fraud
creditor
is
following
are
void
a) Pactum commissorium
(accion
b) Pactum de non alienando
pauliana).
d)
The
c) Pactum leonina
Contracts
entered
into
by
d) Pacto de retro
minors.
76. The borrower in a contract of loan or
73. The following are the requisites before a
mutuum must pay interest to the lender.
contract entered into in fraud of creditors
a) If there is an agreement in

may be rescinded, except:


writing to the effect.
a) There must be credited existing
b) As a matter of course.
prior
c) If the amount borrowed is very
to
the
celebration
of
the
contract.
large.
b) There must be fraud, or at least,
d) If the lender so demands at the
the intent to commit fraud to the
maturity date.
prejudice of the creditor seeking
rescission.
77.
The
liability
c) The creditor cannot in any legal
administrators
manner collect his credit (subsidiary
individual, entity or institution engaged in
character of rescission)
child care over the minor child or damage
d) The object of the contract must
caused by the acts or omissions of the

be legally in the possession of a


unemancipated minor while under their
3rd person in good faith.
supervision, instruction or custody shall be:
and
of
the
school,
teachers,
or
its
the
a) Joint and subsidiary
b) Principal and solidary
Never Let The Odds Keep You From Pursuing What You Know In Your Heart You Were Me
ant To Do.-Leroy Satchel Paige
Page 145 of 180

Civil Law Q&As (2007-2013)


hectorchristopher@yahoo.com
dbaratbateladot@gmail.com
c) Principal and joint
b) quasi-contract
d) Subsidiary and solidary.
c) civil
d) natural
78. The creditor has the right to the fruits
of the thing from the time:
83.
Consent
was
of
given
a) the thing is delivered.
representation
b) the obligation to deliver the
authority. The contract is:
things arises.
a) voidable
c) the contract is perfected.
b) rescissible
d) the fruits are delivered.
c) void
another
by
one
in
but
without
d) unenforceable

79. If one of the parties to the contract is


84. Michael Fermin, without the authority
without juridical capacity, the contract is:
of Pascual Lacas, owner of a car, sold the
a) voidable
same car in the name of Mr. Lacas to Atty.
b) rescissible
Buko. The contract between Atty. Buko and
c) void
Mr. Lacas is --d) unenforceable
a) void because of the absence of
consent from the owner, Mr. Lacas.
80. When both parties to the contract are
b) valid because all of the essential
minors, the contract is:
requisites of a contract are present.
a) voidable
c) unenforceable because Michael
b) rescissible
Fermin had no authority but he
c) void
sold the car in the name of Mr.
d) unenforceable
Lacas, the owner.
d) rescissible because the contract
81. When the consent of one of the parties
caused lesion to Atty. Buko.
was vitiated, the contract is:
a) voidable
85. Which of the following contracts is void?
b) rescissible

a) An oral sale of a parcel of land.


c) void
b) A sale of land by an agent in a
d) unenforceable
public
instrument
where
his
authority from the principal is
82. An obligation which is based on equity
oral.
and natural law is known as:
c) A donation of a wrist watch
a) pure
worth P 4,500.00.
Never Let The Odds Keep You From Pursuing What You Know In Your Heart You Were Me
ant To Do.-Leroy Satchel Paige
Page 146 of 180

Civil Law Q&As (2007-2013)


hectorchristopher@yahoo.com
d) A relatively simulated contract
dbaratbateladot@gmail.com
for fulfillment of the obligation even
if he has not tendered payment of
86. Which of the following expresses a
the purchase price.
correct principle of law? Choose the best
c) The contract between the parties
answer.
is rescissible.
a) Failure to disclose facts when
d)
The
contract
between
the
there is a duty to reveal them, does
parties is subject to ratification
not constitute fraud.
by the parties.
b) Violence or intimidation does not
render
a
contract
annullable
if
employed not by a contracting party
88. Which of the following statements
is wrong?
but by a third person.

a) Creditors are protected in cases of


c) A threat to enforce ones claim
contracts intended to defraud them.
through competent authority, if the
b)
claim is legal or just, does not vitiate
between the parties, their assign
consent.
and heirs, except in case where the
d)
Absolute
simulation
of
Contracts
take
effect
only
a
rights and obligations arising from
contract always results in a void
the contract are not transmissible
contract.
by their nature, or by stipulation or
by provision of law.
87. Aligada orally offered to sell his twoc) If a contract should contain some
hectare rice land to Balane for P 10Million.
stipulation
The
person,

offer
was
orally
accepted.
By
in
he
favor
may
of
a
third
demand
agreement, the land was to be delivered
fulfillment
(through execution of a notarized Deed of
communicated his acceptance to the
Sale) and the price was to be paid exactly
obligor before its revocation.
one-month
d)
from
their
oral
agreement.
Which statement is most accurate?
In
provided
its
contracts
he

creating
real
rights, third persons who come
a) If Aligada refuses to deliver the
into possession of the object of
land on the agreed date despite
the
payment by Balane, the latter may
thereby.
not
successfully
sue
contract
are
not
bound
Aligada
because the contract is oral.
89.
Which
phrase
most
accurately
b) If Aligada refused to deliver the
completes the statement Any third person
land, Balane may successfully sue
who induces another to violate his contract:
Never Let The Odds Keep You From Pursuing What You Know In Your Heart You Were Me
ant To Do.-Leroy Satchel Paige
Page 147 of 180

Civil Law Q&As (2007-2013)


hectorchristopher@yahoo.com
dbaratbateladot@gmail.com
a) shall be liable for damages only if
he is a party to the same contract.
93. The attestation clause contains the
b) shall be liable for damages to
following, except:
the other contracting party.
a) the number of pages used;
c) shall not be liable for damages to
b) that the testator signed or caused
the other contracting party.
another to sign the will and every
d) shall not be liable for damages if
page thereof in the presence of the
the parties are in pari delicto.
instrumental witnesses;
c) notary public;
90. The requisites of succession are as
d)
the
instrumental
witnesses
follows, except:
witnessed and signed the will and
a) Death of decedent
all the pages thereof in the presence
b) Transmissible estate
of the testator and one another.
c)

Existence
and
capacity
of
successor, designated by decedent
94.
The
following
are
the
formalities
or law
required in the execution of holographic
d) Payment of Taxes
will, except:
a) Entirely written;
91. The characteristics of succession are as
b) Dated;
follows, except:
c) Signed by testator himself
a) It is a legal contract.
b)
Only
property,
rights
d) Notarized by a notary public.
and
obligations to the extent of the value
95. The following are the grounds for
of the inheritance are transmitted.
disallowance of wills, except:
c) The transmission takes place only

a) The formalities required by law


at the time of death.
have not been complied with.
d) The transmission takes place
b)
either by will or by operation of law.
mentally incapable of making will.
The
testator
was
insane
or
c) The will was executed through
92. The following rights are extinguished by
force or under duress, or influence
death, except:
of fear or threats.
a) Legal support
d) The will contains an attestation
b) Parental authority
clause.
c) Right to inherit
d) Agency
Never Let The Odds Keep You From Pursuing What You Know In Your Heart You Were Me
ant To Do.-Leroy Satchel Paige
Page 148 of 180

Civil Law Q&As (2007-2013)


hectorchristopher@yahoo.com
dbaratbateladot@gmail.com
96. It is the omission in the testators will of
consideration, which gives him the right to
one, some or all of the compulsory heirs in
buy
direct line, whether living at the time of
property, from another person, at anytime
execution of the will or born after the death
within the agreed period, at a fixed price.
of the testator. What principle is being
What contract is being referred to?
referred to?
certain
merchandise
or
specified
a) Option Contract
a) reserva troncal
b) Contract to Sell
b) preterition
c) Contract of Sale
c) fideicommissary
d) Lease
d) disposicion captatoria
97.
Any
disposition
made
upon

the
condition that the heir shall make some
100. Which of the following contracts of
sale is void?
provision in his will in favor of the testator
a) Sale of EGMs car by KRP, EGMs
or of any other person shall be void. Here,
agent,
both the condition and the disposition are
reduced into writing.
void. What principle is being referred to?
b) Sale of EGMs piece of land by
whose
authority
EGMs
not
a) reserva troncal
KRP,
b) preterition
authority
c) fideicommissary
writing.
d) disposicion captatoria
c) Sale of EGMs car by KRP, a
is
agent,
is
not
whose
reduced

into
person stranger to EGM, without
98.
Which
phrase
most
accurately
EGMs consent or authority.
completes the statement If at the time the
d) Sale of EGMs piece of land by
contract of sale is perfected, the thing
KRP, a person stranger to EGM,
which is the object of the contract has been
without EGMs consent or authority.
entirely lost:
a) the buyer bears the risk of loss.
b) the contract shall be without
any effect.
2011 Taxation Law Exam
MCQ (November 13, 2011)
c) the seller bears the risk of loss.
d) the buyer may withdraw from the
contract.
(1)When does a declaration of absence of a
missing person take effect?
(A) Immediately from the issuance of
99. A contract granting a privilege to a
person,
for
which
he
has
paid
the declaration of absence.
a

Never Let The Odds Keep You From Pursuing What You Know In Your Heart You Were Me
ant To Do.-Leroy Satchel Paige
Page 149 of 180

Civil Law Q&As (2007-2013)


hectorchristopher@yahoo.com
dbaratbateladot@gmail.com
(B) 3 months after the publication of
accepted the substitution. Later, however,
the declaration of absence.
the
(C) 6 months after the publication
defaulted in his obligation. What is the
of the declaration of absence.
effect of the new debtors default upon the
(D) 15 days from the issuance of the
original debtor?
new
declaration of absence.
debtor
became insolvent
and
(A) The original debtor is freed of
liability since novation took place
(2) The authority that school administrators
and
exercise over school children under their
obligation.
supervision,
(B) The original debtor shall pay or
instruction,
or
custody
is
called

this
relieved
him
of
his
perform the obligation with recourse
(A) legal parental authority.
to the new debtor.
(B) substitute parental authority.
(C)
(C) ordinary parental authority.
liable since he gave no consent to
(D) special parental authority.
the substitution.
The
original
debtor
remains
(D) The original debtor shall pay or
(3) Can future inheritance be the subject of
perform 50% of the obligation to
a contract of sale?
avoid unjust enrichment on his
(A)
No,
since
it
will
put
the
part.

predecessor at the risk of harm from


a tempted buyer, contrary to public
(5) Lennie bought a business class ticket
policy.
from Alta Airlines. As she checked in, the
(B) Yes, since the death of the
manager downgraded her to economy on
decedent is certain to occur.
the ground that a Congressman had to be
(C) No, since the seller owns no
accommodated
inheritance while his predecessor
Lennie
lives.
embarrassment of the downgrade. She sued
(D) Yes, but on the condition that
the airlines for quasi-delict but Alta Airlines
the amount of the inheritance can
countered
only
governed by a contract between them, no
be
ascertained
after
the
in
the
suffered
obligations of the estate have been
quasi-delict

paid.
correct?
that,
business
the
discomfort
since
could
class.
her
arise.
Is
and
travel
the
was
airline
(4) Upon the proposal of a third person, a
(A) No, the breach of contract may
new debtor substituted the original debtor
in fact be tortious as when it is
without the latters consent. The creditor
tainted
as
in
this
case
with
Never Let The Odds Keep You From Pursuing What You Know In Your Heart You Were Me
ant To Do.-Leroy Satchel Paige
Page 150 of 180

Civil Law Q&As (2007-2013)


hectorchristopher@yahoo.com
dbaratbateladot@gmail.com
arbitrariness, gross bad faith, and
(A) No, since the marriage did not
malice.
take place.
(B) No, denying Lennie the comfort
(B) Yes, since all the requisites of a
and amenities of the business class
donation
as provided in the ticket is a tortious
present.
act.
(C) No, since the donation and its
(C) Yes, since the facts show a
acceptance are not in a public
breach of contract, not a quasiinstrument.
delict.
(D) Yes, since X freely donated the
(D)
Yes,
since
quasi-delict
of
an
immovable
are
property to Y who became its owner.

presupposes the absence of a preexisting


contractual
relation
between the parties.
(8) Rene and Lily got married after a brief
courtship. After one month, Lily discovered
that while Rene presented himself as a
(6)
Which
of
the
following
is
an
macho man he was actually gay. He would
indispensable requirement in an action for
not go to bed with her. He kept obscene
"quieting of title" involving real property?
magazines of nude men and always sought
The plaintiff must
the company of handsome boys. What legal
(A) be in actual possession of the
remedy does Lily have?
property.
(A) She can file an action for
(B) be the registered owner of the
annulment of marriage on ground
property.
of fraud.
(C) have legal or equitable title to
(B) She can seek a declaration of

the property.
nullity of the marriage based on
(D) be the beneficial owner of the
Renes psychological incapacity.
property.
(C) She can go abroad and file for
divorce in a country that can grant
(7) X and Y were to marry in 3 months.
it.
Meantime,
X
(D) She has none since she had the
donated a house and lot to Y, which
opportunity to examine the goods
donation X wrote in a letter to Y. Y wrote
and freely entered into the marriage.
to
express
his
affection,
back, accepting the donation and took
possession of the property. Before the
(9)
Lucio
executed
a
simple
deed
of
wedding, however, Y suddenly died of heart
donation of P50 million on time deposit

attack. Can Ys heirs get the property?


with a bank in favor of A, B, C, D, and E,
Never Let The Odds Keep You From Pursuing What You Know In Your Heart You Were Me
ant To Do.-Leroy Satchel Paige
Page 151 of 180

Civil Law Q&As (2007-2013)


hectorchristopher@yahoo.com
dbaratbateladot@gmail.com
without indicating the share of each donee.
(D)
No,
since
All the donees accepted the donation in
impediment
writing. A, one of the donees, died. Will B,
inheritance to a stranger.
to
there
Raul
was
no
selling
his
C, D, and E get As share in the money?
(A) Yes, accretion will automatically
(11) When one exercises a right recognized
apply to the joint-donees in equal
by law, knowing that he thereby causes an
shares.
injustice to another, the latter is entitled to
(B) Yes, since the donors intention
recover damages. This is known as the
is to give the whole of P50 million to
principle of
the jointdonees in equal shares.
(A) res ipsa loquitur.

(C) No, A"s share will revert to the


(B) damnum absque injuria.
donor
(C) vicarious liability.
because
accretion
applies
only if the joint-donees are spouses.
(D) abuse of rights.
(D) No, As share goes to his heirs
since
the
donation
did
not
provide for reversion to donor.
(12) Which of the following is NOT a basis
for rendering a disinheritance defective or
imperfect?
(10) Raul, Ester, and Rufus inherited a 10(A) Its cause comes from the guilt
hectare land from their father. Before the
of a spouse in a legal separation
land could be partitioned, however, Raul
case, the innocent-spouse having
sold his hereditary right to Raffy, a stranger
died.
to the family, for P5 million. Do Ester and
(B) The truth of its cause is denied
Rufus have a remedy for keeping the land
and
within their family?

evidence.
not
sufficiently
proved
by
(A) Yes, they may be subrogated
(C) Its cause is not authorized by the
to Raffys right by reimbursing to
law.
him
(D) Its cause is not specified.
within
the
required
time
what he paid Raul.
(B) Yes, they may be subrogated to
(13) Manuel came to Manila and married
Raffys right provided they buy him
Marianne. Unknown to Marianne, Manuel
out before he registers the sale.
had been previously convicted in Palawan
(C) No, they can be subrogated to
of theft and served time for it. After
Raffys
Marianne
right
conformity.
only
with

his
learned
of
his
previous
conviction, she stopped living with him.
Can Marianne seek the annulment of the
Never Let The Odds Keep You From Pursuing What You Know In Your Heart You Were Me
ant To Do.-Leroy Satchel Paige
Page 152 of 180

Civil Law Q&As (2007-2013)


hectorchristopher@yahoo.com
dbaratbateladot@gmail.com
marriage based on Manuels nondisclosure
since
Philippine
of his previous crime?
recognize divorce.
law
does
not
(A) No, since the assumption is that
(D) All the children are legitimate
marriage forgives all past wrongs.
since they were born of the same
(B) Yes, since the non-disclosure
father and mother.
of that crime is the equivalent of
fraud,
which
is
a
ground
for
(15) Who can make a donation?
annulment.
(A) All persons who can enter into
(C) No, in case of doubt, the law
contracts and dispose of their
must be construed to preserve the
property.

institution of marriage.
(B) All persons who are of legal age
(D) No, since Manuel already served
and suffer from no civil interdiction.
the penalty for his crime.
(C) All persons who can make a last
will and testament.
(14) Arthur and Helen, both Filipinos, got
(D) All persons, whether natural or
married and had 2 children. Arthur later
artificial, who own property.
worked in Rome where he acquired Italian
citizenship. He got a divorce from Helen in
(16) The liability of the partners, including
Rome but, on returning to the Philippines,
industrial
he realized his mistake, asked forgiveness
contracts entered into in its name and for
of his wife, and resumed living with her.
its account, when all partnership assets
They had 2 more children. What is the
have been exhausted is
status of their 4 children?
partners
for
partnership
(A) Pro-rata.
(A) The children born before the
(B) Joint.
divorce are legitimate but those
(C) Solidary.

born after it are not since Arthur


(D) Voluntary.
got the divorce when he had
ceased to be a Filipino.
(17) When can a missing person who left
(B) The divorce rendered illegitimate
someone to administer his property be
the children born before it since the
declared an absentee by the court? When
marriage that begot them had been
he has been missing for
nullified.
(A) 2 years from the receipt of the
(C) The children born before and
last news about him.
after the divorce are all legitimate
(B) 7 years from the receipt of the
last news about him.
Never Let The Odds Keep You From Pursuing What You Know In Your Heart You Were Me
ant To Do.-Leroy Satchel Paige
Page 153 of 180

Civil Law Q&As (2007-2013)


hectorchristopher@yahoo.com
dbaratbateladot@gmail.com
(C) 10 years from the receipt of the
principal two days after the principal died,
last news about him.
an event that neither the agent nor the
(D) 5 years from the receipt of the
buyer knew at the time of the sale. What is
last news about him.
the standing of the sale?
(A) Voidable.
(18) Which of the following claims against
(B) Valid.
the debtor enjoys preference over the others
(C) Void.
with respect to his specific immovable
(D) Unenforceable.
property and real rights?
(A) Unpaid price of real property
(21) Spouses A and B leased a piece of land
sold, upon the immovable property.
belonging to B s parents for 25 years. The
(B) Mortgage credits recorded in the
spouses built their house on it worth
registry
P300,000.00. Subsequently, in a case that
of
property,
upon
the

mortgaged real estate.


C filed against A and B, the court found the
(C) Taxes due, upon the land or
latter liable to C for P200,000.00. When the
building.
sheriff was attaching their house for the
(D) Expenses for the preservation
satisfaction of the judgment, A and B
and improvement of property, when
claimed that it was exempt from execution,
the law authorizes reimbursement,
being a family home. Is this claim correct?
upon the preserved or improved
(A) Yes, because while Bs parents
immovable.
own the land, they agreed to have
their
daughter
build
her
family
(19) When bilateral contracts are vitiated
home on it.
with vices of consent, they are rendered
(B) No, because there is no judicial
(A) rescissible.
declaration that it is a family home.
(B) void.
(C) No, since the land does not
(C) unenforceable.
belong to A and B, it cannot

(D) voidable.
qualify as a family home.
(D) Yes, because the A and Bs
(20) An agent, authorized by a special
family actually lives in that house.
power of attorney to sell a land belonging to
the principal succeeded in selling the same
(22) Solomon sold his coconut plantation to
to a buyer according to the instructions
Aragon, Inc. for P100 million, payable in
given the agent. The agent executed the
installments of P10 million per month with
deed of absolute sale on behalf of his
6% interest per annum. Solomon married
Never Let The Odds Keep You From Pursuing What You Know In Your Heart You Were Me
ant To Do.-Leroy Satchel Paige
Page 154 of 180

Civil Law Q&As (2007-2013)


hectorchristopher@yahoo.com
dbaratbateladot@gmail.com
Lorna after 5 months and they chose
(24) When A and B married, they chose
conjugal partnership of gains to govern
conjugal partnership of gains to govern
their
they
their property relations. After 3 years, B
married, Aragon had an unpaid balance of
succeeded in getting her marriage to A
P50 million plus interest in Solomons
annulled
favor. To whom will Aragons monthly
psychological incapacity. What liquidation
payments go after the marriage?
procedure will they follow in disposing of
property
relations.
When
(A) The principal shall go to the
conjugal
of
the
latters
their assets?
will
interests to Solomon.
governing
the

(B) Both principal and interests


conjugal partnership of gains where
shall go to Solomon since they are
the party who acted in bad faith
his exclusive properties.
forfeits his share in the net profits.
(C) Both principal and interests
(B) Since the marriage has been
shall go to the conjugal partnership
declared
since these become due after the
liquidation of absolute community of
marriage.
property shall be followed.
principal
but
ground
They
(D) The
partnership
on
shall
go
the
(A)
to
(C) The
follow
the
liquidation
void,

the
liquidation
of
rule
of
applies
rule
a
since
a
for
coSolomon but the interests to the
ownership
the
conjugal partnership.
annulment brought their property
relation under the chapter on
(23) X and Y, although not suffering from
property
any impediment, cohabited as husband and
marriage.
wife
(D)
without
the
benefit
of
marriage.
Following the birth of their child, the couple

regimes
The
law
on
without
liquidation
of
partnerships applies.
got married. A year after, however, the
(25) X and Y agreed verbally before their
court annulled the marriage and issued a
marriage
decree of annulment. What is the present
illegitimate child of Y and (b) on the
status of the child?
economic regime that will govern X and Ys
(a)
on
the
paternity
of
the
(A) Legitimated.
property relations. Is the verbal agreement
(B) Illegitimate.
valid?
(C) Natural child.
(A) No,
because
a

marriage
(D) Legitimate.
settlement to be valid should be
in writing.
Never Let The Odds Keep You From Pursuing What You Know In Your Heart You Were Me
ant To Do.-Leroy Satchel Paige
Page 155 of 180

Civil Law Q&As (2007-2013)


(B)
Yes,
hectorchristopher@yahoo.com
since
ante-nuptial
dbaratbateladot@gmail.com
(B) Illegitimate, because by the color
agreements need not be in writing.
of its skin, the child could not
(C)
possibly be that of Fidel.
No,
settlement
because
cannot
a
marriage
include
an
(C) Legitimate, because the child
agreement on the paternity of an
was born within a valid marriage.
illegitimate child.
(D) Legitimate, because Fidel agreed
(D) Yes, since even if it is not a valid
to treat the child as his own after
marriage settlement, it is a valid
Gloria told him who the father was.
verbal contract.
(28) The husbands acts of forcibly ejecting
(26) Spouses X and Y have a minor

his wife without


daughter, Z, who needs support for her
conjugal dwelling and refusing to take her
education.
back constitutes
Both
X
and
Y,
who
are
just
cause from the
financially distressed, could not give the
(A) desertion.
needed support to Z. As it happens, Zs
(B) recrimination.
other relatives are financially capable of
(C) constructive abandonment.
giving that support. From whom may Z first
(D) de facto separation.
rightfully demand support? From her
(A) grandfather.
(29) In his will, the testator designated X as
(B) brother.
a legatee to receive P2 million for the
(C) uncle.
purpose of buying an ambulance that the
(D) first cousin.
residents of his Barangay can use. What
kind of institution is this?

(27) Fidel, a Filipino with fair complexion,


(A) a fideicomissary institution.
married Gloria. Before the marriage, Gloria
(B) a modal institution.
confessed to Fidel that she was two-month
(C) a conditional institution.
pregnant with the child of a black African
(D) a collective institution.
who had left the country for good. When the
child was born, Fidel could not accept it
(30) X insured himself for P5 million,
being too black in complexion. What is the
designating
status of the child?
beneficiary.
(A)
Illegitimate,
because
Y,
his
The
wife,
as
his
designation
sole
was
Gloria
irrevocable. A few years later, X had their
confessed that the child is not
marriage annulled in court on the ground

Fidels.
that Y had an existing prior marriage. X
Never Let The Odds Keep You From Pursuing What You Know In Your Heart You Were Me
ant To Do.-Leroy Satchel Paige
Page 156 of 180

Civil Law Q&As (2007-2013)


hectorchristopher@yahoo.com
dbaratbateladot@gmail.com
subsequently died, Is Y entitled to the
(C) Marlon gets 1/2 and Cecilia
insurance benefits?
gets 1/2.
(A) Yes, since the insurance was not
(D) Marlon gets 3/4 and Cecilia 1/4.
dependent on the marriage.
(B) Yes, since her designation as
(33) Contracts take effect only between the
beneficiary was irrevocable.
parties or their assigns and heirs, except
(C) No, Xs designation of Y is
where the rights and obligations arising
revoked by operation of law upon
from the contract are not transmissible by
the annulment of their marriage
their nature, by stipulation, or by provision
based on Ys fault.
of law. In the latter case, the assigns or the
(D)
Yes,
since
revocation,
Xs
without
judicial
designation
of

Y
remains valid and binding.
heirs are not bound by the contracts. This
is known as the principle of
(A) Relativity of contracts.
(31) May a spouse freely donate communal
(B) Freedom to stipulate.
or conjugal property without the consent of
(C) Mutuality of contracts.
the other?
(D) Obligatory force of contracts.
(A) Absolutely not, since the spouses
co-own such property.
(34) A buyer ordered 5,000 apples from the
(B) Yes, for properties that the
seller at P20 per apple. The seller delivered
family
6,000 apples. What are the rights and
may
spare,
regardless
of
value.
obligations of the buyer?
(C) Yes, provided the donation is
(A) He can accept all 6,000 apples
moderate
and pay the seller at P20 per
and
intended
for

charity or family rejoicing.


apple.
(D) Yes, in a donation mortis causa
(B) He can accept all 6,000 apples
that the donor may still revoke in
and pay a lesser price for the 1,000
his lifetime.
excess apples.
(C) He can keep the 6,000 apples
(32) The decedent died intestate leaving an
without paying for the 1,000 excess
estate of P10 million. He left the following
since
heirs: a) Marlon, a legitimate child and b)
anyway.
Cecilia, the legal spouse. Divide the estate.
(D)
the
He
seller
can
delivered
cancel
the
them
whole
(A) Marlon gets 1/4 and Cecilia gets
transaction since the seller violated
3/4.
the terms of their agreement.
(B) Marlon gets 2/3 and Cecilia 1/3.
Never Let The Odds Keep You From Pursuing What You Know In Your Heart You Were Me

ant To Do.-Leroy Satchel Paige


Page 157 of 180

Civil Law Q&As (2007-2013)


hectorchristopher@yahoo.com
from
dbaratbateladot@gmail.com
(35) Lino entered into a contract to sell with
suffered
poisoning
caused
by
a
Ramon, undertaking to convey to the latter
noxious substance found in the sardines.
one of the five lots he owns, without
Mylene filed a case for damages against
specifying which lot it was, for the price of
Acme. Which of the following defenses will
P1 million. Later, the parties could not
hold?
agree which of five lots he owned Lino
(A) The expiry date of the "Sards"
undertook to sell to Ramon. What is the
was clearly printed on its can,
standing of the contract?
still the store sold and Mylene
(A) Unenforceable.
bought it.
(B) Voidable.
(B) Mylene must have detected the
(C) Rescissible.
noxious substance in the sardines

(D) Void.
by smell, yet she still ate it.
(C) Acme had no transaction with
(36) Knowing that the car had a hidden
Mylene; she bought the "Sards" from
crack in the engine, X sold it to Y without
a store, not directly from Acme.
informing the latter about it. In any event,
(D) Acme enjoys the presumption of
the deed of sale expressly stipulated that X
safeness of its canning procedure
was not liable for hidden defects. Does Y
and Mylene has not overcome such
have the
presumption.
right
to
demand
from X
a
reimbursement of what he spent to repair
the engine plus damages?
(38) Fernando executed a will, prohibiting
(A) Yes. X is liable whether or not he
his wife Marina from remarrying after his
was aware of the hidden defect.
death, at the pain of the legacy of P100
(B) Yes, since the defect was not
Million in her favor becoming a nullity. But
hidden; X knew of it but he acted
a year after Fernandos death, Marina was

in bad faith in not disclosing the


so overwhelmed with love that she married
fact to Y.
another man. Is she entitled to the legacy,
(C) No, because Y is in estoppel,
the amount of which is well within the
having changed engine without prior
capacity of the disposable free portion of
demand.
Fernandos estate?
(D)
No,
because
Y
waived
the
warranty against hidden defects.
(A) Yes, since the prohibition against
remarrying is absolute, it is deemed
(37) Acme Cannery produced sardines in
not written.
cans known as "Sards." Mylene bought a
(B) Yes, because the prohibition is
can of Sards from a store, ate it, and
inhuman
and
oppressive
and
Never Let The Odds Keep You From Pursuing What You Know In Your Heart You Were Me
ant To Do.-Leroy Satchel Paige
Page 158 of 180

Civil Law Q&As (2007-2013)


hectorchristopher@yahoo.com
dbaratbateladot@gmail.com
violates Marinas rights as a free
(A) It
is
an
ordinary
woman.
since it was not given to the bride
(C) No, because the nullity of the
or groom.
prohibition also nullifies the legacy.
(B) It is donation propter nuptias
(D) No, since such prohibition is
since it was given with the marriage
authorized by law and is not
in mind.
repressive; she could remarry but
(C) It is an indirect donation propter
must give up the money.
nuptias
since
the
donation
bride
would
eventually inherit the property from
(39) X, the owner, constituted a 10-year
her parents.
usufruct on his land as well as on the

(D) It is a remunatory donation.


building standing on it in Ys favor. After
flood totally destroyed the building 5 years
(41) X and Y, both Filipinos, were married
later, X told Y that an act of God terminated
and resided in Spain although they intend
the usufruct and that he should vacate the
to return to the Philippines at some future
land. Is X, the owner of the land, correct?
time. They have not executed any marriage
(A)
No,
since
the
building
was
destroyed through no fault of Y.
settlements.
What
law
governs
their
property relations?
(B) No, since Y still has the right
(A)
to use the land and the materials
Spanish law and Philippine law.
left on it.
(B) Philippine law since they are
(C) Yes, since Y cannot use the land

both Filipinos.
without the building.
(C) No regime of property relations
(D) Yes, since the destruction of the
will apply to them.
building
(D) Spanish law since they live in
without
the
Xs
fault
terminated the usufruct.
They
may
choose
between
Spain.
(40) In gratitude, the grooms parents made
(42) Birth determines personality. Death
a donation of a property in writing to the
extinguishes it. Under what circumstances
brides
may the personality of a deceased person
parents
shortly
childrens
wedding.
The
accepted.
What

the
donation?
is
before
their
donation
was
nature
of
the
continue to exist?
(A) In case of re-appearance of a
missing person presumed dead.
Never Let The Odds Keep You From Pursuing What You Know In Your Heart You Were Me
ant To Do.-Leroy Satchel Paige
Page 159 of 180

Civil Law Q&As (2007-2013)


hectorchristopher@yahoo.com
dbaratbateladot@gmail.com
(B) In protecting the works of a
(C) the brother or sister in need
deceased under intellectual property
stops
laws.
reason.
(C)
In
case
of
presumptive death
declaration
of
of
a missing
spouse.
schooling
(D) the
need for
without
valid
support
of
a
brother or sister, already of age, is
due to the latter s fault.
(D) In the settlement of the estate
of a deceased person.
(45) Virgilio owned a bare and simple

swimming pool in his garden. MB, a 7-year


(43) Six tenants sued X, the landowner, for
old child, surreptitiously entered the garden
willfully denying them water for their farms,
and merrily romped around the ledges of
which water happened to flow from land
the pool. He accidentally tripped, fell into
under Xs control, his intention being to
the pool, and drowned. MBs parents sued
force them to leave his properties. Is X
Virgilio for damages arising from their
liable for his act and why?
childs death, premised on the principle of
(A) No, because the tenants must be
"attractive nuisance". Is Virgilio liable for
content with waiting for rainfall for
the death of MB?
their farms.
(A) No, the child was 7 years old and
(B) No, since X owns both the land
knew the dangers that the pool
and the water.
offered.
(C) Yes, because the tenants farms
(B)
have the natural right of access to
nuisance, Virgilio had the duty to
water wherever it is located.
prevent children from coming near
(D) Yes, since X willfully caused

it.
injury to his tenants contrary to
(C) No, since the pool was bare
morals, good customs or public
and had no enticing or alluring
policy.
gadgets, floats, or devices in it
Yes,
being
an
attractive
that would attract a 7-year old
(44)
Illegitimate
brothers
and
sisters,
child.
whether of full or half-blood, are bound to
(D) Yes, since Virgilio did not cover
support each other, EXCEPT when
the swimming pool while not in use
(A) the brother or sister who needs
to prevent children from falling into
support lives in another place.
it.
(B) such brothers and sisters are not
recognized by their father.
Never Let The Odds Keep You From Pursuing What You Know In Your Heart You Were Me
ant To Do.-Leroy Satchel Paige
Page 160 of 180

Civil Law Q&As (2007-2013)


hectorchristopher@yahoo.com
dbaratbateladot@gmail.com
(46) The term of a 5-year lease contract
(D) Yes, the donation is not deemed
between X the lessor and Y the lessee,
made until the suspensive condition
where rents were paid from month to
has been fulfilled.
month, came to an end. Still, Y continued
using the property with Xs consent. In
(48)
Illegitimate
children,
those
not
such a case, it is understood that they
recognized by their biological fathers, shall
impliedly renewed the lease
use the surname of their
(A) from month to month under
(A) biological father subject to no
the same conditions as to the
condition.
rest.
(B) mother or biological father, at
(B)
under
the same
terms and
the mothers discretion.

conditions as before.
(C) mother.
(C) under the same terms except the
(D)
rent which they or the court must
judicially opposes it.
biological
father
unless
he
fix.
(D) for only a year, with the rent
(49) Asiong borrowed P1 million from a
raised by 10% pursuant to the
bank, secured by a mortgage on his land.
rental control law.
Without his consent, his friend Boyong paid
the whole loan. Since Asiong benefited from
(47)
a
the payment, can Boyong compel the bank
valuable lot to the municipality on the
to subrogate him in its right as mortgagee
condition that it will build a public school
of Asiong s land?
on
Rex,
such
acceptance
a
philanthropist,

lot
within
of
the
donated
2 years from
its
donation.
The
(A) No, but the bank can foreclose
and pay Boyong back.
municipality properly accepted the donation
(B) No,
but did not yet build the public school after
Asiongs
2 years. Can Rex revoke the donation?
approval.
(A) Yes,
since
the
donation
since
Boyong
loan
paid
without
for
his
is
(C) Yes, since a change of creditor
subject to a resolutory condition

took place by novation with the


which was not fulfilled.
banks consent.
(B) No, but Rex is entitled to recover
(D) Yes, since it is but right that
the value of the land from the
Boyong be able to get back his
municipality.
money and, if not, to foreclose the
(C) No, the transfer of ownership has
mortgage in the manner of the bank.
been completed.
Never Let The Odds Keep You From Pursuing What You Know In Your Heart You Were Me
ant To Do.-Leroy Satchel Paige
Page 161 of 180

Civil Law Q&As (2007-2013)


hectorchristopher@yahoo.com
dbaratbateladot@gmail.com
(50) Congress passed a law imposing taxes
(D) Yes, since Fernando was a
on income earned out of a particular
solidary creditor, payment to him
activity that was not previously taxed. The
extinguished the obligation.
law, however, taxed incomes already earned
within the fiscal year when the law took
(52) What happens to the property regimes
effect. Is the law valid?
that were subsisting under the New Civil
(A) No, because laws are intended to
Code when the Family Code took effect?
be prospective, not retroactive.
(A) The original property regimes
(B) No, the law is arbitrary in that it
are
taxes income that has already been
effective.
spent.
(B) Those enjoying specific regimes
(C) Yes, since tax laws are the
under the New Civil Code may adopt
lifeblood of the nation.
the regime of absolute community of
(D) Yes, tax laws are an exception;
property under the Family Code.
they

(C) Those that married under the


can
be
given
retroactive
effect.
immutable
and
remain
New Civil Code but did not choose
any of its regimes shall now be
(51)
Rudolf
Rodrigo
borrowed
and
Fernando
P1
million
who
from
acted
as
governed by the regime of absolute
community of property.
solidary creditors. When the loan matured,
(D) They are superseded by the
Rodrigo wrote a letter to Rudolf, demanding
Family Code which has retroactive
payment of the loan directly to him. Before
effect.

Rudolf could comply, Fernando went to see


(53) The testator executed a will following
him personally to collect and he paid him.
the formalities required by the law on
Did Rudolf make a valid payment?
succession without designating any heir.
(A) No, since Rudolf should have
The only testamentary disposition in the
split the payment between Rodrigo
will is the recognition of the testator s
and Fernando.
illegitimate child with a popular actress. Is
(B) No, since Rodrigo, the other
the will valid?
solidary creditor, already made a
(A) Yes, since in recognizing his
prior demand for payment from
illegitimate child, the testator has
Rudolf.
made him his heir.
(C) Yes, since the payment covers
(B) No, because the non-designation
the whole obligation.
of heirs defeats the purpose of a will.
Never Let The Odds Keep You From Pursuing What You Know In Your Heart You Were Me
ant To Do.-Leroy Satchel Paige
Page 162 of 180

Civil Law Q&As (2007-2013)


hectorchristopher@yahoo.com
dbaratbateladot@gmail.com
(C) No, the will comes to life only
(C) Voidable, because the Judge
when
acted
the
proper
heirs
are
instituted.
(D) Yes,
beyond
his
territorial
jurisdiction and is administratively
the
illegitimate
recognition
heir
is
of
an
an
liable for the same.
ample
(D) Void, because the Judge did not
reason for a will.
solemnize the marriage within the
premises of his court.
(54) A left B, his wife, in the Philippines to

work in Egypt but died in that country after


(56) X and Y, Filipinos, got married in Los
a years continuous stay. Two months after
Angeles, USA, using a marriage license
As death, B gave birth to a child, claiming
issued by the Philippine consul in Los
it is As child. Who can assail the legitimacy
Angeles, acting as Civil Registrar. X and Y
of the child?
did not know that they were first cousins
(A) As other heirs apart from B.
because their mothers, who were sisters,
(B) The State which has interest in
were
the welfare of overseas contract
young. Since X did not want to continue
workers.
with the relation when he heard of it, he left
(C) Any one who is outraged by Bs
Y, came to the Philippines and married Z.
claim.
Can X be held liable for bigamy?
(D) No one since A died.
separated
when
they
were
quite
(A) No since Xs marriage to Y is void
ab initio or did not exist.
(55) QR and TS who had a marriage license

(B) No since X acted in good faith,


requested a newly appointed Judge in
conscious that public policy did not
Manila to marry them on the beach of
approve of marriage between first
Boracay.
cousins.
Since
the
Judge
maintained
Boracay as his residence, he agreed. The
(C) Yes since he married Z without
sponsors were all public officials. What is
first
the status of the marriage.
declaration
(A) Valid,
since
the
improper
securing
of
a
nullity
judicial
of
his
marriage to Y.
venue is merely an irregularity;
(D) Yes since his first marriage to Y

all
in Los Angeles is valid.
the
elements
of
a
valid
marriage are present.
(B) Void, because the couple did not
(57) Allan bought Billys property through
get local permit for a beach wedding.
Carlos, an agent empowered with a special
power of attorney (SPA) to sell the same.
Never Let The Odds Keep You From Pursuing What You Know In Your Heart You Were Me
ant To Do.-Leroy Satchel Paige
Page 163 of 180

Civil Law Q&As (2007-2013)


hectorchristopher@yahoo.com
dbaratbateladot@gmail.com
When Allan was ready to pay as scheduled,
(B) No, the buyer is entitled to a
Billy called, directing Allan to pay directly
customary 30-day extension of his
to him. On learning of this, Carlos, Billy s
obligation to take delivery of the
agent, told Allan to pay through him as his
goods.
SPA
(C) No, since there was no express
provided
and
to
protect
his
commission. Faced with two claimants,
agreement
Allan consigned the payment in court. Billy
rescission.
protested, contending that the consignation
(D)
is ineffective since no tender of payment
determine that Y was not justified in
was made to him. Is he correct?
failing to appear.
No,
regarding
the

seller
automatic
should
first
(A) No, since consignation without
(59) The wife filed a case of legal separation
tender of payment is allowed in
against her husband on the ground of
the face of the conflicting claims
sexual infidelity
on the plaintiff.
without previously exerting earnest efforts
(B) Yes, as owner of the property
to come to a compromise with him. The
sold, Billy can demand payment
judge dismissed the case for having been
directly to himself.
filed without complying with a condition
(C)
Yes,
since
Allan
made
no
precedent. Is the dismissal proper?
announcement of the tender.
(A) No, efforts at a compromise will
(D) Yes, a tender of payment is
only deepen the wifes anguish.
required for a valid consignation.

(B) No, since legal separation like


validity of marriage is not subject
(58) X sold Y 100 sacks of rice that Y was to
to
compromise
pick up from Xs rice mill on a particular
purposes of filing.
date. Y did not, however, appear on the
(C) Yes, to avoid a family feud that is
agreed date to take delivery of the rice. After
hurtful to everyone.
one week, X automatically rescinded the
(D) Yes, since the dispute could have
sale without notarial notice to Y. Is the
been
rescission valid?
agreeing to legal separation.
settled
agreement
with
the
for
parties
(A) Yes, automatic rescission is
allowed
since,
character
of
consumables,
deteriorate.
having
the

(60) An Australian living in the Philippines


movables
and
acquired shares of stock worth P10 million
easily
in food manufacturing companies. He died
rice
can
in Manila, leaving a legal wife and a child in
Australia and a live-in partner with whom
Never Let The Odds Keep You From Pursuing What You Know In Your Heart You Were Me
ant To Do.-Leroy Satchel Paige
Page 164 of 180

Civil Law Q&As (2007-2013)


hectorchristopher@yahoo.com
dbaratbateladot@gmail.com
he had two children in Manila. He also left
(D) Yes, after full payment, the
a will, done according to Philippine laws,
action became imprescriptible.
leaving all his properties to his live-in
partner and their children. What law will
(62) A court declared Ricardo, an old
govern the validity of the disposition in the
bachelor, an absentee and appointed Cicero
will?
administrator of his property. After a year,
(A) Australia law since his legal wife
it was discovered that Ricardo had died
and legitimate child are Australians
abroad. What is the effect of the fact of his
and domiciled in Australia.
death
(B) Australian
administration
of
his
property?
intrinsic validity of the provisions
(A)
of
absentee but a deceased person,
will
is

since
the
the
a
law
on
governed
by
the
With
Ricardo
no
longer
an
decedents national law.
Cicero will cease to be administrator
(C) Philippine law since the decedent
of his properties.
died in Manila and he executed his
(B) The administration shall be
will according to such law.
given
(D)
Philippine
law
since
by
the
court
over

the
the
jurisdiction
decedents properties are in the
proceedings
to
Philippines.
administrator
whom
having
intestate
a
new
it
will
appoint.
(61) X bought a land from Y, paying him
(C) Cicero automatically becomes
cash. Since they were friends, they did not
administrator
execute any document of sale. After 7 years,
until judicially relieved.
the heirs of X asked Y to execute a deed of
(D) Ciceros alienations of Ricardo s
absolute sale to formalize the verbal sale to
property will be set aside.
of
Ricardos
estate
their father. Unwilling to do so, Xs heirs
filed an action for specific performance
(63) Baldo, a rejected suitor, intimidated

against Y. Will their action prosper?


Judy into marrying him. While she wanted
(A) No, after more than 6 years,
to question the validity of their marriage
the action to enforce the verbal
two years after the intimidation ceased,
agreement has already elapsed.
Judy decided in the meantime to freely
(B) No, since the sale cannot under
cohabit with Baldo. After more than 5 years
the Statute of Frauds be enforced.
following their wedding, Judy wants to file a
(C) Yes, since X bought the land and
case for annulment of marriage against
paid Y for it.
Never Let The Odds Keep You From Pursuing What You Know In Your Heart You Were Me
ant To Do.-Leroy Satchel Paige
Page 165 of 180

Civil Law Q&As (2007-2013)


hectorchristopher@yahoo.com
dbaratbateladot@gmail.com
Baldo on ground of lack of consent. Will her
all heirs in the higher level are disqualified
action prosper?
or unable to inherit?
(A) Yes, the action for annulment is
(A) Nephews and nieces.
imprescriptible.
(B) Brothers and sisters.
(B) No, since the marriage was
(C) State.
merely voidable and Judy ratified
(D) Other collateral relatives up to
it by freely cohabiting with Baldo
the 5th degree of consanguinity.
after the force and intimidation
had ceased.
(66) Roy and Carlos both undertook a
(C) No, since the action prescribed 5
contract to deliver to Sam in Manila a boat
years
docked in Subic. Before they could deliver
from
the
date
of
the
celebration of the marriage.
it, however, the boat sank in a storm. The

(D) Yes, because the marriage was


contract provides that fortuitous event shall
celebrated without Judy s consent
not exempt Roy and Carlos from their
freely given.
obligation. Owing to the loss of the motor
boat, such obligation is deemed converted
(64) Is the wife who leaves her husband
into one of indemnity for damages. Is the
without just cause entitled to support?
liability of Roy and Carlos joint or solidary?
(A)
No,
because
must
(A) Neither solidary nor joint since
always be submissive and respectful
they cannot waive the defense of
to the husband.
fortuitous event to which they are
(B) Yes. The marriage not having
entitled.
been
(B)
dissolved,
the
wife
the
husband
Solidary

or
joint
upon
the
continues to have an obligation to
discretion of Sam.
support his wife.
(C) Solidary since Roy and Carlos
(C) No, because in
leaving the
failed to perform their obligation to
conjugal home without just cause,
deliver the motor boat.
she forfeits her right to support.
(D) Joint since the conversion of
(D) Yes, since the right to receive
their liability to one of indemnity
support
for damages made it joint.
is
not
subject
to
any
condition.
(67) Joanne married James, a person with
(65) In the order of intestate succession
no known relatives. Through James

hard

where the decedent is legitimate, who is the


work, he and his wife Joane prospered.
last intestate heirs or heir who will inherit if

When
James
died,
his
estate
alone
Never Let The Odds Keep You From Pursuing What You Know In Your Heart You Were Me
ant To Do.-Leroy Satchel Paige
Page 166 of 180

Civil Law Q&As (2007-2013)


hectorchristopher@yahoo.com
dbaratbateladot@gmail.com
amounted to P100 million. If, in his will,
(70) Ric and Josie, Filipinos, have been
James designates Joanne as his only heir,
sweethearts for 5 years. While working in a
what will be the free portion of his estate.
European country where the execution of
(A) Joanne gets all; estate has no
joint wills are allowed, the two of them
free portion left.
executed a joint holographic will where they
(B) Joanne gets 1/2; the other
named each other as sole heir of the other
half is free portion.
in case either of them dies. Unfortunately,
(C) Joanne gets 1/3; the remaining
Ric died a year later. Can Josie have the
2/3 is free portion.
joint will successfully
(D) Joanne gets 1/4; the remaining
Philippines?
3/4 is free portion.
probated
in the
(A) Yes, in the highest interest of
comity of nations and to honor the
(68) A warranty inherent in a contract of
wishes of the deceased.
sale, whether or not mentioned in it, is

(B) No,
known as the
prohibits the execution of joint
since
Philippine
law
(A) warranty on quality.
wills and such law is binding on
(B) warranty against hidden defects.
Ric and Josie even abroad.
(C) warranty against eviction.
(C) Yes, since they executed their
(D) warranty in merchantability.
joint will out of mutual love and
care,
values
that
the
generally
(69) The doctrine of stare decisis prescribes
accepted principles of international
adherence
law accepts.
to
precedents
in
order
to
promote the stability of the law. But the
(D) Yes, since it is valid in the

doctrine can be abandoned


country
where
it
was
executed,
(A) When adherence to it would
applying the principle of "lex loci
result in the Governments loss of its
celebrationis."
case.
(B) When the application of the
(71) ML inherited from his father P5 million
doctrine
in legitime but he waived it in a public
would
cause
great
prejudice to a foreign national.
instrument in favor of his sister QY who
(C) When necessary to promote the
accepted the waiver in writing. But as it
passage of a new law.
happened, ML borrowed P6 million from PF
(D) When
the
precedent
has
before the waiver. PF objected to the waiver
ceased to be beneficial and useful.
and filed an action for its rescission on the

ground that he had the right to MLs P5


Never Let The Odds Keep You From Pursuing What You Know In Your Heart You Were Me
ant To Do.-Leroy Satchel Paige
Page 167 of 180

Civil Law Q&As (2007-2013)


hectorchristopher@yahoo.com
dbaratbateladot@gmail.com
million legitime as partial settlement of
(B) Yes,
insofar
as
what ML owed him since ML has proved to
acknowledged
be insolvent. Does PF, as creditor, have the
illegitimate child.
right to rescind the waiver?
(C) None, since the marriage did not
Mary
Arnold
as
his
(A) No, because the waiver in favor
take place.
of his sister QY amounts to a
(D) Yes, if they acquired properties
donation and she already accepted
while living together as husband
it.
and wife.
(B) Yes,
because
the
waiver
is
(73) Joseph, a 17-year old Filipino, married

prejudicial to the interest of a


Jenny, a 21-year old American in Illinois,
third person whose interest is
USA, where the marriage was valid. Their
recognized by law.
parents gave full consent to the marriage of
(C) No, PF must wait for ML to
their children. After three years, Joseph
become solvent and, thereafter, sue
filed a petition in the USA to promptly
him for the unpaid loan.
divorce Jenny and this was granted. When
(D) Yes, because a legitime cannot
Joseph turned 25 years, he returned to the
be waived in favor of a specific heir;
Philippines and married Leonora. What is
it must be divided among all the
the status of this second marriage?
other heirs.
(A) Void, because he did not cause
the
judicial
issuance
of
(72) While engaged to be married, Arnold
declaration of the nullity of his
and
first marriage to Jenny before
Josephine
agreed

in
a
public
instrument to adopt out the economic
marrying Leonora.
regime of absolute community of property.
(B) Valid, because Joseph s marriage
Arnold
to Jenny is void, he being only 17
acknowledged
instrument
Mary,
is
that
the
Josephines
daughter
Josephine died before the marriage could
Leonora has all the elements of a
take place. Does the marriage settlement
valid marriage.
have any significance?
(D) Void, because Joseph is still
since
the
child.
years of age when he married her.
(C) Valid, because his marriage to
None,
illegitimate
same
But

(A)
his
in
instrument
considered married to Jenny since
containing the marriage settlement
the Philippines does not recognize
is essentially void for containing an
divorce.
unrelated matter.
Never Let The Odds Keep You From Pursuing What You Know In Your Heart You Were Me
ant To Do.-Leroy Satchel Paige
Page 168 of 180

Civil Law Q&As (2007-2013)


hectorchristopher@yahoo.com
(74) T died intestate, leaving an estate of
(76)
P9,000,000. He left as heirs three legitimate
brother, Y, authorizing him to sell Xs
children, namely, A, B, and C. A has two
parcel of land in Pasay. X sent the title to Y
children, D and E. Before he died, A
by courier service. Acting for his brother, Y
irrevocably repudiated his inheritance from
executed a notarized deed of absolute sale
T in a public instrument filed with the
of the land to Z after receiving payment.
court. How much, if any, will D and E, as
What is the status of the sale?
As children, get from Ts estate?
X,
who
dbaratbateladot@gmail.com
was
abroad,
phoned his
(A) Valid, since a notarized deed of
(A) Each of D and E will get
absolute
P1,500,000
by
transaction and full payment was
representation

since
right
their
of
father
sale
covered
the
made.
repudiated his inheritance.
(B) Void,
since
X
should
have
(B) Each of D and E will get
authorized agent Y in writing to
P2,225,000 because they will inherit
sell the land.
from the estate equally with B and
(C) Valid, since Y was truly his
C.
brother Xs agent and entrusted
(C) D and E will get none because
with the title needed to effect the
of the repudiation; "B" and "C"
sale.
will get As share by right of
(D) Valid, since the buyer could file
accretion.
an action to compel X to execute a

(D) Each of D and E will get


deed of sale.
P2,000,000 because the law gives
them some advantage due to the
(77) In a true pacto de retro sale, the title
demise of "A".
and ownership of the property sold are
immediately vested in the vendee a retro
(75) No decree of legal separation can be
subject only to the resolutory condition of
issued
repurchase by the vendor a retro within the
(A) unless the childrens welfare is
attended to first.
(B) without
prior
stipulated period. This is known as
(A) equitable mortgage.
efforts
at
(B) conventional redemption.
reconciliation shown to be futile.
(C) legal redemption.
(C) unless the court first directs
(D) equity of redemption.
mediation of the parties.
(78) A natural obligation under the New
(D)
Civil Code of the Philippines is one which
without
prior
investigation
conducted by a public prosecutor.

Never Let The Odds Keep You From Pursuing What You Know In Your Heart You Were Me
ant To Do.-Leroy Satchel Paige
Page 169 of 180

Civil Law Q&As (2007-2013)


(A)
the
hectorchristopher@yahoo.com
obligor
has
a
moral
(80)
When
dbaratbateladot@gmail.com
the
donor
gives
donations
obligation to do, otherwise entitling
without reserving sufficient funds for his
the obligee to damages.
support
(B) refers to an obligation in writing
dependents, his donations are
or
for
the
support
of
his
to do or not to do.
(A) Rescissible, since it results in
(C) the obligee may enforce through

economic lesion of more than 25% of


the court if violated by the obligor.
the value of his properties.
(D) cannot be judicially enforced
(B) Voidable, since his consent to
but
the donation is vitiated by mindless
authorizes
the
obligee
to
retain the obligors payment or
kindness.
performance.
(C) Void, since it amounts to wanton
expenditure beyond his means.
(79)
The
husband
administration
of
the
assumed
familys
sole
(D) Reducible to the extent that
mango
the
donations
impaired
the

plantation since his wife worked abroad.


support due to himself and his
Subsequently, without his wifes knowledge,
dependents.
the husband entered into an antichretic
transaction
with
a
company,
possession
and
management
giving
it
(81) Anne owed Bessy P1 million due on
the
October 1, 2011 but failed to pay her on
plantation with power to harvest and sell
due date. Bessy sent a demand letter to
the fruits and to apply the proceeds to the
Anne giving her 5 days from receipt within
payment of a loan he got. What is the
which to pay. Two days after receipt of the
standing of the contract?
letter, Anne personally offered to pay Bessy
of
(A) It is void in the absence of the
in manager s check but the latter refused to
wifes consent.
accept the same. The 5 days lapsed. May
(B) It is void absent an authorization

Annes
from the court.
extinguished?
obligation
be
considered
(C) The transaction is void and can
(A) Yes, since Bessys refusal of the
neither be ratified by the wife nor
managers
authorized by the court.
presumed funded, amounts to a
(D) It is considered a continuing
satisfaction of the obligation.
offer by the parties, perfected
(B) No, since tender of payment
only upon the wifes acceptance
even in cash, if refused, will not
or the courts authorization.
discharge the obligation without
check,
which
is
proper consignation in court.
Never Let The Odds Keep You From Pursuing What You Know In Your Heart You Were Me
ant To Do.-Leroy Satchel Paige
Page 170 of 180

Civil Law Q&As (2007-2013)


(C)
Yes,
hectorchristopher@yahoo.com
since
Anne
tendered
dbaratbateladot@gmail.com
(83) The owner of a thing cannot use it in a
payment of the full amount due.
way that will injure the right of a third
(D) No, since a managers check is
person. Thus, every building or land is
not considered legal tender in the
subject to the easement which prohibits its
Philippines.
proprietor or possessor from committing
nuisance like noise, jarring, offensive odor,
(82) The residents of a subdivision have
and smoke. This principle is known as
been using an open strip of land as passage
(A) Jus vindicandi.
to the highway for over 30 years. The owner
(B) Sic utere tuo ut alienum non
of that land decided, however, to close it in
laedas.
preparation for building his house on it.
(C) Jus dispondendi.
The residents protested, claiming that they
(D) Jus abutendi.
became

owners
of
the
land
through
acquisitive prescription, having been in
(84) Janice and Jennifer are sisters. Janice
possession of the same in the concept of
sued
owners,
and
business partner for recovery of property
continuously for more than 30 years. Is this
with damages. The complaint did not allege
claim correct?
that Janice exerted earnest efforts to come
publicly,
peacefully,
Jennifer
and
Laura,
Jennifers
(A) No, the residents have not
to a compromise with the defendants and
been in continuous possession of
that
the land since they merely passed
dismissed the complaint outright for failure
through
to comply with a condition precedent. Is the

it
in
going
to
the
highway.
such
efforts
failed.
The
judge
dismissal in order?
(B) No, the owner did not abandon
(A) No, since Laura is a stranger to
his right to the property; he merely
the sisters, Janice has no moral
tolerated his neighbors use of it for
obligation to settle with her.
passage.
(B) Yes, since court should promote
(C) Yes, residents of the subdivision
amicable
have become owners by acquisitive
relatives.
prescription.
(C) Yes, since members of the same
(D) Yes, community ownership by
family, as parties to the suit, are
prescription prevails over private
required to exert earnest efforts to

claims.
settle their disputes before coming
settlement
among
to court.
Never Let The Odds Keep You From Pursuing What You Know In Your Heart You Were Me
ant To Do.-Leroy Satchel Paige
Page 171 of 180

Civil Law Q&As (2007-2013)


hectorchristopher@yahoo.com
dbaratbateladot@gmail.com
(D) No, the family council, which
(87) When does the regime of conjugal
would
partnership of gains begin to exist?
ordinarily
mediate
the
dispute, has been eliminated under
(A) At the moment the parties
the Family Code.
take and declare each other as
husband
and
wife
before
(85) X borrowed money from a bank,
officiating officer.
secured by a mortgage on the land of Y, his
(B) At the time the spouses acquire
close friend. When the loan matured, Y
properties through joint efforts.
offered to pay the bank but it refused since
(C) On the date the future spouses
Y was not the borrower. Is the banks action
executed their marriage settlements
correct?
because this is the starting point of
(A) Yes, since X, the true borrower,

their marital relationship.


did not give his consent to Ys offer
(D) On the date agreed upon by the
to pay.
future spouses in their marriage
(B) No, since anybody can discharge
settlements since their agreement is
Xs obligation to his benefit.
the law between them.
(C) No, since Y, the owner of the
collateral, has an interest in the
(88) Josie, 18, married Dante, 25, without
payment of the obligation.
her parents knowledge and consent, and
(D) Yes, since it was X who has an
lived with him. After a year, Josie returned
obligation to the bank.
to her parents home, complained of the
unbearable battering she was getting from
(86) The right of a mortgagor in a judicial
Dante, and expressed a desire to have her
foreclosure
marriage with him annulled. Who may
property
to
after
redeem
the
mortgaged
the
bring the action?
of

the
(A) Dante.
of
the
(B) Her parents.
mortgaged property or confirmation of the
(C) Josie herself.
sale by the court, is known as
(D) The State.
performance
mortgage
of
but
his
default
the
conditions
before
the
sale
in
(A) accion publiciana.
(B) equity of redemption.
(89) X, a married man, cohabited with Y, an
(C) pacto de retro.
unmarried woman. Their relation bore them
(D) right of redemption.
BB, a baby boy. Subsequently, after X
became a widower, he married Y. Was BB
legitimated by that marriage?
Never Let The Odds Keep You From Pursuing What You Know In Your Heart You Were Me
ant To Do.-Leroy Satchel Paige

Page 172 of 180

Civil Law Q&As (2007-2013)


hectorchristopher@yahoo.com
dbaratbateladot@gmail.com
(A) Yes, since his parents are now
(D) Yes, as long as they leave
lawfully married.
sufficient property for themselves
(B) Yes, since he is an innocent
and for their dependents.
party and the marriage rectified the
wrong done him.
(92) X owed Y P1.5 million. In his will, X
(C) No, since once illegitimate, a
gave Y legacy of P1 million but the will
child
provided that this legacy is to be set off
shall
always
remain
illegitimate.
against the P1.5 million X owed Y. After the
(D) No, since his parents were not
set off, X still owed Y P500,000. Can Y still
qualified
collect this amount?
to
marry
each
other
when he was conceived.
(A) Yes, because the designation

of Y as legatee created a new and


(90) The presence of a vice of consent
separate
juridical
relationship
vitiates the consent of a party in a contract
between them, that of testatorand this renders the contract
legatee.
(A) Rescissible.
(B) It depends upon the discretion of
(B) Unenforceable.
the probate court if a claim is filed
(C) Voidable.
in the testate proceedings.
(D) Void.
(C) No, because the intention of the
testator in giving the legacy is to
(91)
Can
properties
common-law
of
spouses
substantial value
donate
to
one
another?
abrogate his entire obligation to Y.
(D)
No,

because
X
had
no
instruction in his will to deliver
(A) No, they are only allowed to
more than the legacy of P1 million to
give moderate gifts to each other
Y.
during family rejoicing.
(B) No, they cannot give anything of
(93) Josie owned a lot worth P5 million
value
prevent
prior to her marriage to Rey. Subsequently,
placing their legitimate relatives at a
their conjugal partnership spent P3 million
disadvantage.
for the construction of a house on the lot.
(C) Yes, unlike the case of legally
The construction resulted in an increase in
married spouses, such donations
the value of the house and lot to P9 million.
are not prohibited.
Who owns the house and the lot?
to
each
other
to
Never Let The Odds Keep You From Pursuing What You Know In Your Heart You Were Me
ant To Do.-Leroy Satchel Paige

Page 173 of 180

Civil Law Q&As (2007-2013)


(A)
Josie
hectorchristopher@yahoo.com
and
conjugal
(C) When fortuitous circumstances
partnership of gains will own both
prevented the plaintiff from filing the
on a 50-50 basis.
case sooner.
(B) Josie will own both since the
(D) When
value
possession of the property.
of
the
the
dbaratbateladot@gmail.com
house
and
the
the
plaintiff
is
in
increase in the propertys value is
less than her lots value; but she
(95) Conrad and Linda, both 20 years old,
is
applied for a marriage license, making it

to
reimburse
conjugal
partnership expenses.
appear
(C) Josie still owns the lot, it being
married without their parents knowledge
her
the
before an unsuspecting judge. After the
conjugal
couple has been in cohabitation for 6 years,
exclusive
house
property,
belongs
to
the
but
that they were over 25.
They
partnership.
Lindas parents filed an action to annul the
(D) The house and lot shall both
marriage on ground of lack of parental
belong to the conjugal partnership,
consent. Will the case prosper?
with Josie entitled to reimbursement
(A) No, since only the couple can
for the value of the lot.

question
the
validity
of
their
marriage after they became 21 of
(94)
An
action
for
reconveyance
of
a
age;
their
cohabitation
also
registered piece of land may be brought
convalidated the marriage.
against the owner appearing on the title
(B) No, since Lindas parents made
based on a claim that the latter merely
no allegations that earnest efforts
holds such title in trust for the plaintiff.
have been made to come to a
The action prescribes, however, within 10
compromise with Conrad and Linda
years from the registration of the deed or
and which efforts failed.
the date of the issuance of the certificate of

(C)
title of the property as long as the trust had
voidable, the couple being below 21
not been repudiated. What is the exception
years of age when they married.
to this 10-year prescriptive period?
(D) Yes, since Lindas parents never
(A) When the plaintiff had no notice
Yes,
since
the
marriage
is
gave their consent to the marriage.
of the deed or the issuance of the
certificate of title.
(96) Pepito executed a will that he and 3
(B) When the title holder concealed
attesting witnesses signed following the
the matter from the plaintiff.
formalities of law, except that the Notary
Public failed to come. Two days later, the
Never Let The Odds Keep You From Pursuing What You Know In Your Heart You Were Me
ant To Do.-Leroy Satchel Paige
Page 174 of 180

Civil Law Q&As (2007-2013)


hectorchristopher@yahoo.com
dbaratbateladot@gmail.com
Notary Public notarized the will in his law
(B) Separate since their property
office where all signatories to the will
relations with their legal spouses
acknowledged that the testator signed the
are still subsisting.
will in the presence of the witnesses and
(C) Co-ownership since they agreed
that the latter themselves signed the will in
to work for their mutual benefit.
the presence of the testator and of one
(D) Communal since they earned the
another. Was the will validly notarized?
same as common-law spouses.
(A) No, since it was not notarized on
the occasion when the signatories
(98) What is the prescriptive period for filing
affixed their signatures on the will.
an action for revocation of a donation based
(B) Yes, since the Notary Public
on acts of ingratitude of the donee?
has to be present only when the
(A) 5 years from the perfection of the
signatories acknowledged the acts
donation.
required of them in relation to
(B) 1 year from the perfection of
the will.

the donation.
(C) Yes, but the defect in the mere
(C) 4 years from the perfection of the
notarization of the will is not fatal to
donation.
its execution.
(D) Such action does not prescribe.
(D) No, since the notary public did
not require the signatories to sign
(99)
Before
Karen
married
Karl,
she
their respective attestations again.
inherited P5 million from her deceased
mother which amount she brought into the
(97) Venecio and Ester lived as common-law
marriage. She later used part of the money
spouses since both have been married to
to buy a new Mercedes Benz in her name,
other persons from whom they had been
which Karen and her husband used as a
separated
family car. Is the car a conjugal or Karens
in
Hardworking
fact
and
for

several
years.
bright,
each
earned
exclusive property?
incomes from their respective professions
(A) It is conjugal property since the
and enterprises. What is the nature of their
spouses use it as a family car.
incomes?
(B) It is Karens exclusive property
(A) Conjugal since they earned the
since it is in her name.
same while living as husband and
(C) It is conjugal property having
wife.
been bought during the marriage.
Never Let The Odds Keep You From Pursuing What You Know In Your Heart You Were Me
ant To Do.-Leroy Satchel Paige
Page 175 of 180

Civil Law Q&As (2007-2013)


(D) It
is
hectorchristopher@yahoo.com
Karens
exclusive
property since she bought it with
dbaratbateladot@gmail.com
should the lessees sue for damages? (1%)
(1). A, the owner
her own money.
(2). B, the engineer
(100) Because of Xs gross negligence, Y
suffered
injuries
that
resulted
in
the
abortion of the foetus she carried. Y sued X
for, among other damages, P1 million for
the death of a family member. Is Y entitled
to indemnity for the death of the foetus she
carried?
(A) Yes, since the foetus is already
regarded as a child from conception,
though unborn.
(B) No, since Xs would not have
known
that
the
accident
would
result in Ys abortion.
(C) No,
since
birth

determines
personality, the accident did not
result in the death of a person.
(D) Yes, since the mother believed in
her heart that she lost a child.
(3). both A & B
SUGGESTED ANSWER:
3. Both A & B.
The lessee may proceed against A for
breach of contract, and against B for tort
or statutory liability. Under Article 1654
(2) of the New Civil Code, the lessor is
obliged to make all the necessary repairs
in order to keep the leased property
suitable for the use to which it has been
devoted.
Consequently,
under
Article
1659 NCC, the proprietor of a building or
structure is responsible for the damages
resulting
from
its
total
or
partial
collapse, if it is due to lack of necessary
repairs.
Under Article 1723, NCC, the engineer
2010 Civil Law Exam MCQ
or architect who drew up the plans and
(September 12, 2010)
specifications for a building is liable for
No.II. Multiple choice.
damage if 15 years from the completion
of
the
structure

the
same
should
collapse by a reason of a defect by those
(A). A had a 4-storey building which was
plans and specifications, or due to the
constructed by Engineer B. After five years,
defects in the ground. This liability
the
developed cracks and its
maybe enforced against the architect or
stairway eventually gave way and collapsed,
engineer even by a third party who has
resulting to injuries to some lessees. Who
no privity of contract with the architect
building
or engineer under Article 2192, NCC.
Never Let The Odds Keep You From Pursuing What You Know In Your Heart You Were Me
ant To Do.-Leroy Satchel Paige
Page 176 of 180

Civil Law Q&As (2007-2013)


hectorchristopher@yahoo.com
ALTERNATIVE ANSWER:
dbaratbateladot@gmail.com
(B) O, owner of Lot A, learning that
Japanese soldiers may have buried gold
No.1. A , the owner .
and other treasures at the adjoining vacant
The lessee can sue only the lessor for
Lot
breach of contract under Article 1659 in
excavated in Lot B where she succeeded in
relation to Article 1654, NCC. The lessee
unearthing gold and precious stones. How
cannot sue the architect or the engineer
will the treasures found by O be divided?
because there was no privity of contracts
(1%)
between them. When sued, however, the
lessor may file a third party claim
against the architect or the engineer.
B
belonging
to
spouses
No. 2. B, the Engineer .
(3). 50% to O and 50% to the state
1723
the
engineer
Y,
(2). 50% to O and 50% to the spouses X

and Y
Article
&
(1). 100% to O as finder
ANOTHER ALTERNATIVE ANSWER:
Under
X
or
(4). None of the above.
architect who drew up the plans and
specifications for a building is liable for
SUGGESTED ANSWER:
damages if within 15 years from the
completion of the structure, the same
should collapse by reason of a defect in
those plans and specifications, or due to
the defects in the ground. Under Article
2192 (NCC), however, if the damages
should be the result of any of the defects
in the construction mentioned in Art
1723, NCC, the third person suffering
damages may proceed only against the
engineer
or
architect
or
contractor
within the period fixed therein. The
damages suffered by the lessee in the
problem are clearly those resulting from
defects in the construction plans or
specifications.
No. 4. None of the above.
The general rule is that the treasure
shall belong to the spouses X and Y, the
owner of Lot B. Under Article 438 (NCC),
the exception is that when the discovery
of a hidden treasure is made on the
property of another and by chance, onehalf thereof shall belong to the owner of
the
land

and
the
other
one-half
is
allowed to the finder. In the problem,
the finding of the treasure was not by
chance because O knew that the treasure
was in Lot B. While a trespasser is also
not entitled to any share, and there is no
indication in the problem whether or not
O was a trespasser, O is not entitled to a
Never Let The Odds Keep You From Pursuing What You Know In Your Heart You Were Me
ant To Do.-Leroy Satchel Paige
Page 177 of 180

Civil Law Q&As (2007-2013)


hectorchristopher@yahoo.com
mother
dbaratbateladot@gmail.com
share because the finding was not by
or
alone,
even
in
a
public
chance.
document, is not sufficient because the
father and mother did not have a special
power of attorney for the purpose. Under
(C) A executed a Deed of Donation in favor
of B, a bachelor, covering a parcel of land
valued at P1 million. B was, however, out of
the country at the time. For the donation to
be valid, (1%)
Article 745 (NCC), the donee must accept
the donation personally, or through an
authorized person with a special power
of attorney for the purpose; otherwise,
the donation shall be void.
No.3 is also false. B cannot accept the
(1). B may e-mail A accepting the donation.
(2). The donation may be accepted by Bs
father with whom he lives.
convenient to him.
attorney may accept the donation for him.
(5). None of the above is sufficient to make
Bs acceptance valid
No. 5 None of the above is sufficient to
make B s acceptance valid .
donation
must
them signed each and every page of the will.
One of the witnesses was B, the father of
one of the legatees to the will. What is the
effect of B being a witness to the will? (1%)

(2). The will is valid and effective


(3). The legacy given to Bs child is not valid
covered
an
immovable property, the donation and
acceptance
(D) A executed a 5-page notarial will before
(1). The will is invalidated
SUGGESTED ANSWER:
the
accept the donation only during the
a notary public and three witnesses. All of
(4). Bs mother who has a general power of
the
Under Article 749 NCC, the donee may
lifetime of the donor.
(3). B can accept the donation anytime
Since
donation anytime at his convenience.
be
in
public
document and e-mail is not a public
document. Hence, No.1 is false.
No. 2 and No.4 are both false. The
acceptance by the donees father alone
SUGGESTED ANSWER:
No. 3.
The legacy given to B s child is
not valid.
The validity of the will is not affected by
the legacy in favor of the son of an
Never Let The Odds Keep You From Pursuing What You Know In Your Heart You Were Me
ant To Do.-Leroy Satchel Paige
Page 178 of 180

Civil Law Q&As (2007-2013)


hectorchristopher@yahoo.com
attesting witness to the will. However,
dbaratbateladot@gmail.com
ALTERNATIVE ANSWER:
the said legacy is void under Article 823
NCC.
(D). all the above
ALTERNATIVE ANSWER:
(2). A deposit made in compliance with a
legal obligation is:
No. 2 .The will is valid and effective.
Under Article 823 ( NCC ),the legacy
given
in
favor
of
the
son
of
an
instrumental witness to a will has no
effect on the validity of the will. Hence,
(A). an extrajudicial deposit;
(B). a voluntary deposit;
(C). a necessary deposit;
the will is valid and effective.
(D). a deposit with a warehouseman;
(E). letters a and b
2007 Civil Law Exam MCQ
(September 09, 2007)
SUGGESTED ANSWER:
(C). a necessary deposit
No.IX. Multiple choice: Choose the right
answer. (2% each)
(3). A contract of antichresis is always:

(1). The parties to a bailment are the:


(A). a written contract;
(A). bailor;
(B). a contract, with a stipulation that the
debt will be paid through receipt of the
(B). bailee;
(C) comodatario;
(D). all the above;
(E). letters a and b
SUGGESTED ANSWER:
(E). letters a and b
fruits of an immovable;
(C). Involves the payment of interests, if
owing;
(D). All of the above;
(E). Letters a and b
SUGGESTED ANSWER:
Never Let The Odds Keep You From Pursuing What You Know In Your Heart You Were Me
ant To Do.-Leroy Satchel Paige
Page 179 of 180

Civil Law Q&As (2007-2013)


hectorchristopher@yahoo.com
(D). All of the above;
(D).
1/3
of
dbaratbateladot@gmail.com
the
total
debts
must
be
represented by the approving creditors;
(4). An, assignee in a proceeding under the
Insolvency Law does not have the duty of:
(E). Letters a and b
(A). suing to recover the properties of the
SUGGESTED ANSWER:
state of the insolvent debtor;
(C). 3/5 of the number of creditors
(B). selling property of the insolvent debtor;
should agree to the settlement;
(C). ensuring that a debtor corporation
[Note: Items 4&5 on Insolvency Law are
operate
and
not included within the coverage of Civil
effectively while the proceedings are
Law but Commercial Law. It is therefore
pending;
suggested that the examinees be given
the

business
efficiently
full credit for the two items regardless of
(D). collecting and discharging debts owed
their answers.]
to the insolvent debtor.
SUGGESTED ANSWER:
References:
(C). ensuring that a debtor corporation
operate
the
business
efficiently
and

Answers
to
Bar
Examination
effectively while the proceedings are
Questions by the UP LAW COMPLEX
pending;
(2007, 2009, 2010)
(5). In order to obtain approval of the
proposed settlement of the debtor in an
insolvency proceeding.

UP LAW REVIEW

PHILIPPINE ASSOCIATION OF LAW


SCHOOLS (2008)
(A). the court must initiate the proposal
(B). 2/3 of the number of creditors should

lawphil.net

agree to the settlement;


(C). 3/5 of the number of creditors
should agree to the settlement;
Never Let The Odds Keep You From Pursuing What You Know In Your Heart You Were Me
ant To Do.-Leroy Satchel Paige
Page 180 of 180

You might also like